0% found this document useful (0 votes)
565 views278 pages

5 Professional-Module 5

Uploaded by

Laica Abadesco
Copyright
© © All Rights Reserved
We take content rights seriously. If you suspect this is your content, claim it here.
Available Formats
Download as PDF, TXT or read online on Scribd
0% found this document useful (0 votes)
565 views278 pages

5 Professional-Module 5

Uploaded by

Laica Abadesco
Copyright
© © All Rights Reserved
We take content rights seriously. If you suspect this is your content, claim it here.
Available Formats
Download as PDF, TXT or read online on Scribd
You are on page 1/ 278

MODULE

IT Elective 2 HUMAN COMPUTER INTERACTION II

DIFFERENTIAL
CALCULUS

Author
ELVIRA C. CATOLOS, Ph. D.

1|Page
Republic of the Philippines
UNIVERSITY OF RIZAL SYSTEM
Province of Rizal

Course Guide
MATH 2E: DIFFERENTIAL CALCULUS

1ST Semester, AY 2020 - 2021

INTRODUCTION

I welcome you all to this online class in Differential Calculus. This is a three (3) unit course
with fifty four (54) hour time allotment the whole semester. Many were asking when they
use Calculus in the real world. Well, this subject finds applications in many disciplines like
physics, engineering, economics, statistics, and medicine. But why is it most people find
the subject very hard? Calculus by itself is an introductory course, but solution of many
problems requires ones knowledge and competencies in Algebra, Trigonometry and both
Solid and Analytic geometry, making it appears to be a tough course. But don’t you worry
guys; you will not be left alone for I will be with you all the way as your course facilitator
this 1st Semester of Academic Year 2020-2021. With the COVID-19 pandemic ravaging
the globe and with the necessity of observing physical distancing measures, the traditional
schooling experience that requires students to attend classes in person is prohibited. But,
it is truly important and is urgent indeed at this time of crisis to safeguard your generation
of students of the opportunity to learn and be educated. As mandated by law amidst this
pandemic situation, educational institutions have to go for online teaching and learning
even though its implementation posed different problems and challenges to both the
teachers and students. Educational institutions and this includes the University of Rizal
System, were given the academic freedom by the Commission on Higher Education to
implement available distance learning, e-learning and other alternative modes of delivery
to students. This is said to be the new normal in education, using blended and flexible
learning modalities. I dearly advise you all to practice hard work, perseverance, patience
and all other positive attitudes needed to facilitate understanding of the modules and
instructional videos especially developed for your needs and I do hope these learning
materials will develop in you the sense of independent learning. So, hang on guys, and
hope you will all enjoy these online learning experiences.
THE FACULTY

Hi everyone! Yours truly is DR. ELVIRA C. CATOLOS, Professor IV at the University of


Rizal System (URS)-Morong Campus. I am an engineer by profession; but, God’s plan for
me is to become a teacher, spending the 36 years of my existence passionately mentoring
and molding young minds. I got my bachelor degree in Chemical Engineering from the
University of Santo Tomas in Manila, earned my Master of Engineering Education, major
in Chemical Engineering from the University of the Philippines in Diliman, Quezon City as
an EDPITAF scholar; and, my Doctorate Degree in Educational Management from the
University of Rizal System where I was an Academic Excellence and Best Research
Award Recipient. The GURONASYON Foundation, Inc. in the Province of Rizal gave me
the recognition as the 2014 Most Outstanding College Faculty and by the Local
Government Unit of Tanay, Rizal as the Natatanging Anak ng Tanay sa Larangan ng
Edukasyon during the 2015 Parangal sa Natatanging Anak ng Tanay (PANATA). The
University of Rizal System bestowed me with the Exemplary Behaviour Award and the
Best Research Award (Developmental Category), respectively, during the September
2019 and 2012 Program on Awards and Incentives for Service Excellence (PRAISE)
awarding ceremonies. My passion for teaching becomes my catalyst to author books in
Plane and Spherical Trigonometry, Analytic Geometry, General Mathematics for Senior
High School, Mathematics in the Modern World; and to develop more instructional
materials, namely: Worktext in Precalculus, Worktext in Basic Calculus, Worktext in
Differential Calculus; and, Worktext in Integral Calculus which are all copyrighted and with
ISBN.

COURSE DESCRIPTION

Differential Calculus is an introductory course covering the core concepts of limit,


continuity and differentiability of functions involving one or more variables. This also
includes the application of differential calculations in solving problems on optimization,
rates of change, related rates, tangents and normal, curve tracing, and approximations.
COURSE OBJECTIVES
To understand the core concepts of limit, continuity and differentiability of functions
involving one or more variables, the differentiation process of algebraic and
transcendental functions and apply derivatives of functions in solving problems on rates of
change, rectilinear motion, angle of intersection of curves, optimization, time-rates,
tangents and normal; and, limit evaluation using L’ Hospital’s Rule.

SPECIFIC OBJECTIVES:

After completing this course, the students must be able to:

1. Understand concept of function, its, domain, range, range of correspondence,


graph and limit.
2. Evaluate limit of a function analytically and graphically.
3. Identify the point of discontinuity of a function.
4. Understand concept of increment and derivative
5. Solve problem on rate of change using derivative of a function.
6. Know the differentiation formulas for algebraic and transcendental functions and be
able to apply them on problems on rectilinear motion, angle of intersection of
curves, optimization, time-rates, tangents and normal; and, limit evaluation using L’
Hospital’s Rule.
7. Understand the concept of partial differentiation and apply to it in getting partial
derivative of a function in two or more variables.

COURSE PREREQUISITE

This is an introductory course, it has no pre-requisite course. However, students are


expected to be equipped with the essential knowledge and competencies in Algebra,
Trigonometry, Solid and Analytic Geometry to facilitate learning of the course.

COURSE MATERIALS

At the start of the semester, your email address will be asked from you so I can invite you
to join the Google classroom that will be created for our class and each of you needs to
log-in to have your own student account. Our class group chat will also be created to
facilitate sharing of messages and information. Furthermore, we can also enroll on the
URS Learning Management System (LMS) and log-in at agreed day and time for chat is
most welcome. This LMS is called MOODLE (Modular Object–Oriented Dynamic Learning
Environment. The MOODLE can be accessed if you log-in to http://www.urs.....and click
on the MOODLE link. Course enrolment key will be provided or can be requested from the
Technical Support of the URS Management Information System (MIS).Student should
have access to the internet since online and asynchronous discussion and sharing,
concerns and inquiries may be sent to an online platform to be identified later. Aside from
the modules especially developed for your need, instructional videos on the different
modules can be viewed at my you tube channel. And, I suggest also that you browse the
internet so you may get hold of other supplementary reading materials to enhance your
learning. All these could be of help to you while preparing your assignments and activities.
Our virtual classroom, GC and LMS will serve as the avenue so you can access with copy
of the modules especially developed for your need, copy of the course guide, can submit
all course requirements and even can take quiz and examination online.

COURSE STRUCTURE

The entire course consists of six (6) units divided into twenty (20) modules.
UNIT 1 - Function and Limit of a Function
Module 1. Relation and Function
Module 2. Function, its Continuity and Limit
UNIT 2 – Increment and Derivative of a Function
Module 3. Increment and Derivative
Module 4. Rate of Change
Unit 3 - Derivative of Algebraic Function
Module 5. Differentiation Formulas for Algebraic Function
Module 6. Slope of Tangent and Normal Line
Module 7. Angle of Intersection of Curves
Module 8. Rectilinear Motion
Module 9. Higher Order Derivative
Module 10. Implicit Differentiation
Module 11. Chain Rule of Differentiation
Module 12. Maximum and Minimum Value of Function
Module 13. Optimization Problems
Module 14. Time-Rates
UNIT 4 – Derivative of Transcendental Function
Module 15. Derivatives of Trigonometric Function and their Applications
Module 16. Derivatives of Inverse Trigonometric Function and their Applications
Module 17. Derivatives of Exponential and Logarithmic Functions and their Applications
Module 18. Derivative of a Variable Raised to Another Variable
Unit 5 – Indeterminate Forms
Module 19. Concept of Indeterminate Forms
Unit 6 – Partial Differentiation
Module 20. Concept of Partial Differentiation
COURSE SCHEDULE

DATE ACTIVITY MEDIUM


August 24, 2020 Creation of class group chat, Course LMS
Orientation/URS Mission, Vision,
Quality Policy, URS Hymn
August 26, 2020 Reading and Viewing of instructional Study From Home
video of Module 1 and Module 2
September 2, 2020 Open Forum on the submitted Zoom platform/
answers to SAQ and Activity Class group chat
September 7, 2020 Quiz No. 1 Google Classroom
September 14, 2020 Reading and Viewing of instructional Study From Home
video of Module 3 and Module 4
September 16, 2020 Open Forum on the submitted Zoom platform
answers to SAQ and Activity and Quiz Class group chat
No. 2
September 21, 2020 Reading and Viewing of instructional Study From Home
video of Module 5 and Module 6
September 23, 2020 Open Forum on the submitted Zoom platform
answers to SAQ and Activity and Quiz Class group chat
No. 3
September 28, 2020 Reading and Viewing of instructional Study From Home
video of Module 7
September 30, 2020 Preliminary Examination Google Classroom

October 5, 2020 Reading and Viewing of instructional Study From Home


video of Module 8 and Module 9
October 7, 2020 Open Forum on the submitted Zoom platform
answers to SAQ and Activity and Quiz Class group chat
No. 4 Google Classroom
October 12, 2020 Reading and Viewing of instructional Study From Home
video of Module 10 and Module 11
October 14, 2020 Open Forum on the submitted Zoom platform
answers to SAQ and Activity and Quiz Class group chat
No. 5 Google Classroom
October 19, 2020 Reading and Viewing of instructional Study From Home
video of Module 12
October 21, 2020 Open Forum on the submitted Zoom platform
answers to SAQ and Activity Class group chat
October 26, 2020 Quiz No. 6 Google Classroom
October 28, 2020 Reading and Viewing of instructional Study From Home
November 2, 2020 video of Module 13 and Module 14
November 4, 2020 Open Forum on the SAQ and Activity Zoom platform
Class group chat
November 9, 2020 Quiz No. 7 Google Classroom
November 11, 2020 Mid-term Examination Google Classroom

November 16, 2020 Reading and Viewing of instructional Study From Home
video of Module 15 and Module 16
November 18, 2020 Open Forum on the SAQ and Activity Zoom platform
Class group chat
November 23, 2020 Quiz No. 8 Google Classroom
November 25, 2020 Reading and Viewing of instructional Study From Home
video of Module 17 and Module 18
November 30, 2020 Open Forum on the SAQ and Activity Zoom platform
Class group chat
December 2, 2020 Quiz No. 9 Google Classroom
December 7, 2020 Reading and Viewing of instructional Study From Home
video of Module 19 and Module 20
December 9, 2020 Open Forum on the SAQ and Activity Zoom platform
and Quiz No. 10 Class group chat
December 11, 2020 Final Examination Google Classroom

COURSE REQUIREMENTS AND GRADING SYSTEM:


To pass the course, the students should satisfactorily pass the following requirements
according to the existing University rules and regulations. These course requirements are
to be taken/submitted using any of the online platforms like messenger, e-mail, University
LMS

CLASS STANDING .…………………….. 60%


 Quizzes …………… 30%
 Projects (Activity Sheets) …………… 20%
 Recitation …………… 10%
MAJOR EXAMINATION ……………………… 40%

TOTAL 100%

ACADEMIC INTEGRITY
Academic dishonesty: Any form of cheating or plagiarism in this course will result in zero
on the exam, assignment or project. Allowing others access to your work potentially
involves you in cheating. Working with others to produce very similar reports is plagiarism
regardless of intent.

FOR QUESTIONS AND INQUIRIES: email address: [email protected]


FB Account: Elvira Catolos
MODULE 1
RELATION AND FUNCTION

Specific Objectives:

At the end of the module, students must be able to:


1. Understand concept of function, its, domain, range,
range of correspondence and graph.

2. Differentiate a relation from a function.

3. To determine the domain and range using analytical and


graphical method.

4. Evaluate a function at a given value of the independent


variable.
UNIT 1-FUNCTION AND LIMIT OF A FUNCTION

INTRODUCTION
In our daily living, we often encounter quantities that do come in pair. For
example, the number of kilograms of rice and the amount of money needed to
purchase. Furthermore, the number of miles a car travelled and the liters of
gasoline consumed. Likewise, the plant growth in centimeters and the amount of
rainfall it received. When one quantity changed, the other also changed. These
pairings are best represented as ordered pairs.

RELATION

If we let the ordered pair be( ), we call a set of ordered pairs as a relation. The
set of all the first elements (the values of x) in the ordered pairs is referred to as
the domain of the relation while the set of all the second elements (the values of
y) forms the range. Thus, in a relation, there is a correspondence between the
domain and range, such that to each element of the domain there is assigned one
or more elements of the range.

The given mapping diagram better


explains the definition of relation, its
domain and range. This relation
consists of five ordered pairs, namely:
( )( )( ) ( )
and( ). Its domain is set
* + and its range is
set* +.

GRAPH OF A RELATION

There is a one-to-one correspondence between the ordered-pairs ( )and


the points on the rectangular or Cartesian plane. Each point on the plane
corresponds to one and only one ordered pair( ). While the domain of a relation
is usually apparent from the definition of the relation, the range is often determined
from its graph. The graph of the above relation consisting of points is shown at the
right.

Differential Calculus Module 1-Relation and Function Page 1


UNIT 1-FUNCTION AND LIMIT OF A FUNCTION

RULE OF CORRESPONDENCE

The rule of correspondence is any equation describing how the elements of


the domain and range of any relation are paired. It virtually gives the range of the
relation. Let us consider three relations described by the same rule of
correspondence but having different domains.

Example 1. Given: Relation *( )| +.

Relation is a set of ordered pairs


consisting of all the possible pairings of the
elements of the domain and range that are
formed according to the given rule of
correspondence. Hence, the elements of
relation are ordered
pairs( ) ( ) ( ). Thus, the domain
of relation is set* +, its range is
set* + and its graph consists of only three
points.

Example 2. Relation *( )| +.

Relation which can simply be denoted as *( )⌋ + consists


of an infinite number of ordered pairs. It is a general rule that if the domain is not
indicated, it means that it consists of all real numbers without any exception. Any
real number that is excluded in the domain must be clearly indicated in the
notation used. This matter is exhibited on the graph of relation which is a line
represented by linear equation y  x  2 extending indefinitely up to the right and
down to the left. The domain of relation is * | +, where the real number
set is and its range is* | +. Its graph is shown below.

Differential Calculus Module 1-Relation and Function Page 2


UNIT 1-FUNCTION AND LIMIT OF A FUNCTION

Example 3. Relation *( )| +.

It is understood that the domain of consists of all real values except .


Hence, point ( )is not in set C. This fact is revealed on the graph by drawing
an open circle around the point. Therefore, domain * | +and range * |
+belong to relation .

FUNCTION

Function is a special kind of relation. It is a set of ordered-pairs ( ) of real


numbers in which no two pairs have the same first element . Furthermore, it is a
relation in which each -element has only one -element associated with it.
Relations , and described on the above examples are all functions since for
every value of the first element , there is one and only one corresponding value
of the second element y.

VERTICAL LINE TEST

The vertical line test tells whether a relation is a function. Given the graph
of the relation, if every vertical line drawn crosses the graph in only one point,
then, the relation is a function. On the contrary, if one can draw a vertical line that
goes through two points, is not a function of .
The graph of relations and shown below reveals that is a function
since any vertical line drawn through its graph intersects it in one and only one
point. Moreover, is not a function since any vertical line drawn through its graph
crosses it in more than one point.

Graph of 𝑄 Graph of 𝑊

Differential Calculus Module 1-Relation and Function Page 3


UNIT 1-FUNCTION AND LIMIT OF A FUNCTION

CONSTANT AND VARIABLE

In Mathematics, a constant is a quantity that maintains a fixed value


throughout a particular problem. Absolute constants such as √ retain the
same values in all problems. Arbitrary constants remain constant in a particular
problem but may assume different values in other problems.
A variable is a quantity that may assume various values in the course of a
problem. In equation y  1  x , letter x whose values would be freely assumed is
called the independent variable and letter y whose value depends on the assumed
value of x is called the dependent variable.

FUNCTION NOTATION

To be able to discuss functions and their properties, we use a symbol,


usually a letter of the alphabet to stand for a function. The most often used
are . Sometimes, subscripts are employed so that, for example
and would stand for four different functions. To write a function, we
enclose the independent variable in parentheses preceded by a chosen letter. In
symbol form, ( ), read “function of ”, with the chosen letter indicating that there
exists a relationship between variable x and another variable.
In equation √ , ( ) is read is a function of , with the
Greek letter indicating a relationship between dependent variable and
independent variable , hence, the ordered pair ( )can be denoted by , ( )-
or ( √ ). Function is single-valued function.
Moreover, in function ( ) √ , ( ) is a double-valued function. For
example, when ( ) .
A function that depends on two or more independent variables is
symbolically represented in a similar manner. Hence, a function of variables and
is written as ( ) and is read function of and . The function ( ) when
and is denoted by ( ).

FUNCTION EVALUATION

This is the process of finding value of function, say ( ), given value of the
independent variable . The notation ( ) refers to the value of function
when . Likewise, in ( ), ( ) means the value of the function
when and .

Example 4. Suppose that is a function defined by the equation ( )


. Evaluate ( ) ( ) ( ) ( ) ( ) ( ) , ( )-. Draw the graph of for the
portion of the domain .

Solution: Substituting the given value of the independent variable , we have


 f (0)  (02  2(0)  3  3
 f (1)  (1) 2  2(1)  3  1  2  3  0
Differential Calculus Module 1-Relation and Function Page 4
UNIT 1-FUNCTION AND LIMIT OF A FUNCTION

 f (2)  (2) 2  2(2)  3  4  4  3  5


 f (1)  (1) 2  2(1)  3  4
 f (2)  (2) 2  2(2)  3  4  4  3  3
 f (3)  (3) 2  2(3)  3  9  6  3  0
     
f  f x   f x 2  2 x  2  x 2  2 x  3  2 x 2  2 x  3  3
2

 x 4  4 x 2  9  4 x3  6 x 2  12 x  2 x 2  4 x  6  3
 x 4  4 x3  4 x 2  16 x  12

Tabulating the x values and the corresponding y or f  x  values,


x 2 1 0 1 2 3
f ( x)  y 5 0 3 4 3 0
x, y   2,5  1,0 0,3 1,4 2,3 3,0
The graph of ( ) is a parabola with vertex at ( )

f ( x  h)  f ( x ) 1
Example 5. Find the value of , h  0 , given function f ( x)  2 .
h x
1
Solution: Evaluate f ( x  h) 
x  h2
x 2  x  h 
2
1 1

f ( x  h)  f ( x) x  h 2 x 2 x 2 x  h 2
Therefore,  
h h h

f x  h   f x  x 2  x 2  2hx  h 2  h2 x  h   (h  2 x)
   2  2
hx 2 x  h  hx x  h  x x  h 
2 2 2
h

Example 6. Discuss the distinction between the given functions ( )and ( )


9  x2
defined H ( x)  , and, G ( x)  3  x
3 x

Differential Calculus Module 1-Relation and Function Page 5


UNIT 1-FUNCTION AND LIMIT OF A FUNCTION

Solution:
At the first glance, it appears that the functions are the same since 9  x 2 is
factorable. However, the domain of G  x  is x  R , meaning, x is any real number.
However, for the function H  x , the values of both numerator and denominator are
zero when x  3 . Therefore, ( ) and ( ) are identical for all x -values except
x  3 . The graph of H x  has an open circle drawn around the point ( ) since
this point does not lie on its graph.

9− 𝑥 2
Graph of 𝐺(𝑥) 𝑥 Graph of 𝐻(𝑥) +𝑥
𝑥

Example 7: Find the domain and range of function  ( x)  x  4 .


Solution:
The function  ( x)  x  4 is defined only at x-values equal or greater than
4. That is, for the function to be a real number, the radicand x  4  0 or x  4
Hence, the domain of the function is x | x  4.

The definition of the given function shows that at values of in the


interval corresponding value of the function is zero or more than zero. That
is same as saying the range of the function is* | +. The graph of function is
the upper half of the parabola with vertex at( ).

Differential Calculus Module 1-Relation and Function Page 6


UNIT 1-FUNCTION AND LIMIT OF A FUNCTION

Example 8. Find the domain and range for the function defined as f ( x)  x 2  2 ,
for .
Solution:
The domain of the function f x   x 2  2 is x | 2  x  1 . To find the range,
when x  2 , f (2)  6. It could be observed from the graph that range is all real
numbers more than but less than In symbol form, range is y | 2  y  6 . The
graph has an open circle at  2,6  indicating that the domain excludes
and the range does not include .
The graph of the function is a portion of parabola ( ) having
vertex at ( ) opening upward.

Note: If the rule of correspondence defining a given function does not explicitly
point out the domain, one should be sharp enough to identify it. Say for
x
example, f ( x)  2 is a function defined for all values of except ,
x 4
since division by zero is undefined. Similarly, if h( x)  1  x 2 , the domain
consists of values that satisfy the quadratic inequality . Solution
of this inequality and the domain of the function is the interval  1  x  1 . The
graph of the function is the upper half of the circle having center at the origin
and of radius equal to one.

PIECEWISE-DEFINED FUNCTION

This is a function whose domain is divided into parts and each part is
defined by a different function rule. It is defined on a series of intervals. The word
piecewise is used to describe any property of a piecewise-defined function that
holds for each piece but may not hold for the whole domain of the function.
A common piecewise-defined function is the absolute value.

| | {

Differential Calculus Module 1-Relation and Function Page 7


UNIT 1-FUNCTION AND LIMIT OF A FUNCTION

Example 9. Find the domain and range of given piecewise-defined function ( ).


Draw its graph and find value of when and .

( ) 2

Solution: Based on the given parts of the domain, we say that the domain of the
given piecewise-defined function is x | x  R. Let us draw the graph of the
given function ( ).

The graph above shows the range of ( ) is * ⌋ + and


when , ( ) . Furthermore, when , ( ) .

Example 10. Find the domain and range of function defined below and draw its
graph.
- x 2  1 , x  1

H ( x)   1 ,x 0
 x  2 , x  1
2

The domain of the given function is * | +. Below is
the graph of ( ). Based on the graph of the function ( ) shown below, it is
evident that the range is * | +.

Differential Calculus Module 1-Relation and Function Page 8


UNIT 1-FUNCTION AND LIMIT OF A FUNCTION

Example 11: Find the domain and range of piecewise-defined function ( )


whose graph is shown below. Evaluate H 2 .

Solution: Domain * | + Range is * | +. From the


given graph, the value of the function when is ( ) .

Example 12. Find the domain and range of the function graphed below.

Solution: The domain is * | + while the range is * | +.

Example 13. Find the domain and range of y   25  x 2 .


Solution: The graph of the function is the lower half of circle having
its center at the origin ( ) and radius equal to 5. For the value of to be
real, . This inequality has solution  5  x  5 . Hence, the domain of the
function is * | +. The value are excluded on the
domain as indicated by the open circles at those values of . And from its graph
below, it is evident that the range of the function is * | +.

Differential Calculus Module 1-Relation and Function Page 9


UNIT 1-FUNCTION AND LIMIT OF A FUNCTION

SAQ1

ACTIVITY 1.1 – A

NAME: ____________________________________________________ SCORE: ______________

SECTION: ___________DATE: _______________ PROF: __________________________________

Identify which of the following representations is/are a function and not a function. Write F if a
function and NF if not a function on the space provided before each number.

__________ 1. *( ) ( )( )( )+

__________ 2. *( ) ( )( )( )( )( )+

__________ 3. *( )( )( )( )+

__________ 4. {*( )| +}

__________ 5. {*( )| +}

__________ 6. 2{( )| √ }3

__________ 7. {2( )| 3}

+
__________ 8. {2( )| 2− 3}

__________ 9. {*( )| +}

__________ 10. 2{( )| √ }3

Differential Calculus Module 1-Relation and Function Page 10


UNIT 1-FUNCTION AND LIMIT OF A FUNCTION

ASAQ1

ACTIVITY 1.1 – A

NAME: ____________________________________________________ SCORE: ______________

SECTION: ___________DATE: _______________ PROF: __________________________________

Identify which of the following representations is/are a function and not a ANSWER

function. Write F if a function and NF if not a function on the space provided

before each number.

__________ 1. *( ) ( )( )( )+ F

__________ 2. *( ) ( )( )( )( )( )+ F

__________ 3. *( )( )( )( )+ NF

__________ 4. {*( )| +} F

__________ 5. {*( )| +} F

F
__________ 6. 2{( )| √ }3
F
__________ 7. {2( )| 3}
F
+
__________ 8. {2( )| 2− 3} F

__________ 9. {*( )| +} NF

NF
__________ 10. 2{( )| √ }3

Differential Calculus Module 1-Relation and Function Page 11


UNIT 1-FUNCTION AND LIMIT OF A FUNCTION

SAQ2

ACTIVITY 1.1 – B

NAME: ____________________________________________________ SCORE: ______________

SECTION: ___________DATE: _______________ PROF: __________________________________

Given the graph of a relation, determine its domain and range. Write answer on the space
provided under the given graph.
1. 2.

Domain:______________________________ Domain:____________________________

Range:________________________________Range: ______________________________

3 4.

Domain:______________________________Domain: ____________________________

Range: _______________________________Range: ______________________________

Differential Calculus Module 1-Relation and Function Page 12


UNIT 1-FUNCTION AND LIMIT OF A FUNCTION

ASAQ2

ACTIVITY 1.1 – B

NAME: ____________________________________________________ SCORE: ______________

SECTION: ___________DATE: _______________ PROF: __________________________________

Given the graph of a relation, determine its domain and range.


1. 2.

Domain: * | + Domain: : * | +

Range: * | + Range: * | +

3 4.

Domain: * | + Domain: * | +

Range: * | + Range: * | +

Differential Calculus Module 1-Relation and Function Page 13


UNIT 1-FUNCTION AND LIMIT OF A FUNCTION

ACTIVITY 1.1 – C

NAME: ____________________________________________________ SCORE: ______________

SECTION: ___________DATE: _______________ PROF: __________________________________

Given the following relations, identify the domain and range and draw their graphs.
1. A  3,6 , 0,3,  2,1,  4,1


2. B  x, y  y  2 x  4 


3. C  x, y  y  2 x  4, x  1 

 1
4. D   x, y  y  
 x


5. E  x, y  y  x3 

Differential Calculus Module 1-Relation and Function Page 14


UNIT 1-FUNCTION AND LIMIT OF A FUNCTION

6. *( )| +


7. G  x, y  y   3  2 x 


8. H  x, y  y  4 x  1 


9. I  x, y  x 2  y 2  4


10. J  x, y  y  24  2 x  x 2 

Differential Calculus Module 1-Relation and Function Page 15


UNIT 1-FUNCTION AND LIMIT OF A FUNCTION

ACTIVITY 1.1 – D

NAME: ____________________________________________________ SCORE: ______________

SECTION: ___________DATE: _______________ PROF: __________________________________

Given the piecewise-defined functions, draw the graph and identify the domain and range.

1. ( ) 2

2. ( ) {

3. ( ) {

Differential Calculus Module 1-Relation and Function Page 16


UNIT 1-FUNCTION AND LIMIT OF A FUNCTION

ACTIVITY 1.1 – E

NAME: ____________________________________________________ SCORE: ______________

SECTION: ___________DATE: _______________ PROF: __________________________________

Given the graph of piecewise-defined functions, determine its domain and range.

1. 2.

Domain: _______________________________ Domain: ________________________________

Range: _________________________________ Range: _________________________________

3. 4.

Domain: _______________________________ Domain: ________________________________

Range: _________________________________ Range: _________________________________

Differential Calculus Module 1-Relation and Function Page 17


UNIT 1-FUNCTION AND LIMIT OF A FUNCTION

ACTIVITY 1.1 – F

NAME: ____________________________________________________ SCORE: ______________

SECTION: ___________DATE: _______________ PROF: __________________________________

Evaluate the given functions at the indicated values of .


1. Given: f ( x)  2  x , find:
2

a. f (3) = _____ c. f  1 = _____ e. f 1 = _____


b. f  2  = _____ d. f 0  = _______ f. f 2  = _____
Draw the graph of f (x) for  3  x  2 .

2. Given: g ( x)  x  2 x  1 , find:
2

a. g (4) = _____ c. g  2  = _____e. g 0  = _____ g. g 2  = _____ i. g 4  = _______


b. g  3 = _____ d. g  1 = _____ f. g 1 = _____ h. g 3 = _____ j. g (a  1) = ____
Draw the graph of g (x) for  4  x  4 .

3x  4
3. Given:  ( x)  , find:
2x  3
a.  (4) = _____ d.   1 = _____ g  2  = _____
b.   3 = _____ e.  0  = _____ h.  3 = _____
c.   2  = _____ f.  1 = _____ i.  4  = _____
Which value of x is not an element of the domain? Draw the graph of   x  for x on  4,4 using
the values above and additional values, if needed.

Differential Calculus Module 1-Relation and Function Page 18


UNIT 1-FUNCTION AND LIMIT OF A FUNCTION

ACTIVITY 1.1 – G

NAME: ____________________________________________________ SCORE: ______________

SECTION: ___________DATE: _______________ PROF: __________________________________

x 1  3 
1. Given the function g ( x)  x and h( x)  , find hg (x) and g h( ) .
2

1 x  2 

2. Given: f ( x)  x  x  4 , find , (
2
)-.

x 1
3. Given the function g ( x)  x and h( x) 
2
, find , ( )- and 0 . /1.
1 x

Differential Calculus Module 1-Relation and Function Page 19


UNIT 1-FUNCTION AND LIMIT OF A FUNCTION

ACTIVITY 1.1 – H

NAME: ____________________________________________________ SCORE: ______________

SECTION: ___________DATE: _______________ PROF: __________________________________

2x2
A. Prove: If f x   , show that f ( x)  f ( x) .
x 4  16

B. If r ( x) 
( )− ( )
x , show that − ( )+ ( )
.

C. Given h( x)  x 2  4 x  5 , what is the domain of function h ? Plot the graph of h for x values
in the interval  2,6  .

Differential Calculus Module 1-Relation and Function Page 20


MODULE 2
CONTINUITY AND LIMIT OF A
FUNCTION

Specific Objectives:

At the end of the module, students must be able to:


1. Understand concept of continuity and limit of a function.

2. Evaluate limit of a function, given the rule of


correspondence.

3. Identify the limit of a function, given its graph.


UNIT 1-FUNCTION AND LIMIT OF A FUNCTION

LIMIT OF A FUNCTION

In Calculus, the idea of limit is very important. The concept of limit is at the
foundation of almost all mathematical analysis, and an understanding of it is absolutely
essential. Deep understanding of limit is very rewarding since it facilitates a good grasp
of all the basic processes of Calculus.
x2  x  2
Let us consider a particular function, say f ( x)  . This function is
x 1
defined for all values of x except x  1 since at x  1 both numerator and denominator
0
take zero value or f ( x)  , which is a meaningless expression. We will study how the
0
function f behaves when we assume values of x getting closer and closer to 1. There
are two ways by which value of x may approach 1, one is by assuming values less than
1 and approaching 1; the other way is by taking values greater than 1, still approaching 1.

To get a better idea of what is happening as x takes values approaching 1,


consider the constructed Table 1 at the right.
Table 1
By means of factoring, we can write f (x) in the form
x2  x  2
f ( x) 
x  2x  1 x f ( x ) 
x 1
x 1 0 2
If x  1 , we are allowed to divide both numerator 0.50 2.50
and denominator by  x  1 .Therefore, f ( x)  x  2 , 0.25 2.25
provided x  1 . 0.80 2.80
0.90 2.90
The value of this function as seen on the Table 1 0.99 2.99
approaches a value of 3 when the variable x 0.999 2.999
approaches 1 by assuming values of x less than 1. 0.9999 2.9999
That is, as 0.99999 2.99999
→ , → 3, provided, 0.999999 2.999999

In symbol form, lim− 𝑓 𝑥 = 3. This is read “limit of f (x) as x approaches 1 through


𝑥→1
values less than 1 is equal to 3”. The value of the limit of the function, in particular, is
called the Left-Hand Limit.

We observe that as x gets closer and closer to 1, f (x) gets closer and closer to
3; and the closer x is to 1, the closer f (x) is to 3. We can see that we can make the
value of f (x) as close to 3 as we please by taking x close enough to 1. Another way of
saying this is that we can make the absolute value of the difference between f (x) and 3
as small as we please by making the absolute value of the difference between x and 1
small enough. That is, f ( x)  3 can be made as small as we please by making x  1
small enough. But bear in mind that f(x) never takes on the value 3.
It is apparent that f (x) can be made as close to 3 as we please by taking x
sufficiently close to 1, and this property of the function f does not depend on f being
Differential Calculus Module 2-Continuity and Limit of a Function Page 21
UNIT 1-FUNCTION AND LIMIT OF A FUNCTION

defined when x  1 . This fact gives the distinction between limit of f (x) as x
approaches 1 and the function value at x  1 ; that is limit of f (x) as x approaches 1 is
3, but f (1) does not exist. Take note that the limit of the function as x approaches a
certain value, say for example x  a may not be the function value when x  a .
Similarly, taking a look at Table 2, when x approaches 1 through values greater
than 1, the value of the function f gets closer and closer to 3 but not equal to 3. That is,
when x  1 , f ( x)  3 , provided in symbol form, lim+ 𝑓 𝑥 = 3
𝑥→1

This is read “the limit of f (x) as x approaches 1 through values greater than 1 is equal
to 3”. This resulting value of the limit of f (x) as x  1 is specifically called the Right-
Hand Limit of the function.

The graph of the function f appears to be a straight line with a “hole” (an open circle) at
the point 1,3 , that is, at = , f 1  3 . Table 2

x x2  x  2
f ( x) 
x 1
2 4
1.5 3.5
1.25 3.25
1.10 3.10
1.01 3.01
1.001 3.001
1.0001 3.0001
1.00001 3.00001
1.000001 3.000001
1.0000001 3.0000001

DEFINITION OF LIMIT OF A FUNCTION

Given a function f and numbers and L , we say that f (x) approaches L as a


limit as approaches a if for each positive number (read “epsilon”) there is appositive
number  (read “delta”) such that f (x) is defined and
f ( x)  L   , whenever 0  x  a   ,   0 and   0 .
In abbreviated notation, for the definition of limit, we write f ( x)  L as x  a . (This
means “ x nears but is never equal to a .) In symbol form, lim 𝑓 𝑥 = 𝐿 .
𝑥→𝑎

The values of are arbitrarily chosen and can be as small as desired, and that
the value of  is dependent on the chosen. It should be pointed out that the smaller
the value of  , the smaller will be the corresponding value of  .

Differential Calculus Module 2-Continuity and Limit of a Function Page 22


UNIT 1-FUNCTION AND LIMIT OF A FUNCTION

The above definition states that the function values f (x) approach a limit L as x
approaches a number if the absolute value of the difference between f (x) and L can
be made as small as we please by taking x sufficiently near a but not equal to a .

Moreover, it is important to realize that the above definition does not mention
about the value of the function when x  a . That is, it is not essential that the function is
defined for x  a in order for the limit to exist.
In addition, even if the function is defined for x  a , it is possible for the limit of
f (x) to exist even without having the same value for f a  .

THEOREMS ON LIMIT OF A FUNCTION


If lim 𝑓 𝑥 = 𝐴 and lim 𝑔 𝑥 = 𝐵 then
𝑥→𝑎 𝑥→𝑎
1. lim 𝑐 = 𝑐 , where
𝑥→𝑎

2. lim 𝑥 = 𝑎
𝑥→𝑎

lim 𝑓 𝑥 ± 𝑔 𝑥 = lim 𝑓 𝑥 + lim 𝑔 𝑥 = 𝐴 ± 𝐵


3. 𝑥→𝑎 𝑥→𝑎 𝑥→𝑎

lim 𝑓 𝑥 ∙ 𝑔 𝑥 = lim 𝑓 𝑥 ∙ lim 𝑔 𝑥 = 𝐴 ∙ 𝐵


4. 𝑥→𝑎 𝑥→𝑎 𝑥→𝑎

𝑓 𝑥 lim 𝑓 𝑥 𝐴
5. lim = 𝑥→𝑎 = 𝑝𝑟𝑜𝑣𝑖𝑑𝑒𝑑 𝐵 0
𝑥→𝑎 𝑔 𝑥 lim 𝑔 𝑥 𝐵
𝑥→𝑎
𝑛
𝑛 𝑛
6. lim 𝑓 𝑥 = lim 𝑓 𝑥 = 𝐴
𝑥→𝑎 𝑥→𝑎

𝑛 𝑛 𝑛
lim 𝑓 𝑥 = lim 𝑓 𝑥 = 𝐴
𝑥→𝑎 𝑥→𝑎
7.

Note: The symbols lim+ 𝑓 𝑥 and lim− 𝑓 𝑥 are referred to as the One -Sided Limit
𝑥→𝑎 𝑥→𝑎
of .
However, 𝑥→𝑎lim 𝑓 𝑥 is called the Two-Sided Limit of .

Theorem: The two-sided limit of the function as approaches say value exists if both
the one-sided limits of the function exist and are equal. That is,
𝐥𝐢𝐦 𝒇 𝒙 = 𝐥𝐢𝐦+ 𝒇 𝒙 = 𝐥𝐢𝐦− 𝒇 𝒙
𝒙→𝒂 𝒙→𝒂 𝒙→𝒂

Example 13. Evaluate the following limits.

a. lim 3𝑥 −
𝑥→ 2

Use Theorem 1. lim 3𝑥 − = lim 3𝑥 − lim


𝑥→ 2 𝑥→ 2 𝑥→ 2

Differential Calculus Module 2-Continuity and Limit of a Function Page 23


UNIT 1-FUNCTION AND LIMIT OF A FUNCTION

lim 3𝑥 −
Use Theorems 1 and 4. 𝑥→ = lim 3 ∙ lim 𝑥 −
2 𝑥→ 2 𝑥→ 2

Use Theorems 1 and 2. lim 3𝑥 − =3 − − = −6 − = −7


𝑥→ 2

𝑥
b. lim
𝑥→3 3 − 5𝑥
𝑥 lim 𝑥
𝑥→3
Use Theorem 5. lim =
𝑥→3 3 − 5𝑥 lim 3 − 5𝑥
𝑥→3

𝑥 3
lim
Use Theorems 2 and 3.𝑥→3 =
3 − 5𝑥 lim 3 − lim 5𝑥
𝑥→3 𝑥→3

𝑥 3 3 3 3
Use Theorems 1 and 4. lim = = = = =−
𝑥→3 3 − 5𝑥 3 − lim 5 ∙ lim 𝑥 3 − 5 3 3− 5 − 4
𝑥→3 𝑥→3

4 − 𝑥2
lim
c. 𝑥→1 + 𝑥3
4 − 𝑥2 4 − 𝑥2
Use Theorem 7. lim = lim
𝑥→1 + 𝑥3 𝑥→1 + 𝑥 3

4 − 𝑥2 lim 4 − 𝑥 2 lim 4 − lim 𝑥 2


𝑥→1 𝑥→1 𝑥→1
Use Theorem 5 and 3. lim = =
𝑥→1 + 𝑥3 lim + 𝑥3 lim + lim 𝑥 3
𝑥→1 𝑥→1 𝑥→1

2
4− 𝑥2 4 − lim 𝑥 4− 2 3 6
𝑥→1
Use Theorems 1 and 6. lim = 3 = = =
𝑥→1 + 𝑥3 + 3
+ lim 𝑥
𝑥→1

Take note that the answers to the examples presented above were obtained by directly
substituting the value approached by the variable. Consider now the following illustrative
examples.

3𝑥 2 − 𝑥 − 0
d. lim
𝑥→2 𝑥2 − 4
3𝑥 2 − 𝑥 − 0 0
Solution: By direct substitution method, lim =
𝑥→2 𝑥2 − 4 0
0
,
However, the expression is an indeterminate. which is an
0

Eliminate the indeterminate form by factoring both the numerator and denominator. The
purpose of which is to remove factor from the numerator and denominator that has zero
value at = .

Differential Calculus Module 2-Continuity and Limit of a Function Page 24


UNIT 1-FUNCTION AND LIMIT OF A FUNCTION

𝑥− 3𝑥 + 5
= lim
𝑥→2 𝑥− 𝑥+
3𝑥 + 5 3 +5
= lim = =
𝑥→2 𝑥+ + 4

𝑥4 − 8
e. lim
𝑥→3 𝑥 2 − 5𝑥 − 3

𝑥4 − 8 0
Solution: lim 2
=
𝑥→3 𝑥 − 5𝑥 − 3 0
.
The factor x  3 which is zero when x  3 needs to be eliminated from the numerator
and denominator.
𝑥2 − 9 𝑥2 + 9 𝑥 − 3 𝑥 + 3 𝑥2 + 9 𝑥 + 3 𝑥2 + 9
= lim = lim = lim
𝑥→3 𝑥−3 𝑥+ 𝑥→3 𝑥−3 𝑥+ 𝑥→3 𝑥+
3+3 3 2+9 6 9+9 6 8 08
= = = =
3 + 7 7 7

3− 𝑥+5
f. lim
𝑥→4 𝑥−4

3− 𝑥+5 0
Solution: lim =
𝑥→4 𝑥−4 0
Factor x  4 must be removed from the numerator and denominator. To do it, multiply
the members of the fraction by the conjugate of the numerator to eliminate the radical of
index two. Then, recall the product of a sum and difference of two terms:
a  ba  b  a 2  b 2 .
3− 𝑥+5 3+ 𝑥+5 9− 𝑥+5 4−𝑥
= lim ∙ = lim = lim
𝑥→4 𝑥−4 3 + 𝑥 + 5 𝑥→4 𝑥 − 4 3 + 𝑥 + 5 𝑥→4 𝑥 − 4 3 + 𝑥 + 5

− 𝑥−4 − −
= lim = lim = =−
𝑥→4 𝑥−4 3+ 𝑥+5 𝑥→4 3 + 𝑥+5 3+3 6

g. 𝑥 3 − 7𝑥
lim
𝑥→0 𝑥3
𝑥 3 − 7𝑥 0
Solution: lim =
𝑥→0 𝑥3 0
Eliminate since it is the cause of zero value on the numerator and denominator.
𝑥 𝑥2 − 7 𝑥 2 − 7 −7 7
= lim 3
= lim 2
= = − = −∞
𝑥→0 𝑥 𝑥→0 𝑥 0 0

Differential Calculus Module 2-Continuity and Limit of a Function Page 25


UNIT 1-FUNCTION AND LIMIT OF A FUNCTION

Note: The numerator approaches  7 and the denominator is a positive quantity


approaching 0. The quantity   is NOT a real number and is NOT an
indeterminate form. Hence, the limit of the given function does not exist.

cos 𝑥 −
h. lim
𝑥→0 𝑐𝑜𝑠𝑥 −

𝑐𝑜𝑠 𝑥 − 0
Solution: lim =
𝑥→0 𝑐𝑜𝑠𝑥 − 0
Recall the trigonometric identity: cos 2 x  2 cos 2 x  1

Substitution into the given expression results to:


𝑐𝑜𝑠 2 𝑥 − − 𝑐𝑜𝑠 2 𝑥 −
= lim = lim
𝑥→0 𝑐𝑜𝑠𝑥 − 𝑥→0 𝑐𝑜𝑠𝑥 −
2

= lim
→0 −

+ −
= lim
→0 −
= lim 𝑐𝑜𝑠𝑥 + = + =4
𝑥→0

Differential Calculus Module 2-Continuity and Limit of a Function Page 26


UNIT 1-FUNCTION AND LIMIT OF A FUNCTION

SAQ3

ACTIVITY 1.1 – I

NAME: ____________________________________________________ SCORE: ______________

SECTION: ___________DATE: _______________ PROF: __________________________________

Evaluate the limit of the given functions.

1. 4𝑥 − 5
lim
𝑥→3 5𝑥 −

𝑥+
2. lim
𝑥→ 1 𝑥 2 − 3𝑥 + 4

3. 𝑥 2 − 3𝑥 + 4
lim
𝑥→4 𝑥2 − 𝑥 −

4𝑥 2 − 9
4. lim
3 𝑥+3
𝑥→
2

Differential Calculus Module 2-Continuity and Limit of a Function Page 27


UNIT 1-FUNCTION AND LIMIT OF A FUNCTION

ASAQ3

ACTIVITY 1.2 – I

NAME: ____________________________________________________ SCORE: ______________

SECTION: ___________DATE: _______________ PROF: __________________________________

Evaluate the limit of the given functions using the direct substitution method.

4𝑥 − 5
1. lim 𝐴𝑛𝑠𝑤𝑒𝑟:
𝑥→3 5𝑥 −

𝑥+
2. lim 𝐴𝑛𝑠𝑤𝑒𝑟: −
𝑥→ 1 𝑥 2 − 3𝑥 + 4 8

𝑥 2 − 3𝑥 + 4 8
3. lim 𝐴𝑛𝑠𝑤𝑒𝑟:
𝑥→4 𝑥2 − 𝑥 − 7

4𝑥 2 − 9
4. lim 𝐴𝑠𝑤𝑒𝑟: − 6
𝑥→
3 𝑥+3
2

Differential Calculus Module 2-Continuity and Limit of a Function Page 28


UNIT 1-FUNCTION AND LIMIT OF A FUNCTION

ACTIVITY 1.2 – J

NAME: ____________________________________________________ SCORE: ______________

SECTION: ___________DATE: _______________ PROF: __________________________________

Evaluate the limit of the given functions.

1. 3𝑥 2 − 7𝑥 + 0
lim
𝑥→4 4𝑥 2 − 5𝑥 + 36

𝑡3 −
2. lim
𝑡→1 𝑡 −

𝑥−
3. lim
𝑥→1 𝑥−

4. ℎ+5−
lim
ℎ∙→ 1 ℎ+

Differential Calculus Module 2-Continuity and Limit of a Function Page 29


UNIT 1-FUNCTION AND LIMIT OF A FUNCTION

3
𝑥−
5. lim
𝑥→1 𝑥−

𝑥3 − 𝑥2 − 𝑥 + 0
6. lim
𝑥→ 2 𝑥 2 + 3𝑥 +

𝑥2 − 𝑥 − 3
7. lim
𝑥→ 1 𝑥 3 + 𝑥 2 + 6𝑥 + 5

𝑦 3 − 𝑦 2 + 0𝑦 + 8
8. lim
𝑦→4 3𝑦 3 − 7𝑦 2 + 6𝑦 + 6

Differential Calculus Module 2-Continuity and Limit of a Function Page 30


UNIT 1-FUNCTION AND LIMIT OF A FUNCTION

LIMIT OF A FUNCTION INVOLVING INFINITY


1
If we consider the function f ( x)  , it is an observation that as x  0 through
x
positive values, the corresponding values of the function get bigger and bigger. In case
like this, it indicates the behaviour of the function. We say that f (x) increases without
limit or f (x)   as x  0  . In symbol form:

lim+ f x = +∞
x→0

Likewise, as x  0 through negative values, the value of the function decreases


without limit. Thus, in symbol from: lim− 𝑓 𝑥 = −∞
𝑥→0

The introduction of the symbol  does not in any way justify its use as a number.
It is best to mention that the word “infinite” signifies only a state of being non-finite.
Division by zero is a meaningless operation and it is not intended that the symbol 
1
represents . Once again, it is to be stressed out that   is not a symbol for a real
0
number.

When the limit of the function as approaches a certain value, say a , is positive
infinity, we say that the limit of the function does not exist. The symbol   indicates the
behaviour of the function as gets closer and closer to value .

In the same manner, getting   for the limit of the function simply indicates that
the behaviour of the function whose function values decrease without bound. Getting
  once again tells us that the limit of the function does not exist.
1
The behaviour of the function f ( x)  is graphically shown below.
x

lim = +∞
𝑥→0+ 𝑥

lim = −∞
𝑥→0− 𝑥

1

Example 14. lim− 4𝑥 = 4 = = =0
𝑥→0 4∞ ∞

1
lim+ 4𝑥 = 4+∞ = +∞
Example 15. 𝑥→0

Differential Calculus Module 2-Continuity and Limit of a Function Page 31


UNIT 1-FUNCTION AND LIMIT OF A FUNCTION

Theorems on Limit of a Function Involving Infinity

lim 𝑐𝑥 = ∞ 𝑐 > 0
1. 𝑥→+∞
lim 𝑐𝑥 = −∞ 𝑐 < 0
2. 𝑥→+∞
𝑐
lim
3. 𝑥→±∞ =0
𝑥
𝑐
4. lim+ = +∞ 𝑐 > 0
𝑥→0 𝑥

𝑐
lim − = −∞ 𝑐 > 0
5. 𝑥→0 𝑥

Example 16. Evaluate the following:


4𝑥 − 5
a. lim
𝑥→+∞ 6𝑥 + 7


Solution: Substitution of +∞ for results to the indeterminate form ∞. In case like this,
we use a standard technique in working with infinite limits by dividing each term on the
numerator and denominator by the highest power of the variable . Then, use Theorem
3 on limit of function involving infinity. Thus,
5
4𝑥 − 5 4−𝑥 7
= lim = lim provided 𝑥 0 and 𝑥 −
𝑥→+∞ 6𝑥 + 7 𝑥→+∞ 6 + 7 6
𝑥
4𝑥 − 5 4−0 4
= lim = = =
𝑥→+∞ 6𝑥 + 7 6+0 6 3

4𝑥 + 3
b. =
lim
𝑥→ 3
∞ 3𝑥 2 +


Solution: The limit takes the indeterminate form− ∞. Use the technique described on
the previous illustrative example by dividing both numerator and denominator by 2 and
then using Theorem 3.
4 3
4𝑥 + 3 𝑥 + 𝑥2
= lim 2
= lim
𝑥→ ∞ 3𝑥 + 𝑥→ ∞
3+ 2
𝑥
0+0 0
= = =0=0
3+0 3

Differential Calculus Module 2-Continuity and Limit of a Function Page 32


UNIT 1-FUNCTION AND LIMIT OF A FUNCTION

6𝑥 3 + 𝑥 2 + 𝑥 −
c. : lim
𝑥→+∞ 𝑥2 + 𝑥 +

Solution: Divide each term on the numerator and denominator by 3 , the highest
power of and then use Theorem 3 since the evaluated limit of the given

function equals∞. Hence,

6𝑥 3 + 𝑥 2 + 𝑥 − 6+𝑥+ −
= lim = lim 𝑥2 𝑥 3 = 6 + 0 + 0 + 0 = 6 = +∞
𝑥→+∞ 𝑥2 + 𝑥 + 𝑥→∞
+ + 0+0+0 0
𝑥 𝑥2 𝑥3

Note: In illustrative Example 15, a, b and c, you will observe that in evaluating limit of
function of the form as x approaches ±∞, if:

i. The degree of the numerator equals the degree of the denominator; the limit
of as approaches +∞ or − ∞ equals the ratio of the numerical coefficient of
the highest power of on the numerator to the numerical coefficient of the
highest power of on the denominator.
ii. The degree of the numerator is less than the degree of the denominator; the limit
of as approaches +∞ or − ∞ equals zero.
iii. The degree of the numerator is greater than the degree of the denominator, the
limit of as approaches +∞ or − ∞ equals either  or   as the case may be.

d. lim 𝑥=0
𝑥→0+

Take note that in evaluating the limit of the given function as → 0+ , the values taken
by are all greater than zero but approaching zero.

lim− 𝑥 = does not exist


e. 𝑥→0

The limit of the given function as → 0 does not exist since the values taken by are
all less than zero but approaching zero. Hence, the corresponding values of the given
function are imaginary or not real numbers.
5
f. lim = +∞
𝑥→4 + 𝑥 −4
As takes values greater than 4 but approaching 4, the denominator x  4 is always
greater than zero but approaching zero.

Differential Calculus Module 2-Continuity and Limit of a Function Page 33


UNIT 1-FUNCTION AND LIMIT OF A FUNCTION

5
g. lim− = −∞
𝑥→4 𝑥−4
However, when x assumes values less than 4 but approaching 4, the denominator
x  4 takes values less than zero but approaching zero.
CONTINUITY OF A FUNCTION
A function f (x) is said to be continuous at = if and only if the following
conditions are satisfied by the given function:

1. exists;

lim 𝑓 𝑥 exists; and,


2. 𝑥→𝑎

3. 𝑓 𝑎 = 𝑥→𝑎
lim 𝑓 𝑥

Consider the graph of function f (x) shown below. Observe that at x  1 the function is
discontinuous since the conditions for the continuity of a function is not satisfied. The
function when = , that is lim 𝑓 𝑥 =
= while 𝑥→1

Therefore, = is the point of discontinuity of the given function since 𝑓 lim 𝑓 𝑥 .


𝑥→1

Remember: A rational function in x is a continuous function for all values of x except


those values for which the denominator is zero.

Example 17. Find the point/s of discontinuity of the following functions. State the
condition/s of continuity of a function, if any, which is/are not satisfied.
x 1
a. f ( x) 
x3
The point of discontinuity of f (x) is at x  3 since the denominator on the rule of
correspondence of f (x) equals zero at = 3. Hence, f (3) does not exist or is undefined.
lim 𝑓 𝑥
To determine the 𝑥→3 lim+ 𝑓 𝑥 and lim− 𝑓 𝑥 .
evaluate𝑥→3 𝑥→3

Differential Calculus Module 2-Continuity and Limit of a Function Page 34


UNIT 1-FUNCTION AND LIMIT OF A FUNCTION

And to do this, how about assuming values of approaching 3 through values more
than 3; and then, assume values of approaching 3 through values less than 3.
Remember the right-hand and the left-hand limit of a function?

As can be gleaned from the tables below, as → 3+ → +∞ and while →


3 → −∞. Both limits do not exist, the function behaves differently both for
→ +∞ and → −∞ Hence, lim 𝑓 𝑥 does not exist. Evidently, the conditions for
𝑥→3
continuity of the function at x  3 are not satisfied.

x x 1 x x 1
f ( x)  f ( x) 
x 3 x 3
3.1 41 2.9 - 39
3.01 401 2.99 - 399
3.001 4001 2.999 - 3999
3.0001 40001 2.9999 - 39999
3.00001 400001 2.99999 - 399999
3.000001 4000001 2.999999 - 3999999
3.0000001 40000001 2.9999999 - 39999999

x 2  25
c.  x  
x 2  5x  6

The factors of the denominator are  x  6  x  1 . Equating these factors to zero


will give x  6 and x  1 .These values are the points of discontinuity of the given

x
b. g ( x) 
tan x

To determine the point of discontinuity, we look for value of the variable that will
make the denominator equal to zero. We set tan x  0 and then, solve for x. This is a
trigonometric equation whose solutions in the interval 0  x  2 are x  0 ,  and 2 .
Therefore, the points of discontinuity are at = 0

x 2  25
c.  x  
x 2  5x  6
The factors of the denominator are  x  6  x  1 . Equating these factors to zero
will give x  6 and x  1 .These values are the points of discontinuity of the given
function   x  since at these values the function values are not defined.

Differential Calculus Module 2-Continuity and Limit of a Function Page 35


UNIT 1-FUNCTION AND LIMIT OF A FUNCTION

Example 17. Graph of h(x) is shown below.

From the graph, the following properties of the given function can be extracted:

1. ℎ =

lim+ ℎ 𝑥 =
2. 𝑥→1
lim− ℎ 𝑥 = 3
3. 𝑥→1
lim ℎ 𝑥 does not exist
4. 𝑥→1

lim+ ℎ 𝑥 =
5. 𝑥→0

lim− ℎ 𝑥 = 0
6. 𝑥→0

7. lim ℎ 𝑥 = does not exist


𝑥→0

8. lim ℎ 𝑥 =
𝑥→+∞

9. lim ℎ 𝑥 = +∞
𝑥→ ∞

10. Points of discontinuity:


(a) = since ℎ lim ℎ 𝑥 ; and,
𝑥→1

(b) = 0 since ℎ 0 lim ℎ 𝑥


𝑥→0

Differential Calculus Module 2-Continuity and Limit of a Function Page 36


UNIT 1-FUNCTION AND LIMIT OF A FUNCTION

SAQ4

ACTIVITY 1.2 – K

NAME: ____________________________________________________ SCORE: ______________

SECTION: ___________DATE: _______________ PROF: __________________________________

Evaluate the limit of the following functions:

𝑥2
1. lim
𝑥→∞ − 𝑥2

𝑥+3
2. lim
𝑥→+∞ 3𝑥 + 5

𝑥 2 + 7𝑥 + 3
3. lim
𝑥→∞ 𝑥 3 + 3𝑥 +

𝑥4 + 𝑥2 + 5
4. lim
𝑥→+∞ 𝑥 3 − 6𝑥 +

𝑥2 + 4
5. lim
𝑥→+∞ 𝑥+

Differential Calculus Module 2-Continuity and Limit of a Function Page 37


UNIT 1-FUNCTION AND LIMIT OF A FUNCTION

ASAQ4

ACTIVITY 1.2 – K

NAME: ____________________________________________________ SCORE: ______________

SECTION: ___________DATE: _______________ PROF: __________________________________

Evaluate the limit of the following functions:

𝑥2
1. lim : −
𝑥→∞ − 𝑥2

𝑥+3
2. lim 𝐴𝑛𝑠𝑤𝑒𝑟:
𝑥→∞ 3𝑥 + 5 3

𝑥 2 + 7𝑥 + 3
3. lim 𝐴𝑛𝑠𝑤𝑒𝑟: 0
𝑥→∞ 𝑥 3 + 3𝑥 +

𝑥4 + 𝑥2 + 5
4. lim 𝐴𝑛𝑠𝑤𝑒𝑟: ∞
𝑥→+∞ 𝑥 3 − 6𝑥 +

𝑥2 + 4
5. lim 𝐴𝑛𝑠𝑤𝑒𝑟:
𝑥→+∞ 𝑥+

Differential Calculus Module 2-Continuity and Limit of a Function Page 38


UNIT 1-FUNCTION AND LIMIT OF A FUNCTION

SAQ5

ACTIVITY 1.2 – L

NAME: ____________________________________________________ SCORE: ______________

SECTION: ___________DATE: _______________ PROF: __________________________________

Evaluate the following limits of function. The graph of the given function may be useful on limit
evaluation.

𝑥
__________ 𝑎 lim−
𝑥→5 5 − 𝑥2
𝑥
__________ 𝑏 lim+
𝑥→5 5 − 𝑥2
𝑥
__________ 𝑐 lim
𝑥→∞ 5 − 𝑥 2
𝑥
__________ 𝑑 lim Graph of =
𝑥→ ∞ 5 − 𝑥 2 25

𝑥+
__________ 𝑎 lim+
𝑥→0 𝑥

𝑥+
__________ 𝑏 lim
𝑥→0− 𝑥
𝑥+
__________ 𝑐 lim
𝑥→∞ 𝑥

+1
Graph of =

3𝑥 2 + 𝑥 − 5
__________3 𝑎 lim
𝑥→ ∞ 𝑥2 + 4

3𝑥 2 + 𝑥 − 5
__________3 𝑏 lim
𝑥→ ∞ 𝑥2 + 4
3𝑥 2 + 𝑥 − 5
__________3 𝑐 lim
𝑥→ 0− 𝑥2 + 4
3𝑥 2 + 𝑥 − 5
__________3 𝑑 lim 3 +2 5
𝑥→ 0+ 𝑥2 + 4 Graph of = +4

Differential Calculus Module 2-Continuity and Limit of a Function Page 39


UNIT 1-FUNCTION AND LIMIT OF A FUNCTION

ASAQ5
ACTIVITY 1.2 – L

NAME: ____________________________________________________ SCORE: ______________

SECTION: ___________DATE: _______________ PROF: __________________________________

Evaluate the following limits of function. The graph of the given function may be useful on limit
evaluation.
𝑥
__________ 𝑎 lim− 25 𝑥
𝐴𝑛𝑠𝑤𝑒𝑟: −∞
𝑥→5
𝑥
__________ 𝑏 lim+ 25 𝑥
𝐴𝑛𝑠𝑤𝑒𝑟: ∞
𝑥→5
𝑥
__________ 𝑐 lim 𝐴𝑛𝑠𝑤𝑒𝑟: 0
𝑥→∞ 25 𝑥

𝑥
__________ 𝑑 lim 𝐴𝑛𝑠𝑤𝑒𝑟: 0 Graph of =
𝑥→ ∞ 5 − 𝑥2 25

𝑥+
__________ 𝑎 lim+ 𝐴𝑛𝑠𝑤𝑒𝑟: ∞
𝑥→0 𝑥

𝑥+
__________ 𝑏 lim 𝐴𝑛𝑠𝑤𝑒𝑟: − ∞
𝑥→0− 𝑥
𝑥+
__________ 𝑐 lim 𝐴𝑛𝑠𝑤𝑒𝑟: 0
𝑥→∞ 𝑥

+1
Graph of =

3𝑥 2 + 𝑥 − 5
__________3 𝑎 lim 𝐴𝑛𝑠𝑤𝑒𝑟: 3
𝑥→ ∞ 𝑥2 + 4
3𝑥 2 + 𝑥 − 5
__________3 𝑏 lim 𝐴𝑛𝑠𝑤𝑒𝑟: 3
𝑥→ ∞ 𝑥2 + 4
3𝑥 2 + 𝑥 − 5 5
__________3 𝑐 lim 2
𝐴𝑛𝑠𝑤𝑒𝑟: −
𝑥→ 0 − 𝑥 +4 4
5
3𝑥 2 + 𝑥 − 5 : −4
__________3 𝑑 lim Graph of 𝑦 =
3𝑥 +2𝑥 5
𝑥→ 0+ 𝑥2 + 4 𝑥 +4

Differential Calculus Module 2-Continuity and Limit of a Function Page 40


UNIT 1-FUNCTION AND LIMIT OF A FUNCTION

ACTIVITY 1.2 – M

NAME: ____________________________________________________ SCORE: ______________

SECTION: ___________DATE: _______________ PROF: __________________________________

Based on the given graphs of , extract the indicated properties of the function and write answer on
the space provided..
1. 3.

a. 𝑓 − = ___________________
b. − = ____________________
b. 𝑓 = _____________________
𝑏 lim − 𝑓 𝑥 = ________________
𝑥→ 1 𝑐 lim 𝑓 𝑥 = ________________
𝑥→ 2+
𝑐 lim + 𝑓 𝑥 = ________________ 𝑑 lim 𝑓 𝑥 = _______________
𝑥→ 1 𝑥→ 2−

2. 𝑒 lim 𝑓 𝑥 = ___________________
𝑥→2
𝑓 lim+ 𝑓 𝑥 = __________________
𝑥→1
𝑔 lim− 𝑓 𝑥 = __________________
𝑥→1

ℎ lim 𝑓 𝑥 = ___________________
𝑥→1

𝑖 lim 𝑓 𝑥 = _________________
𝑥→+∞

𝑗 lim 𝑓 𝑥 = _________________
𝑥→ ∞
a. 0 = __________________
b. lim 𝑓 𝑥 = ____________
𝑥→0+
c. lim 𝑓 𝑥 = ____________
𝑥→0−
d. lim 𝑓 𝑥 = ______________
𝑥→0

e. lim 𝑓 𝑥 = ____________
𝑥→+∞

lim 𝑓 𝑥 = ____________
f 𝑥→ ∞

Differential Calculus Module 2 – Continuity and Limit of a Function Page 21


MODULE 3
INCREMENT AND DERIVATIVE

Specific Objectives:

At the end of the module, students must be able to:


1. Understand concept of increment and derivative

2. Apply the increment method to find derivative of a given


function.
UNIT 2 – INCREMENT AND DERIVATIVE OF A FUNCTION

INCREMENT
An increment is a small, unspecified, nonzero change in the value of a quantity.
The symbol most commonly used is the uppercase Greek letter delta (  ).
Consider the case of the graph of a function y = f (x) in Cartesian (rectangular)
coordinates, as shown in the figure. The slope of this curve at a specific point P is
y
defined as the limit of m = , as x (read “delta x”) approaches zero, provided the
x
y
function is continuous (the curve is not “broken”). The value depends on defining
x
two points in the vicinity of P. In the illustration, one of the points is P itself, defined as
 
xP , yP  and the other is Q xq , yq , which is near P. The increments here are y  yq  y p
and x  xq  x p . As point Q approaches point P, both of these increments approach
y
zero, and the ratio of increments approaches the slope of the curve at point P.
x
When the increment is positive, it means “increase in the value of the quantity” while a
negative increment signifies a “decrease”.

The term increment is occasionally used in physics and engineering to represent


a small change in a parameter such as temperature T (T ) , electric current I (I ) or
time t (t ) .

DERIVATIVE
We will extend our discussion of limits and examine the idea of the derivative, the
basis of differential calculus. We will assume a particular function of x, such that
y  f x   x 2

Differential Calculus Module 3 – Increment and Derivative of a Function 42


UNIT 2 – INCREMENT AND DERIVATIVE OF A FUNCTION

If x is assigned the value 5, the corresponding value of y will be 5 or 25. Now, if
2

we increase the value of x by 3, making it 8, we have increment x  3 . This results in


an increase in the value of y, and we call this increase an increment or y . From this we
write
y  y  x  x  = 5  3 = 64.
2 2

y  x  x   y
2

Thus,
y  5  x   25
2

y
We are interested in the ratio because the limit of this ratio as x
x
approaches zero is the derivative of function with respect to
As we recall from the discussion of limits, as x is made smaller, y gets
y
smaller also. In our example, the ratio approaches 10 as shown on the table
x
below. Let x = 5, correspondingly, y = 25, then assume values of x that tend to
y
approach zero. Take note that as x  0 ,  10 .
x

x 1 0.1 0.01 0.001 0.0001 1x10-5


y  x  x   x 2 11 1.01 0.1001 0.010001 0.001 1x10-4
2
Variable
y 11 10.1 10.01 10.001 10.000 10.00001
x 1

y
The symbol gives the average rate of change of y with respect to x , that is,
x
with x changing from x to x  x , and with y correspondingly changing from y to y  y .
In effect, the value of the function f (x) becomes y  f ( x  x) . Furthermore, if for a fixed
y
value of x , the quotient approaches a limit as the increment x approaches zero,
x
this limit is called the derivative of y with respect to x for the given value of x . This is
f x , y ' , f ' x , Dx y, Dx f x  .
dy d
denoted by symbol or
dx dx

Thus, by definition, 𝑑𝑦 ∆𝑦 𝑓 𝑥 + ∆𝑥 − 𝑓 𝑥
= lim = lim
𝑑𝑥 ∆𝑥→0 ∆𝑥 ∆𝑥→0 ∆𝑥

when y  f  x  . If the derivative of


dy
Differentiation is the process of finding
dx
f  x  exists, then, f  x  is said to be a differentiable function of

Differential Calculus Module 3 – Increment and Derivative of a Function 43


UNIT 2 – INCREMENT AND DERIVATIVE OF A FUNCTION

Note: If y  f  x  , the instantaneous rate of change of y per unit change in x at x1


is , or, equivalently, the derivative of y with respect to x at x1 , if it exists.

THE FOUR-STEP RULE OR THE INCREMENT METHOD OF DIFFERENTIATION


This is the long process of finding the derivative of a given function using the
increment of a variable and it may be formulated as follows:

1. Replace +∆ for and +∆ for

2. To get y , subtract the original function of ,that is f  x  , from the new function of
x  x  , which is f x  x  . Thus, y  f x  x   f x .
y
3. Divide both sides of the resulting equation in Step 2 by x to define .
x

4 . Take the limit as x approaches zero of all the terms in the equation from Step 3.The
dy
resulting equation is the derivative of f(x) with respect to x or simply .
dx

Example 1. Using the 4-Step Rule or the Increment Method, find the derivative of with
dy
respect to or .
dx

a. y  x 2  2 x  3 ---------------------------------------------------(1)
Step 1: y  y  x  x   2x  x   3 -----------------------(2)
2

Step 2: Subtracting (2) to (1) will give y .



y  x  x   2x  x   3  x 2  2 x  3
2

 
y  x 2  2 xx  x   2 x  2x  3  x 2  2 x  3
2

y  2 xx  x   2x


2

y  x2 x  x  2
y
Step 3: Divide the resulting equation in Step 2 by x to define .
x
y
 2 x  x  2
x
Step 4. Take the limit of all the terms in the resulting equation in Step 3as approaches
zero. ∆𝑦
lim = 2𝑥 + 2 = 2 𝑥 + 1
∆𝑥→0 ∆𝑥

Differential Calculus Module 3 – Increment and Derivative of a Function 44


UNIT 2 – INCREMENT AND DERIVATIVE OF A FUNCTION

1
b. y  , when x = 2.
x  12
1
y  y 
x  x  12
1 1
y  
x  x  1 x  12
2

Recall: Square of a Trinomial of a Trinomial

y 
 x  1   x  x  1
2 2

x  x  12 x  12

y 
x  
 2 x  1  x 2  x   1  2 xx  2 x  2x
2 2

 x  x  1 x  1
2 2

 x   2 xx  2x


2
y 
x  x  12 x  12
x x  2 x  2
y 
x  x  12 x  12
y  x  2 x  2

x x  x  12 x  12
y  x  2 x  2

x x  x  12 x  12

Therefore,
∆𝑦 −∆𝑥 − 2𝑥 + 2
lim = lim
∆𝑥→0 ∆𝑥 ∆𝑥→0 𝑥 + ∆𝑥 − 1 2 𝑥 − 1 2

dy 0  2x  2  2 x  2  2x  1 2
   
dx x  0  1 x  1
2 2
x  14
x  1 x  13
4

dy 2 2
When = 2:   3  2
dx 2  1 1
3

Differential Calculus Module 3 – Increment and Derivative of a Function 45


UNIT 2 – INCREMENT AND DERIVATIVE OF A FUNCTION

Example 2. At what point on the curve y  x is the derivative of y with respect to x


equal to 4.

dy
Solution: We use the Increment Method to define .
dx

y  y  x  x
y  x  x  x
√𝑥 + ∆𝑥 + √𝑥
∆𝑦 = √𝑥 + ∆𝑥 − √𝑥 ∙
√𝑥 + ∆𝑥 + √𝑥

√𝑥 + ∆𝑥 − 𝑥 √𝑥 + ∆𝑥 + 𝑥
∆𝑦 =
√𝑥 + ∆𝑥 + 𝑥

   
2 2 Recall: Product of Sum and Difference of Two Squares
x  x  x
y  Terms
x  x  x
x  x  x x
y  
x  x  x x  x  x
∆ 1
=
∆ √ +∆ +√
∆ 1
lim = lim
∆ →0 ∆ ∆ →0 √ + ∆ + √

But by definition,
∆ 1 1
= lim = =
∆ →0 ∆ √ + +√ 2√
To determine the unknown point on the curve, we use the given condition that

1 1
= 4.Thus, 
2 x 4

Cross-multiply and simplify. 4 = 2√ 2=√

Square both sides of the above equation to solve for . =4

Hence, the unknown point on the curve y  x where the derivative of with respect to

is 4,2  .
1
equals
4

Differential Calculus Module 3 – Increment and Derivative of a Function 46


UNIT 2 – INCREMENT AND DERIVATIVE OF A FUNCTION

SAQ6
ACTIVITY 2.3 – A

NAME: ____________________________________________________ SCORE: ______________

SECTION: ___________DATE: _______________ PROF: __________________________________

Find using the Increment Method of Differentiation.

1. y  3x  2 x  5
2

2. y  x  4 x
3

5
3. y 
2 x

Differential Calculus Module 3 – Increment and Derivative of a Function 47


UNIT 2 – INCREMENT AND DERIVATIVE OF A FUNCTION

1
4. y 
7 x2

2x  3
5. y 
x 1

6. y  4  x

Differential Calculus Module 3 – Increment and Derivative of a Function 48


UNIT 2 – INCREMENT AND DERIVATIVE OF A FUNCTION

ASAQ6
ACTIVITY 2.3 – A

NAME: ____________________________________________________ SCORE: ______________

SECTION: ___________DATE: _______________ PROF: __________________________________

Find using the Increment Method of Differentiation.

1. y  3x  2 x  5
2
𝐴𝑛𝑠𝑤𝑒𝑟: 2 3𝑥 − 1

2. y  x  4 x 𝐴𝑛𝑠𝑤𝑒𝑟: 3𝑥 2 − 4
3

5 5
3. y  𝐴𝑛𝑠𝑤𝑒𝑟: 2
2 x 2−𝑥

Differential Calculus Module 3 – Increment and Derivative of a Function 49


UNIT 2 – INCREMENT AND DERIVATIVE OF A FUNCTION

1 2
4. y  𝐴𝑛𝑠𝑤𝑒𝑟: −
7 x2 7𝑥 3

2x  3 5
5. y  𝐴𝑛𝑠𝑤𝑒𝑟:
𝑥+1 2
x 1

√4 − 𝑥
6. y  4  x 𝐴𝑛𝑠𝑤𝑒𝑟: −
2 4−𝑥

Differential Calculus Module 3 – Increment and Derivative of a Function 50


UNIT 2 – INCREMENT AND DERIVATIVE OF A FUNCTION

ACTIVITY 2.3 – B

NAME: ____________________________________________________ SCORE: ______________

SECTION: ___________DATE: _______________ PROF: __________________________________

In each of the following, find at the indicated value of using the Increment Method.

1. = +1 ; =3

4
2. y  ; =5
x 1

1
3. y  ; 𝑥=2
4 x3

Differential Calculus Module 3 – Increment and Derivative of a Function 51


MODULE 4
RATE OF CHANGE

Specific Objectives:

At the end of the module, students must be able to:


1. Understand concept of rate of change.

2. Solve rate of change problems using derivative of a


function.
UNIT 2 – INCREMENT AND DERIVATIVE OF A FUNCTION

RATE OF CHANGE
y
Recall that the symbol gives the average rate of change of y with respect to
x
x . That is with x changing from x to x  x , y correspondingly changes from to
. If a quantity say changes with quantity , the rate of change of with respect
to represented by at a particular instant, say when is called the
instantaneous rate of change of with respect to

1
Example 1. For the function y  , at what value of is the rate of change of with
x
respect to equal to
dy
Solution: The instantaneous rate of change of y with respect to x is represented by
dx
at the unknown value of
1
y  y 
x  x
1 1
y  
x  x x
x  x  x 
y 
x x  x 
 x
y 
xx  x 
y 1

x xx  x 
𝑑𝑦 1 1
lim
𝑑𝑥 𝑥→0 𝑥 𝑥 𝑥 𝑥2
Using the given condition that the rate of change of y with respect to x equals  4 will
result to the unknown value of

dy
 4
dx
1
 4
x2
1
x2 
4

1
x
2

Differential Calculus Module 4 – Rate of Change Page 52


UNIT 2 – INCREMENT AND DERIVATIVE OF A FUNCTION

Example 2. Find the rate at which the volume of the cube changes with respect to its
edge when the edge measures 2 cm.
Here, we like to find the instantaneous rate of change of the volume with respect
to the edge of cube when the edge measures 2 cm, that is, when .

Recall the formula to find volume V of a cube having edge e , that is, V  e3 .Using
the Increment Method of differentiation,

V  V  e  e 
3

V  e  e   e3
3

V  e3  3e 2 e  3ee   e   e3
2 3

V  3e 2 e  3ee 2 e 
3


V  e 3e 2  3ee  e 
2

V
 3e 2  3ee  e 
2

e
𝑉
lim dV
lim 3e𝑒22 𝑒 𝑒 𝑒 2
𝑒→0 𝑒 𝑒→0
de

cm3
 32  12
dV
When e  2 cm,
2

de cm

Example 3. Boyle’s Law for the expansion of a gas is PV  4 , where P is the number of
pounds per square unit of pressure, V is the number of cubic units in the
volume of the gas, and k is a constant. Find the instantaneous rate of
change of the volume with respect to the pressure when V  8 .

Solution: To find the instantaneous rate of change of the volume with respect to the
pressure , first, we express the volume as a function of pressure. Thus, . Then,
dV
using the Increment Method, we derive expression for .
dP

4
V  V 
P  P
4 4
V  
P  P P
4 P  4P  P 
V 
PP  P 

Differential Calculus Module 4 – Rate of Change Page 53


UNIT 2 – INCREMENT AND DERIVATIVE OF A FUNCTION

 4P
V 
PP  P 
V 4

P PP  P 
𝑑𝑉 𝑉
lim lim
𝑑𝑃 𝑃→0 𝑃 𝑃→0 𝑃 𝑃 𝑃 𝑃2

When . Substitution on the above equation will yield:


2

𝑑𝑉
2 ∙ 16
𝑑𝑃 1 1

Differential Calculus Module 4 – Rate of Change Page 54


UNIT 2 – INCREMENT AND DERIVATIVE OF A FUNCTION

SAQ7
ACTIVITY 2.3 – C

NAME: ____________________________________________________ SCORE: ______________

SECTION: ___________DATE: _______________ PROF: __________________________________

Use the Increment Method of differentiation to solve for the unknown.

1
1. For the function y  , at what values of will the rate of change of with respect to be
2 x
equal to ?

2. Find the rate of change of the area of a circle with respect to its diameter when the radius is 4 cm.
2 2
Area of a circle .

3. Find the rate of change of the volume of a right circular cone of constant altitude of 15 cm with
2
respect to its base radius when the radius measures 2 cm. Volume of a right circular cylinder .

Differential Calculus Module 4 – Rate of Change Page 55


UNIT 2 – INCREMENT AND DERIVATIVE OF A FUNCTION

4. Find the rate of change of the hypotenuse of a right triangle having a constant base of 2cm with
respect to its altitude when the altitude measures 21 cm. Hint: Use Pythagorean Theorem.

5. Find the rate of change of the area of an equilateral triangle when the side measures 16 cm.

√ 2
Area of an equilateral triangle .

Differential Calculus Module 4 – Rate of Change Page 56


UNIT 2 – INCREMENT AND DERIVATIVE OF A FUNCTION

ASAQ7
ACTIVITY 2.3 – C

NAME: ____________________________________________________ SCORE: ______________

SECTION: ___________DATE: _______________ PROF: __________________________________

Use the Increment Method of differentiation to solve for the unknown.

1
1. For the function y  , at what values of will the rate of change of with respect to be
2 x
equal to ? 6

2. Find the rate of change of the area of a circle with respect to its diameter when the radius is 4 cm.
2 2
Area of a circle .

3. Find the rate of change of the volume of a right circular cone of constant altitude of 15 cm with
2
respect to its base radius when the radius measures 2 cm. Volume of a right circular cylinder .
6

Differential Calculus Module 4 – Rate of Change Page 57


UNIT 2 – INCREMENT AND DERIVATIVE OF A FUNCTION

4. Find the rate of change of the hypotenuse of a right triangle having a constant base of 2cm with
respect to its altitude when the altitude measures 21 cm. Hint: Use Pythagorean Theorem.
√2

5. Find the rate of change of the area of an equilateral triangle when the side measures 16 cm.

√ 2
Area of an equilateral triangle . √

Differential Calculus Module 4 – Rate of Change Page 58


MODULE 5
DIFFERENTIATION FORMULAS
FOR ALGEBRAIC FUNCTION

Specific Objectives:

At the end of the module, students must be able to:


1. Know the differentiation formulas for algebraic function.

2. How to use correctly the differentiation formulas for


algebraic function.
UNIT 3 – DERIVATIVE OF ALGEBRAIC FUNCTION

ALGEBRAIC FUNCTION

An algebraic function is one formed by a finite number of algebraic operations on


constants and/or variables. These algebraic operations include addition, subtraction,
multiplication, division, raising to powers, and extracting roots. Polynomial and rational
functions are particular kinds of algebraic functions.
Polynomial function is defined by f x   an x n  an1 x n1  an2 x n2  ...  a1 x  a0

Where a0 , a1 ,..., an are real numbers a n  0 and  is a non-negative integer. Function f is
a polynomial function of degree n .

Rational function is a function that can be expressed as a quotient of two


polynomial functions.
x 1 x 2  25
Example 1. f x   2 Example 2. hx  
x 4 x3

DIFFERENTIATION FORMULAS OF ALGEBRAIC FUNCTIONS

The following differentiation formulas were derived using the Increment Method
of differentiation. On the list below, let and be functions of ; while and are
constants.

1. Derivative of a Constant 6. Derivative of Square Root of a Function


d
c   0 d
u
1 d
u 
dx dx 2 u dx
2. Derivative of x with respect to x 7. Derivative of a Product of Two Factors
d
x   1 d
uv   u d v   v d u 
dx dx dx dx
3. Derivative of a Power of x 8. Derivative of a Product of Three Factors
d
dx
cx n   cnx n1
d
dx
uvw  uv d w  uw d v  vw d u
dx dx dx
4. Derivative of a Sum/Difference of Terms 9. Derivative of a Quotient
d
u  v   d u   d v  d
vu   u v 
d
dx dx dx d u
   dx dx
dx  v  v 2

5. Derivative of a General Power 10. Derivative of a Constant Over a Function


d
cu n   cnu n1 u 
d d  c  c d
  u 
dx dx dx  u  u 2 dx

Let me illustrate how to use the listed differentiation formulas. Most people
find the differentiation process hard. Students failed to arrive at the correct derivative
because of inadequate knowledge of trigonometry, geometry and algebra. But do not be
threatened, illustrative examples have steps presented in detailed way.

Differential Calculus Module 5 – Differentiation Formulas for Algebraic Function Page 59


UNIT 3 – DERIVATIVE OF ALGEBRAIC FUNCTION

or f ' x  of the given algebraic functions.


dy
Example 1. Find the derivative
dx
a. y  x3  4 x 2  6 x  8

The given function is a sum of terms. To differentiate, we use Formula 4:


d
u  v   d u   d v  , followed with the use of Formula 3: d cxn   cnxn1 as well as
dx dx dx dx
Formula 2:
d
x   1 and Formula 1: c   0 .
d
dx dx

( )

( ) ( )

(Recall: )


b. y  2 x 2  3  2

There are two possible ways of finding the derivative of the given function which
is a general power of .

Method 1. First, we transform the given function to a sum of terms by expanding the
right side of the given equation above using the special product called square of a
binomial: a  b   a 2  2ab  b 2 . Hence,
2

  2
y  4 x 4  12 x 2  9  4 x 4  12 x 2  9
 4 x 4  12 x 2  9  44x 4 1  122x 2 1  0
dy d d d
dx dx dx dx
dy
dx

 16 x3  24 x  8 x 2 x 2  3 

Method 2. We use directly the Formula 5:


d
dx
 
cu n  cnu n1 u  , with
d
dx
u  2 x 2  3, n  2
dy

 2 2x2  3 
2 1 d
   d

2 x2  3  2 2 x2  3  2 x2  3
d 
dx dx  dx dx 
dy
dx
 
 2 2 x 2  3 22x 2 1  0  
dy
dx
   
 2 2 x 2  3 4 x   8 x 2 x 2  3

Observe that both methods of finding the derivative give to same result.

Differential Calculus Module 5 – Differentiation Formulas for Algebraic Function Page 60


UNIT 3 – DERIVATIVE OF ALGEBRAIC FUNCTION

c. y 
4 x  13
x2

This time, the given function is a quotient having the numerator a special product called
a cube of binomial.

Method 1. First, we expand the special product a  b  a3  3a 2b  3ab2  b3 , the result of


3

which, then divided by x 2 . Doing so results to:

y
4 x   34 x  1  34 x 1  1
3 2 2 3

x2
64 x3  48 x 2  12 x  1
y  64 x  48  12 x 1  x  2
x2

The simplified form is now a sum of terms and we differentiate using Formula 4:
d
u  v   d u   d v .
dx dx dx

 64 x   48 
dy d
dx dx
d
dx
d
dx

12 x 1  
d 2
dx
x  
 641x 0  0  12 1x  2   2x 3
dy
dx
𝑛
dy 12 2 Recall: 𝑛 𝑎 . Hence: 𝑥
 64  2  3 𝑎 𝑥3
dx x x

v u   u v 
d d
d u
Method 2. We use directly the quotient Formula 9:    dx dx , with:
dx  v  v 2

( ) and . We need and for substitution on the formula.

u  4 x  1 𝑣 𝑥
3

du
dx
 34 x  1
2 d

dx

4 x  1  34 x  12 4 x0  0  𝑑𝑣
𝑑𝑥
𝑥

 34 x  1 4  124 x  1


du 2 2

dx
Substitution on the quotient formula gives

y' 
 2

x 2 124 x  1  4 x  1 2 x 
3

x  2 2

Differential Calculus Module 5 – Differentiation Formulas for Algebraic Function Page 61


UNIT 3 – DERIVATIVE OF ALGEBRAIC FUNCTION

Simplify the numerator by bringing-out the common monomial factor ( ) .

2 x4 x  1 6 x  4 x  1 2 x4 x  1 6 x  4 x  1


2 2
y'  
x4 x4

y' 
2

 
2 x4 x  1 2 x  1 2 x 16 x 2  8 x  1 2 x  1
x4 x4

y' 

2 32 x3  16 x 2  2 x  16 x 2  8 x  1
x3
64 x3  12 x  2 64 x 3 12 x 2
y'   3  3  3
x3 x x x
12 2
y '  64  2  3
x x
Note: The derivative was expanded to show that the result of Method 2 is right.
However, it is always best to express the derivative of a function in its factored-
form.

 
d. y  3x 4  2 x 2  4 x  1 x5  4 x  2 
Method 1. Get the product of the factors on the right side of the given equation to bring
the product to a sum of terms, then, use the Formula 4: u  v   u   v  .
d d d
dx dx dx

y  3x 9  2 x 7  4 x 6  13 x5  6 x 4  8 x 3  20 x 2  12 x  2
dy d 9 d d d d d d d d
 3x  2 x 7  4 x 6  13 x 5  6 x 4  8 x 3  20 x 2  12 x  2
dx dx dx dx dx dx dx dx dx dx
 39x8  27 x 6  46x 5  135x 4  64x 3  83x 2  202x  12
dy
dx
dy
 27 x8  14 x 6  24 x 5  65 x 4  24 x 3  24 x 2  40 x  12
dx

Method 2. Use the product Formula 9:


d
uv   u d v   v d u  with:
dx dx dx

u  3x 4  2 x 2  4 x  1
du
 12 x 3  4 x  4
dx
Substitute now the above on Formula 9.

Differential Calculus Module 5 – Differentiation Formulas for Algebraic Function Page 62


UNIT 3 – DERIVATIVE OF ALGEBRAIC FUNCTION

dy
dx
   
 3x 4  2 x 2  4 x  1 5 x 4  4  x 5  4 x  2 12 x 3  4 x  4  
dy
 15 x8  10 x 6  20 x5  5 x 4  12 x 4  8 x 2  16 x  4
dx

 12 x8  48 x 4  24 x 3  4 x 6  16 x 2  8 x  4 x 5  16 x  8 
dy
 27 x8  14 x 6  24 x 5  65 x 4  24 x 3  24 x 2  40 x  12
dx
dy
 27 x8  14 x 6  24 x 5  65 x 4  24 x 3  24 x 2  40 x  12
dx

e. y  x 3  4 x 2  6 x  7

Solution: To differentiate, we use the Formula 6: d u


1 d
u  with:
dx 2 u dx

du
u  x3  4 x 2  6 x  7 ,  3x 2  8 x  6 . Substitution of these on Formula 6 results to
dx

y' 
1d 3
x  4x2  6x  7  
2 x  4x  6x  7
3 2 dx

y' 
1
3x 2  8 x  6  
2 x  4x  6x  7
3 2

3x 2  8 x  6
y' 
2 x3  4 x 2  6 x  7

y' 
3x 2

 8x  6 x3  4 x 2  6 x  7
Rationalize the fraction.

2 x3  4 x 2  6 x  7 
4
f. y 
x  25
2

The differentiation of the given function can be dome in three ways.


v u   u v 
d d
d u
Method 1. We apply the Quotient Formula 9:    dx dx , with:
dx  v  v2
u4 v  x 2  25
du , dv ,
0  2x
dx dx

Substitution yields y ' 


x 2

 25 0   42 x 

 8x
x 2
 25  2
x 2
 25 
2

Differential Calculus Module 5 – Differentiation Formulas for Algebraic Function Page 63


UNIT 3 – DERIVATIVE OF ALGEBRAIC FUNCTION

d  c  c d
Method 2. We use the special Quotient Formula 10:   u  ,with:
dx  u  u 2 dx
du
c  4, u  x 2  25,  2x .
dx

4
y'  
4 d 2
x  25   2 x  
 x  25 dx
2
 2
x  25
2 2
 
 8x
y ' 
x 2
 25 
2

Method 3. We may rewrite the given function to y  4x 2  25 , then, use the general
1

power Formula 5:
d
dx
cu n   cnu n1 u  , with c  4, n  1, n  1  2, u  x 2  25,
d
dx
du
dx
 2x

 8x

y '  4 1 x 2  25  2 x  
2

x 2
 25 
2

Observe that all methods of differentiation presented yield same result.


2x
g. y 
4x  5

The given function can be differentiated in two different ways.


Method 1: Rewrite the given function to y  2 x4 x  5 2 , then, apply Product Formula 7:
1

d
uv   u d v   v d u .
dx dx dx

v  4 x  5 2
1

u  2x
  4 x  5 2 4 x  5
dv 1  1 d
1
du
2 dx 2 dx
dx
  4 x  5 2 4  
dv 1 
3 2
4 x  52
3
dx 2

Therefore, after doing the necessary substitutions, we got

 2 
  4 x  5 2 2
1
y'  2 x 
 4 x  52 
3
 

 4x 2  4x 2
y'    
4 x  5 4 x  5 4 x  5 4 x  5 4x  5
3
2

Differential Calculus Module 5 – Differentiation Formulas for Algebraic Function Page 64


UNIT 3 – DERIVATIVE OF ALGEBRAIC FUNCTION

 4 x  24 x  5  4 x  8 x  10 4 x  10
y'   
4 x  5 4 x  5 4 x  5 4 x  5 4 x  5 4 x  5
22 x  5
y' 
4 x  5 4 x  5

v u   u v 
d d
d u
Method 2. We use the Quotient Formula 9:    dx dx
dx  v  v2

v  4x  5
dv

1
4  2
dx 2 4 x  5 4x  5
 2 
4 x  5 2  2 x  u  2x
 4x  5 
y' 
4x  5 du
2
24 x  5  4 x 4 x  10 dx
y'  
4 x  5 4 x  5 4 x  5 4 x  5
22 x  5
y' 
4 x  5 4 x  5

4
 x3  8 
h. y   3 
 2x 1 

Solution: To differentiate the given function, apply the general power Formula 5:
cu n   cnu n1 u  , with:
d d 3
3
.
dx dx
Using the quotient formula to find


   
du 2 x 3  1 3x 2  x 3  8 6 x 2

 
 51x 2
.
dx 
2x3 1
2
 2x3 1
2
 
dy  x3  8 
 4 3 
3
  51x 2 
 3 

 204 x 2 x3  8 
3

Therefore, using Formula 5 yields: 2


dx  2x  1     3

 2 x  1  2 x 3  1 2 x 3  1
2


dy  204 x 2 x3  8


3

dx 
2 x3  1
5

Differential Calculus Module 5 – Differentiation Formulas for Algebraic Function Page 65


UNIT 3 – DERIVATIVE OF ALGEBRAIC FUNCTION

SAQ8
ACTIVITY 3.5 – A

NAME: ____________________________________________________ SCORE: ______________

SECTION: ___________DATE: _______________ PROF: __________________________________

Find the derivative of the following functions. If ever possible, express final answer in factored
form.

1. y  3x 5  3x 4  7 x 3  5 x 2  x  8 4  9x
4. y 
4  9x

5 3
2. y  3x5  4 x3   x3  2 
x2 5. y   3 
 x 1 

3. ( ) 1
6. y  3x  3
5x

Differential Calculus Module 5 – Differentiation Formulas for Algebraic Function Page 66


UNIT 3 – DERIVATIVE OF ALGEBRAIC FUNCTION

ASAQ8
ACTIVITY 3.5 – A

NAME: ____________________________________________________ SCORE: ______________

SECTION: ___________DATE: _______________ PROF: __________________________________

Find the derivative of the following functions. If ever possible, express final answer in factored
form.

1. y  3x  3x  7 x  5x  x  8
5 4 3 2 4  9x
4. y 
4  9x
( )

5 3
2. y  2 x 5  4 x 3   x3  2 
x2 5. y   3 
 x 1 
( )
( )

3. ( ) 6. y  3x  3 5 x
( ) ( )
3
√ √

Differential Calculus Module 5 – Differentiation Formulas for Algebraic Function Page 67


UNIT 3 – DERIVATIVE OF ALGEBRAIC FUNCTION

ACTIVITY 3.5 – B

NAME: ____________________________________________________ SCORE: ______________

SECTION: ___________DATE: _______________ PROF: __________________________________

Find the derivative of the following functions. If ever possible, express final answer in factored
form.

1. y  3x 2  5 x  1 4. y  x5 1  2 x 7


2. y  3x 4  2 x 2  4 
2
5. y  3 2 x 2  4 x  3

 
3. y  x3  6 x 2  4 x3  6. y 
2  3x  2 x 2
x3

Differential Calculus Module 5 – Differentiation Formulas for Algebraic Function Page 68


UNIT 3 – DERIVATIVE OF ALGEBRAIC FUNCTION

3 x
7. y  10. y 
x 2
4 
3
2x  1

8. y 
x2  1  
11. y  x 3  2 3x 2  4
x

x4 5
9. y   4  
12. y  x 3  2 3x 2  4
5 x

Differential Calculus Module 5 – Differentiation Formulas for Algebraic Function Page 69


MODULE 6
SLOPE OF TANGENT AND
NORMAL LINE

Specific Objectives:

At the end of the module, students must be able to:


1. Know the relation of derivative of a function to slope of
tangent and normal line to the graph of a function.

2. Find equation of the tangent and normal line to a curve


at a given value of the independent variable.
UNIT 3 – DERIVATIVE OF ALGEBRAIC FUNCTION

THE SLOPE OF THE TANGENT AND THE DERIVATIVE


Consider two points ( ), a fixed point and a variable point ( ) on
the graph of function ( ). Line is a secant line and line the tangent line to
the curve at point . Let point Q approach point P along the curve. From the figure, we
QR y
see the slope of the secant line PQ = = . As Q  P , that is, as x  0 , the slope
PR x
of PQ takes the slope of the tangent line at P as its limit. Thus, by definition,
Δ𝑦
Slope of tangent line at P  x, y  =lim
𝑥→0 Δ𝑥 = at point ( )

Note: The slope of the tangent line defines the slope of the curve at the point of
tangency. Thus,
Slope of the curve at point equals 𝑓′(𝑥1 )

Recall that a normal line to the curve of y  f  x  at point of tangency x1 , y1  is


perpendicular to the tangent at the point of tangency. Therefore, in Analytic Geometry,
recall that the slope of the normal line is the negative reciprocal of the slope of the
1
tangent. In symbol form, slope of normal line at x1 , y1  =  at x1 , y1  . To get the
dy
dx
equation of the tangent and normal to the curve at the point of tangency x1 , y1  , we use
the point-slope form of the equation of the line, that is y  y1  mx  x1  .

Equation of tangent line at (𝑥1 𝑦1 ):

Equation of the normal line at (𝑥1 𝑦1 )

Differential Calculus Module 6 – Slope of Tangent and Normal Line Page 70


UNIT 3 – DERIVATIVE OF ALGEBRAIC FUNCTION

Example 2. Find the slope of the given curve at the indicated point.

 
2 1 
a. y  4 x 2  3 ;  ,16 
2 

Solution: Use the power formula


d
dx
 
cu n  cnu n1 u  to find f ' x  
d
dx
dy
dx
.

f ' x  
dy
dx
  
 2 4 x 2  3 8 x   16 x 4 x 2  3
1 
At point  ,16  , slope of the tangent line is given below.
2 
 1   1    1 
2
1
 f '    16  4   3  84   3  81  3  32
2  2    2    4 

Since the slope of the curve at the point of tangency is defined by the slope of the
1 
tangent line at that point, therefore, slope of the curve at  ,16  equals 32.
2 

b. y  2 x 2 4  x ; 0,0 

Solution: Use the product formula of differentiation,


d
uv   u d v   v d u 
dx dx dx

 1
dy
 f ' x   2 x 2   1  4  x 4 x 
dx 2 4  x 

dy  x2
  4x 4  x
dx 4x

dy  x 2  4 x4  x 

dx 4 x

dy  x 2  16 x  4 x 2 16 x  5 x 2
 
dx 4 x 4 x

At point 0,0  , f ' 0 


dy 0
  0 . Therefore, slope of the curve at 0,0  =slope of the
dx 2
tangent at 0,0  =0. This implies that the tangent to the curve at 0,0  is a horizontal line.

Differential Calculus Module 6 – Slope of Tangent and Normal Line Page 71


UNIT 3 – DERIVATIVE OF ALGEBRAIC FUNCTION

Example 3. Find the equation of the tangent line and normal line to the graph of
4
y at 1,2  .
x 1

Solution: Use the differentiation formula


d  c  c d
  2 u  , y'   4 2 1. At point( )
dx  u  u dx x  1
dy 4 4
   1 .Hence, slope of the tangent line at( )equals while slope of
dx 1  12
4
the normal line at the point is 1. Using the point-slope form of the equation of a line,
Equation of tangent line: y  2  1 x  1

y  2  x 1

x  y 3  0

Equation of the normal line at 1,2 : y  2  1 x  1

y  2  x 1

x  y 1  0

Example 4: Find the equation of the tangent line to the curve y  x 2  x  6 at the points
of intersection of the curve with the x-axis.
Solution: First, find the point of intersection of the curve and the x-axis which is
represented by .
Therefore, 0  x 2  x  6  x  2x  3
Solve for x. x  2 and x  3
Thus, points of intersection are  2,0 and 3,0 .

 f ' x   2 x  1
dy
Differentiate.
dx
Slope of tangent at  2,0 : f '  2  2 2  1  4  1  5

Equation of tangent line at  2,0 : y  0  5 x  2

y  5 x  10

5 x  y  10  0

Slope of tangent line at 3,0  : f ' 3  23  1  6  1  5 .

Similarly, equation of tangent at 3,0  : y  0  5 x  3

5 x  y  15  0

Differential Calculus Module 6 – Slope of Tangent and Normal Line Page 72


UNIT 3 – DERIVATIVE OF ALGEBRAIC FUNCTION

Example 5. At what points are the tangent to curve y  x3  4

(a) parallel to line y  12 x  1  0 ,


(b) perpendicular to line 18 y  6 x  7  0 .

Solution: (a). Reducing the given equation of line y  12 x  1  0 to slope-intercept form


y  mx  b gives slope of line equal to 12 which is equal to the slope of the
tangent line at the unknown points. Therefore,

f '  x   3x 2
12  3x 2
4  x2
Substitute -value on equation y  x3  4 of the curve to get corresponding y-value.
When x  2, y  12 while x  2, y  4 . So, the points on the curve where the tangent is
parallel to y  12 x  1  0 are 2,12  and  2,4  .

1
Solution: (b). The slope of line 18 y  6 x  7  0 after reduction to y  mx  b is 
. Hence,
3
slope of the tangent to the curve perpendicular to 18 y  6 x  7  0 is 3 which is equal to
y ' . Therefore,
y' x   3
3x 2  3
x2 1
x  1
Substitution of x  1to the equation of curve y  x3  4 gives y  5 while when x  1, y  3 .
Thus, the points on the curve y  x3  4 where the tangent line is perpendicular to line
18 y  6 x  7  0 are  1,3 and 1,5 .

Differential Calculus Module 6 – Slope of Tangent and Normal Line Page 73


UNIT 3 – DERIVATIVE OF ALGEBRAIC FUNCTION

SAQ9
ACTIVITY 3.6 – C

NAME: ____________________________________________________ SCORE: ______________

SECTION: ___________DATE: _______________ PROF: __________________________________

I. Find the equation of the tangent line to the given curves at the given point of tangency.

1. y  x 3 x  1 at the point 2,8


4

2. y  x 2  25 at the point 13,12 

II. Find the equation of the normal line to the curves at the given point of tangency.

 
1. y  x 2  1 at the point  2,9
2

x2  4
2. y  at the point 0,4 
x 1

Differential Calculus Module 6 – Slope of Tangent and Normal Line Page 74


UNIT 3 – DERIVATIVE OF ALGEBRAIC FUNCTION

ASAQ9
ACTIVITY 3.6 – C

NAME: ____________________________________________________ SCORE: ______________

SECTION: ___________DATE: _______________ PROF: __________________________________

I. Find the equation of the tangent line to the given curves at the given point of tangency.

1. y  x 3 x  1 at the point 2,8


4
:

2. y  x 2  25 at the point 13,12  :

II. Find the equation of the normal line to the curves at the given point of tangency.

 
1. y  x 2  1 at the point  2,9
2
:

x2  4
2. y  at the point 0,4  :
x 1

Differential Calculus Module 6 – Slope of Tangent and Normal Line Page 75


UNIT 3 – DERIVATIVE OF ALGEBRAIC FUNCTION

ACTIVITY 3.6 – D

NAME: ____________________________________________________ SCORE: ______________

SECTION: ___________DATE: _______________ PROF: __________________________________

Solve the following problems.


3 1 2
1. At what value/s of x will the normal to the curve perpendicular to the line
2

3
2. Find equation of the normal to the curve that is parallel to the line

3. Find the points where the tangent to the curve √ is perpendicular to the line .

2x 1
4. At what point/s will the tangent/s to the curve y  be parallel to the line y  4 x  3  0 ?
2x 1

Differential Calculus Module 6 – Slope of Tangent and Normal Line Page 76


MODULE 7
ANGLE OF INTERSECTION OF
CURVES

Specific Objectives:

At the end of the module, students must be able to:

1. Find point of intersection of curves of given functions.


2. Determine the angle of intersection between two
intersecting curves.
UNIT 3 – DERIVATIVE OF ALGEBRAIC FUNCTION

ANGLE OF INTERSECTION OF TWO CURVES


Angle of intersection denoted by Greek letter phi ( is defined as the angle
between the tangents to the curves at their point of intersection. Unless otherwise
specified, the angle between the tangents will be taken as the acute angle between the
lines. It is found by doing the following steps.

(1). Determine the point/s of intersection of the given curves using elimination or
substitution method.
(2). Find the slopes m1 and m2 of the tangent lines to the curves at their point of
dy
intersection by evaluating of each function at their point of intersection.
dx

m2  m1
(3). Then, use the formula tan   .
1  m2 m1

 If tan   0 ,  is the acute angle of intersection.


 If | | is the obtuse angle of
intersection.
 If m2 m1  1 , the tangents are perpendicular lines, hence   90o .
Moreover, If then, and this signifies that the
tangents are coincident lines.

Example 6. Find the angle of intersection of curves (1) y  x 2  2 x  1 and (2) y  2 x  3 .

Solution: Their point of intersection is obtained using


substitution method.

x 2  2x  1  2x  3
x 2  4x  4  0
x  22  0
x2

When x  2 , y  1 . Hence, the point of intersection of the given parabola and line is 2,1 .

Differentiating each given function and evaluating the derivative at x  2 ,

 f1 ' x   m1  2 x  2
dy
dx
At x  2 , f1 ' 2  22  2  4  2  2
This means the tangent to the parabola at the point where has a slope of

Differential Calculus Module 7 – Angle of Intersection of Curves Page 77


UNIT 3 – DERIVATIVE OF ALGEBRAIC FUNCTION

 f 2 ' x   m2  2 . This signifies that the


dy
Likewise, differentiating y  2 x  3 results to
dx
slope of the tangent line to line y  2 x is always equal to 2 at any point on it.

Since m1  m2  2 , then, tangent lines at the point of intersection of the parabola and line
are coincident lines, hence,   0 o.

1
Example 7. Find the acute angle of intersection of curves y  x  and y  1  x 2 .
x
Solution: Find their point of intersection using substitution method.

1
x  1  x2
x

x 2  1  x  x3

x3  x 2  x  1  0

The polynomial above is factorable by grouping.

x 3

 x 2  x  1  0
x 2 x  1  x  1  0
x 1x 2  1  0
Hence, x  1 , and at this value of y  2.
The roots of equation are imaginary numbers, hence, rejected.
dy
Again, slope of tangents is represented by or f '  x  .
dx
Differentiating, f 1 x   m1  1  2 . Furthermore, f 2 '  x   m2  2 x
1
x
At 1,2,  , f1 ' 1  m1  1  1  0 while f 2 ' 1  m2  21  2 . Therefore,
20
tan  2
1  02
(
  63.43o

Differential Calculus Module 7 – Angle of Intersection of Curves Page 78


UNIT 3 – DERIVATIVE OF ALGEBRAIC FUNCTION

SAQ10
ACTIVITY 3.7 – E

NAME: ____________________________________________________ SCORE: ______________

SECTION: ___________DATE: _______________ PROF: __________________________________

Find the acute angle/s of intersection of the given curves.

(a). y  4 x  x 2 and y  8  x 2

(b). and

Differential Calculus Module 7 – Angle of Intersection of Curves Page 79


UNIT 3 – DERIVATIVE OF ALGEBRAIC FUNCTION

ASAQ10
ACTIVITY 3.7 – E

NAME: ____________________________________________________ SCORE: ______________

SECTION: ___________DATE: _______________ PROF: __________________________________

Find the acute angle/s of intersection of the given curves.

(a). y  4 x  x 2 and y  8  x 2

(b). and

Differential Calculus Module 7 – Angle of Intersection of Curves Page 80


MODULE 8
RECTILINEAR MOTION

Specific Objectives:

At the end of the module, students must be able to:

1. Understand concept of velocity and acceleration in relation


to derivative of a function.
2. Solve using derivative velocity and acceleration problems.
UNIT 3 – DERIVATIVE OF ALGEBRAIC FUNCTION

RECTILINEAR MOTION

It is the motion of a particle along a straight line path. Equation of the form
( )called equation of motion completely described the motion of the particle, where
is the time and is the displacement of the particle at any particular time, measured
from a chosen fixed point in its path called reference point. Most of the time, the starting
point of motion (the position of the particle when t  0) is the selected reference point.
The velocity and acceleration of the particle at time t can be computed using the
equations below.
∆𝑠 𝑑𝑠 ∆𝑣 𝑑𝑣
𝑣 lim 𝑎 lim
∆𝑡→0 ∆𝑡 𝑑𝑡 ∆𝑡→0 ∆𝑡 𝑑𝑡

HORIZONTAL RECTILINEAR MOTION

This is motion of the particle is along a horizontal straight line. Since


displacement, velocity and acceleration are all vector quantities, we will use the
following sign convention:
 If s  0 , the particle is positioned at the right of the reference point
 If s  0 , the particle is positioned at the left of the reference point
 If v  0 , the particle is moving in the direction of increasing s (moving to the right
away from the reference point,  )
 If v  0 , it is moving in the direction of decreasing s (moving to the
left, from the reference point,  )
 If v  0 , the particle is at rest at that particular time
 If a  0 , velocity is increasing
 If a  0 , velocity is decreasing which means the particle is decelerating

VERTICAL RECTILINEAR MOTION

Free-fall motion is a good example of a vertical rectilinear motion. The moving


particle, called a freely-falling body, is acted upon only by its weight with the air
resistance considered negligible. Its acceleration is due to gravity.
ft m cm
Hence, a   g  32 2  10 2  980 2 .
s s s
The first two values are rounded-off to the nearest integer for computational
convenience. For and v , the following sign convention applies:
 If s  0 , the particle is positioned above the reference point
 If s  0 , the particle is positioned below the reference point
 
If v  0 , the particle is moving in the direction of increasing s , 
 
If v  0 , it is moving in the direction of decreasing s , 

Differential Calculus Module 8 – Rectilinear Motion Page 81


UNIT 3 – DERIVATIVE OF ALGEBRAIC FUNCTION

In Physics, the position of the freely-falling body at any time t is described by equation
1
𝑠 𝑣0 𝑡 + 𝑎𝑡 2
2

where: s = displacement at a particular time t


vo = initial velocity of the moving particle (its velocity when )
a  g

Example 8. A particle is moving along a horizontal straight line according to equation of


motion s  2t 3  4 , where in meters, is the displacement of the particle at t
seconds. Find the displacement, velocity and acceleration when t  2
seconds.

Solution: Considering the given equation of motion st   2t 3  4 , when t  2,


s2  22  4  16  4  12 .This implies that the particle is positioned 12 meters at the
3

right of the chosen reference point.

Differentiate the given equation of motion with respect to time to get the velocity
equation. Thus,
vt    23t 2  6t 2
ds
dt
When t  2 , v2  622  64  24  . This means the particle is moving to the right
m
sec
when t  2 .
Differentiate the velocity equation with respect to time to get the acceleration equation.
Hence, at    62t  12t
dv
dt
When t  2 , a2   122   24 2 . The positive sign of the computed acceleration means
m
s
the particle is speeding-up when t  2 seconds.

Example 9. At what time will the acceleration of the particle moving according to the law
s  t 3  9t 2  15t be equal zero? Where is the particle at that particular time?
Solution: Equation of motion: st   t 3  9t 2  15t

vt  
ds
Velocity equation:  3t 2  18t  15
dt
at  
dv
Acceleration equation:  6t  18
dt
At the required time, the given condition is a  0.

Differential Calculus Module 8 – Rectilinear Motion Page 82


UNIT 3 – DERIVATIVE OF ALGEBRAIC FUNCTION

0  6t  18
6t  18
18
t  3
6
Position of the particle when t  3 :
s  3  93  153  27  81  45
3 2

s  9
Therefore, when t  3 , the particle is moving at constant speed (zero acceleration) and is
9 units to the left of the chosen reference point (the starting point).

1 5
Example 10.A particle moves such that s  t 4  t 3  2t 2 . Find its position when it is at
4 3
rest.
Solution: The particle when it is at rest means its velocity equals zero.

vt    4t 3  3t 2  4t


ds 1 5
Velocity equation: dt 4 3
vt   t  5t  4t
3 2

Condition at the required position: v  0


t 3  5t 2  4t  0
 
t t 2  5t  4  0
t t  4t  1  0
Therefore, the time when the particle is at rest is when t  0 , t  1 and t  4 .
Finding its position at each time when the particle is at rest:
s 0   0 (At the reference point)
3  20  24 7
s1 
1 5
 2  (The particle is at the right of the reference point)
4 3 12 12
s4  4  4  24   64 
1 4 5 3 320
 32
2

4 3 3
320 288  320 32
 96     (The particle is at the left of the reference point)
3 3 3

Example 11. A stone moves in a vertical line under gravity alone, with negligible air
m
resistance. If it is thrown initially upward at a velocity of 10 , how far and
s
for how long a time the stone will rise?
Solution: This kind of motion is a free-fall motion. Hence, the stone is moving as a
1
freely-falling body. Use the equation of motion s  vo t  at 2 .
2

Differential Calculus Module 8 – Rectilinear Motion Page 83


UNIT 3 – DERIVATIVE OF ALGEBRAIC FUNCTION

Since initial velocity vo  10m / s, then, a   g  10m / s 2 . Thus, the equation of motion
2
is: +1 + 2( 1 )
2
+1 5

Differentiating with respect to time t , the velocity equation is vt   10  10t.
However, at the highest point of the stone’s flight, the stone momentarily stops. Hence,
at the maximum displacement, v  0 .
1 1
1 sec

Using t  1sec, the maximum displacement S max  101  51  10  5  5m.


2

Example 12. From a point 56 feet above the ground, at what velocity will a stone strike
the ground if it hits the ground after one second?

Solution: The starting point, the chosen reference point, is 56’ above the ground.

Reference point

56’

Ground

When the stone hits the ground, the displacement is negative since its position is
below the chosen reference point, the starting point of motion. The acceleration
ft
a  32 2 . Therefore,
s
2
+ 2 ( 32)
2
16

When on the ground, t  1s, s  56' :  56  vo 1  161


2

vo  56  16  40 ft / s


Since calculated vo is negative, it implies that the stone is thrown initially downward.
Thus,
s  40t  16t 2

Differential Calculus Module 8 – Rectilinear Motion Page 84


UNIT 3 – DERIVATIVE OF ALGEBRAIC FUNCTION

To find the velocity when it hits the ground:


 40  162t
ds
v
dt
v  40  32t

When it strikes the ground, t  1s , v  40  321  72 ft / s  . 


Computed velocity of the stone when it hits the ground is negative, indicating that the
direction of motion at that particular time is downward.

Example 13. A stone is dropped without giving any push on it from the top of a building
256 feet high. Find (a) the velocity of the stone 3 seconds after it was
dropped, (b) the time and the velocity of the stone when it reaches the
ground.
Solution: Since the stone is dropped without giving any push on it, then, initial velocity
ft
vo equals zero, with acceleration a   g  32 2 .
s
1
s  vot  at 2
2
s  16t 2

Velocity equation: v  32t


When t  3s , v  323  96 ft / s

When it strikes the ground, s  256 ft ,


 256  16t 2
256
t2   16
16
t 2  16  0
t  4t  4  0
t  4s t  4s (Rejected)

When t  4s , velocity v  324  128 ft / s  . 

Differential Calculus Module 8 – Rectilinear Motion Page 85


UNIT 3 – DERIVATIVE OF ALGEBRAIC FUNCTION

SAQ11
ACTIVITY 3.8 – F

NAME: ____________________________________________________ SCORE: ______________

SECTION: ___________DATE: _______________ PROF: __________________________________

Solve for the unknown/s.


1 3 1 2
1. A particle moves horizontally according to the law s  t  t  12t  1. Where is the particle
3 2
and at what velocity is it moving when t  1 ? When will the particle come to rest and what is its
acceleration at that time?

2. The path taken by a particle is a horizontal line as it moves according to the law
1 7 1
s  t 4  t 3  7t 2  t  1 . At what time will its acceleration be equal to one? What is its velocity
6 6 2
at that time?

3. Two particles leave the same point at the same time and both are moving along a horizontal line
according to the law s  4t 3  2t 2  3 and s  7  28t  10t 2 , respectively. When are they moving at
the same speed? Find their positions, velocity and acceleration at that time?

Differential Calculus Module 8 – Rectilinear Motion Page 86


UNIT 3 – DERIVATIVE OF ALGEBRAIC FUNCTION

ft
4. A ball is thrown vertically downward from a height of 512 feet with a velocity of 64 . How long
s
will it take the ball to reach the ground and at what velocity?

ft
5. A stone is thrown vertically upward with an initial velocity of 80 from the top of a building.
s
How high will it rise? When and at what velocity will it strike the ground if the height of the building
is 576 feet?

Differential Calculus Module 8 – Rectilinear Motion Page 87


UNIT 3 – DERIVATIVE OF ALGEBRAIC FUNCTION

ASAQ11
ACTIVITY 3.8 – F

NAME: ____________________________________________________ SCORE: ______________

SECTION: ___________DATE: _______________ PROF: __________________________________

Solve for the unknown/s.


1 3 1 2
1. A particle moves horizontally according to the law s  t  t  12t  1. At what velocity is the
3 2
particle moving when t  1 ? When will the particle come to rest and what is its acceleration at that
time? (1) 12 4 (4) 7

2. The path taken by a particle is a horizontal line as it moves according to the law
1 7 1
s  t 4  t 3  7t 2  t  1 . At what time will its acceleration be equal to one? What is its velocity
6 6 2
22
at that time? 5 (5)

3. Two particles leave the same point at the same time and both are moving along a horizontal line
according to the law s  4t 3  2t 2  3 and s  7  28t  10t 2 , respectively. When are they moving at
the same speed? Find their positions and acceleration at that time?
5 5 2 11 2 2 2

Differential Calculus Module 8 – Rectilinear Motion Page 88


UNIT 3 – DERIVATIVE OF ALGEBRAIC FUNCTION

ft
4. A ball is thrown vertically downward from a height of 512 feet with a velocity of 64 . How long
s
will it take the ball to reach the ground and at what velocity? 4 192

ft
5. A stone is thrown vertically upward with an initial velocity of 80 from the top of a building.
s
How high will it rise? When and at what velocity will it strike the ground if the height of the building
is 576 feet? 9 2 8

Differential Calculus Module 8 – Rectilinear Motion Page 89


UNIT 3 – DERIVATIVE OF ALGEBRAIC FUNCTION

ACTIVITY 3.8 – G

NAME: ____________________________________________________ SCORE: ______________

SECTION: ___________DATE: _______________ PROF: __________________________________

1. A stone is dropped from rest from a point 125 meters from the ground. Find:
(a). its velocity when it falls on the ground.
(b). the time it takes the stone to reach the point 80 meters above the ground.

2. A diver jumps upward at a velocity of 32 from a diving board that is 48 above the water.
At what time and at what velocity will the diver hit the water?

3. From a point 160 feet above the ground, a body is thrown vertically upward. When it is 32 feet
ft
above the starting point, it is moving downward at 16 . Find:
s
(a). the initial velocity of motion.
(b). the time when it returns back to the starting point.
(c). the time and velocity when the body hits the ground.
(d). the highest point of its flight measured from the ground.

Differential Calculus Module 8 – Rectilinear Motion Page 90


UNIT 3 – DERIVATIVE OF ALGEBRAIC FUNCTION

4. A ball is thrown vertically from a point 30 feet above the ground.


(a). At what velocity must it be thrown for it to strike the ground in 2 seconds? Is the ball initially
thrown vertically upward or downward?
(b). What is its velocity when it strikes the ground?
(c). Where is the ball one second after it was thrown initially? Is it moving upward or already on its
way down? (Hint: Take the time and displacement at the maximum point of its flight by setting
v  0. )

5. A stone thrown vertically upward from the ground reaches a height of 5 m in one second. Find
how high the ball will rise. Assume the stone moves as a freely-falling body.

6. A ball is thrown vertically upward from a point 13 m from the horizontal ground. When and at
what velocity will it strike the ground if its velocity 3 m the point of origin is 2 m⁄ and is
still moving upward?

Differential Calculus Module 8 – Rectilinear Motion Page 91


MODULE 9
HIGHER ORDER DERIVATIVES

Specific Objectives:

At the end of the module, students must be able to:

1. Understand concept of higher order derivative.


2. Use an appropriate differentiation formula to find higher
order derivative of a given function.
UNIT 3 – DERIVATIVE OF ALGEBRAIC FUNCTION

DERIVATIVES OF HIGHER ORDER

If y  f  x  is a differentiable function of x , then, y ' or f ' is sometimes termed the


d
y '  y"  f "
first derivative of y with respect to x . If y’ is a differentiable function, then,
dx
(read as “y double prime”). This is called the second derivative of y with respect to x .

Likewise,
d
 y"  y"'  f '" is the third derivative of y with respect to x , provided y"
dx
d n 1
exists. Moreover, y  y n   f n  is the n th derivative of y with respect to x , where n
dx
is a positive integer greater than 1.
dy d  dy  d 2  y 
Based on Leibniz notation, is the first derivative,   . The n th
dx dx  dx  dx 2

dny n d n f x 
derivative of y with respect to x has the notation or Dx y or .
dxn dx n

Example 14.Given: y  2 x5  3x4  6 x3  x2  1 . Find y 4  .


y '  10 x 4  12 x 3  18 x 2  2 x
y"  40 x 3  36 x 2  36 x  2
y '"  120 x 2  72 x  36
y 4   240 x  72  2410 x  3

Example 15. Given: , Find


d c c d
Use Formula 10:   u .
dx  u  u 2 dx
( ) ( )
( ) ( ) ( )
Use again Formula 10:
( )
( )( )( )
( ) ( )
For the 3rd time, use Formula 10.
( )( ) ( )
( ) ( )

Example 16. Given: ( )( ) .

Use Formula 7: uv   u v   v u 


d d d
dx dx dx
( )( )( ) ( ) ( ) ( )

Differential Calculus Module 9 – Higher Order Derivative Page 92


UNIT 3 – DERIVATIVE OF ALGEBRAIC FUNCTION

Bring-out the common monomial factor. ( ) [ ( ) ( )]


Simplify further. ( ) ( ) ( ) ( )
Differentiate again: [( ) ( ) ( )( )( )( )]
Bring-out the common monomial factor. ( )( )[ ( ) ( )]
( )( )
Differentiate again. [( )( ) ( )( )]
( )[ ( ) ( )]
( )
( )

d2y
Example 17. Find 2
, given y  x 2  4 .
dx
d 1 du
Differentiate using the formula u
dx 2 u dx
dy

1 d 2
dx 2 x 2  4 dx
x 4 
1
 
2 x   x
2 x2  4 x2  4
d 2 y d  dy  d  x 
     
dx 2
dx  dx  dx  x 2  4 

v u   u v 
d d
d u
Using the quotient formula    dx dx
dx  v  v2
  x2
x 2  4 1  x 2 x 
1
 x2  4 
d2y 2 x 4  x2  4
2
 
dx 2  x  4
2
2
x2  4

d2y


x2  4  x2


2
x2  4  x2

4
dx 2  
x2  4 x2  4 
x2  4 x2  4  x 2
4 
3

d2y 4
But y  x 2  4 , therefore, 
dx 2 y 3

Differential Calculus Module 9 – Higher Order Derivative Page 93


UNIT 3 – DERIVATIVE OF ALGEBRAIC FUNCTION

SAQ12
ACTIVITY 3.9 – H

NAME: ____________________________________________________ SCORE: ______________

SECTION: ___________DATE: _______________ PROF: __________________________________

I. Find the second derivative of the given function.

( ) ( )

2. √

3.

Differential Calculus Module 9 – Higher Order Derivative Page 94


UNIT 3 – DERIVATIVE OF ALGEBRAIC FUNCTION

ASAQ12
ACTIVITY 3.9 – H

NAME: ____________________________________________________ SCORE: ______________

SECTION: ___________DATE: _______________ PROF: __________________________________

II. Find the second derivative of the given function.

( ) ( ) ( )

2. √ ( )√

3. ( )

Differential Calculus Module 9 – Higher Order Derivative Page 95


MODULE 10
IMPLICIT DIFFERENTIATION

Specific Objectives:

At the end of the module, students must be able to:

1. Understand concept of implicit differentiation.


2. Perform implicit differentiation to find derivative of a given
function.
UNIT 3 – DERIVATIVE OF ALGEBRAIC FUNCTION

IMPLICIT DIFFERENTIATION

We encounter some equations in x and y that do not explicitly define y as a


function of x. It is not easy manipulating the equation to solve for y in terms of x, even
though such function exists. The technique of finding without solving the given
function for y is termed implicit differentiation. Generally, if the given function takes the
dy
form F x, y   0 , we find the by following the steps listed below.
dx
a. Whenever possible, we solve the given equation of the curve for y and then,
differentiate y with respect to x. This is true only for very simple equations; for
complicated functions, this step is to be avoided.
b. Considering y as a function of x, differentiate each term of the given equation with
respect to x, bearing in mind that y is a function of x, and solve the resulting equation
for y’. This process of finding the derivative is implicit differentiation.

Example 18. Find y’, given y 2  3x3 y  5x 2  4  0


It is not possible to solve the given equation for y as a function of x. Thus, we
differentiate implicitly.

2y
dy  dy
 3 x3  

 y 3x 2   10 x  0
dx  dx 
dy dy
Collect terms having . 2 y  3x 3  9 x 2 y  10 x  0
dx dx
Factor-out .
dy
dx
 
2 y  3x 3  10 x  9 x 2 y

dy x10  9 xy 

dx 2 y  3x 3

Example 19.Find y" , given 2 x 2  y 2  5

Using implicit differentiation:


d
dx
 
2x2 
d 2
dx
 
y  5
d
dx
dy
4x  2 y 0
dx
dy
 2y  4 x
dx

Differential Calculus Module 10 – Implicit Differentiation Page 96


UNIT 3 – DERIVATIVE OF ALGEBRAIC FUNCTION

, provided  y  0 
dy 4 x 2 x
 
dx 2 y y
dy d2y
Using the quotient formula, differentiate implicitly with respect to to get or y" .
dx dx 2
 dx dy   dy 
dy 2 x  y dx  x dx   y  x dx 
Simplify using  . y"  2 2   2 2 
dx y  y   y 
   
  2 x   y2  2x2 
2 y  x  2 
 y 
y' '    
y
2

y y2

 
 2 y 2  2x2  1
y"     y2
 y 
2y  2 x   22 x 2  y 2 
2 2
y' '  
y3 y3

However, from the given, 2 x 2  y 2 can be replaced by Substituting and simplifying, we


 25  10
got: y"   3
y3 y

Example 20. Find slope and equation of the tangent line to curve 2 x 2  3xy  2 y 2  2 at
 3
point   1, .
 2
Implicitly differentiate:
 
d 2x 2
 3 xy  
d  
d 2 y 2 d 2

dx dx dx dx
 dy  dy
4 x  3 x  y   4 y  0
 dx  dx
dy dy
4 x  3x  3 y  4 y 0
dx dx
dy
4 y  3x   3 y  4 x
dx

dy 3 y  4 x

dx 4 y  3x

Differential Calculus Module 10 – Implicit Differentiation Page 97


UNIT 3 – DERIVATIVE OF ALGEBRAIC FUNCTION

 3
Hence, at point   1,  , the slope of the tangent line is
 2

 3
3    4 1  9  4
 y'  
dy 2
 2
 3 63
4    3 1
dx
 2

 9  8 1
dy 1 1 1
 2  2   
dx 3 3 2 3 6

Equation of the tangent line using the point-slope form y  y1  m x  x1  :


 3 1
y      x   1
 2 6

y   x  1
3 1
2 6
2 y  3 x 1

2 6
x 1
2y  3 
3
6y  9  x 1
x  6y  8  0

1
Example 21. At what point of the curve xy  6 is the slope of the tangent line equal to  .
6
dy
Use implicit differentiation: x  y0
dx
dy  y
 = slope of the tangent line
dx x
6

6 dy 1 1 6
But y  and  :   x  2
x dx 6 6 x x
x 2  36
x 2  36  0
x  6x  6  0
x6 x  6
y 1 y  1
1
Thus, at points 6,1 and  6,1 , the slope of the tangent line is equal to  .
6

Differential Calculus Module 10 – Implicit Differentiation Page 98


UNIT 3 – DERIVATIVE OF ALGEBRAIC FUNCTION

Example 22. Find the angle of intersection of circles x 2  y 2  4 and x 2  y 2  2 y  0 .


First, find the point of intersection of the given circles using substitution method. But

x2  y2  2 y  0
(4)  2 y  0
y2
When y  2 , x  0 . Since they intersect at only a point, therefore, the circles are tangent
circles at point 0,2  .Implicitly differentiating x 2  y 2  4 results to:
2 x  2 yy'  0
x
y'    m1
y
Likewise, differentiating x 2  y 2  2 y  0 :
2 x  2 yy'2 y '  0
2 y '  y  1  2 x
 2x x
y'    m2
2 y  1 y  1

00
At point 0,2  , m1  m2  0 . Therefore, tan    0.
1 0
  Arc tan 0  0

Therefore, the tangent lines to the tangent circles at point 0,2  are coincident lines,
thus,   0o .

Differential Calculus Module 10 – Implicit Differentiation Page 99


UNIT 3 – DERIVATIVE OF ALGEBRAIC FUNCTION

SAQ13
ACTIVITY 3.10 – I

NAME: ____________________________________________________ SCORE: ______________

SECTION: ___________DATE: _______________ PROF: __________________________________

dy
I. Using implicit differentiation, find .
dx
1. x y  2
3

2. xy  x  y

3. xy  x  2 x y  3
2 2 2

4. ( )

Differential Calculus Module 10 – Implicit Differentiation Page 100


UNIT 3 – DERIVATIVE OF ALGEBRAIC FUNCTION

II. Using implicit differentiation, find y’’(x)

1.

2.

III. Find equation of the tangent and normal to at point( , ).

IV. At what points are tangents to circle of slope equal to ?

Differential Calculus Module 10 – Implicit Differentiation Page 101


UNIT 3 – DERIVATIVE OF ALGEBRAIC FUNCTION

ASAQ13
ACTIVITY 3.10 – I

NAME: ____________________________________________________ SCORE: ______________

SECTION: ___________DATE: _______________ PROF: __________________________________

dy
V. Using implicit differentiation, find .
dx
𝑦
2. x y  2
3
𝐴𝑛𝑠𝑤𝑒𝑟:
𝑥

𝑦
2. xy  x  y 𝐴𝑛𝑠𝑤𝑒𝑟:
𝑥

𝑥 𝑦 𝑥𝑦
3. xy  x  2 x y  3
2 2 2
𝐴𝑛𝑠𝑤𝑒𝑟:
(𝑥 𝑦)

𝑥 6𝑥𝑦 𝑦
4. ( ) 𝐴𝑛𝑠𝑤𝑒𝑟:
6𝑥 𝑦 𝑥

Differential Calculus Module 10 – Implicit Differentiation Page 102


UNIT 3 – DERIVATIVE OF ALGEBRAIC FUNCTION

VI. Using implicit differentiation, find y’’(x)

3. 𝐴𝑛𝑠𝑤𝑒𝑟:
𝑦

4. 𝐴𝑛𝑠𝑤𝑒𝑟:
𝑦

VII. Find equation of the tangent and normal to at point( , ).


𝐴𝑛𝑠𝑤𝑒𝑟: 𝑥 𝑦 ; 𝑥 𝑦

VIII. At what points are tangents to circle of slope equal to ?


𝐴𝑛𝑠𝑤𝑒𝑟: ( , ); ( , )

Differential Calculus Module 10 – Implicit Differentiation Page 103


UNIT 3 – DERIVATIVE OF ALGEBRAIC FUNCTION

ACTIVITY 3.10 – J

NAME: ____________________________________________________ SCORE: ______________

SECTION: ___________DATE: _______________ PROF: __________________________________

I. Find using implicit differentiation.


1. 4 y 2  3x 2  8

2. 6x2  7 y 2  3

II. Find equation of the tangent and normal to curve 3x  2 xy  y  3 at the point 1,0  .
2 2

III. At what point does the line y  2 x  1cross the tangent line to the curve 2 x  3xy  2 y  0 at
3 2

point  1,2  ?

Differential Calculus Module 10 – Implicit Differentiation Page 104


UNIT 3 – DERIVATIVE OF ALGEBRAIC FUNCTION

IV. Find the point of intersection of the tangents to circle x  y  10 at 3,1 and to parabola
2 2

y 2  2 x at 2,2.

V. Find the angle of intersection


(a). between parabolas and .

(b). circles x  y 8x  2 y  7  0 and x  y 3x  7 y  12  0 . (Hint: Eliminate the quadratic


2 2 2 2

terms to find their point of intersection.)

Differential Calculus Module 10 – Implicit Differentiation Page 105


MODULE 11
CHAIN RULE OF
DIFFERENTIATION

Specific Objectives:

At the end of the module, students must be able to:

1. Understand concept of chain rule of differentiation.


2. Perform chain rule of differentiation to find derivative of a
given function.
UNIT 3 – DERIVATIVE OF ALGEBRAIC FUNCTION

CHAIN RULE OF DIFFERENTIATION

We are used to have y  f  x  in finding the derivative of y with respect to x . If this


is not the given case, the Chain Rule is one of the most important tools in differentiation.

Given Differentiation Formula


(a ). y  f u  and x  g u  dy
dy du

dx dx
du
(b). y  f u  and u  g  x  dy dy du
 
dx du dx
( ) ( ) dy 1

dx dx
dy
dy
Note: In Case (b), in many instances, it is conveniently possible to express in terms
dx
of x alone.
dy d2y
Example 23.Find and 2
, given y  t 3  3 , t  2 x  1
dx dx
dy dy dt
Solution: The given functions fall on Case (b). Hence,  
dx dt dx
t  2x  1 y  t3  3 dy dy dt
 
dx dt dx
dt

1
2 dy
 3t 2
 3t 
dx 2 2 x  1 dt dy 1 3t 2
2

dt

1 dx 2x  1 2x  1
dx 2x  1 But, t  2 x  1 .
Therefore,

dy 3 2 x  1


2

 3 2 x  1.
dx 2x  1

To find , use Formula 6:


d
u
1 d
u .
dx 2 u dx
d2y d   3 2x 1
 2  
1 3
 y '  3  .
dx 2
dx  2 2x 1  2x 1 2x 1


Example 24. If y  w3  1 and x  
5 2
w 1
, find
dy
dx
.

Differential Calculus Module 11 – Chain Rule of Differentiation Page 106


UNIT 3 – DERIVATIVE OF ALGEBRAIC FUNCTION

dy
dy du
The given functions fall under Case (a), so,  .
dx dx
du
2
d  c  c d
Use    2 u  . dx
 1   2 2
dx  u  u dx dw w  1 2
w  1

Use the general power formula 5:


d
dx
 
cu n  cnu n 1 u .
d
dx
dy
dw
 4
 
 5 w3  1 3w 2  15w 2 w3  1
4
 
Therefore,
dy
dy dw 15w2 w3  1
 
4
 
dx dx 2
dw w  12
4 w  1
 
2
dy 2
Divide the fractions.  15w w3  1 
dx 2

Finally,
dy


15w2 w3  1 w  1 
4 2

dx 2

y
Example 25. Find equation of the tangent line to curve x  at point 1,2  .
y 2
2

dy 1
Solution: Slope of the tangent line is given by which is equal to .
dx dx
dy
dy 1
The given function falls under Case (c).  .
dx dx
dy


 
dx y 2  2 1  y2 y  y 2  2  2 y 2  2  y 2
   
2  y2
dy 
y2  2
2
 y2  2
2
y2  2 
2
y2  2 
2
   
dy

1

1
 
y2  2
 mTL
 
2

Therefore,
dx dx

2  y2  y2  2 
dy y2  2
2
 

Differential Calculus Module 11 – Chain Rule of Differentiation Page 107


UNIT 3 – DERIVATIVE OF ALGEBRAIC FUNCTION

At point 1,2  , mTL 


2  2
2 2


2
2
4
 
2
22  2 6 6 3
Thus, equation of tangent line is:

y2
2
x  1
3
3 y  6  2 x  2
2x  3y  8  0

Differential Calculus Module 11 – Chain Rule of Differentiation Page 108


UNIT 3 – DERIVATIVE OF ALGEBRAIC FUNCTION

SAQ14
ACTIVITY 3.11 – K

NAME: ____________________________________________________ SCORE: ______________

SECTION: ___________DATE: _______________ PROF: __________________________________

Using the appropriate method of differentiation, find

4 y  3y2
1. x 
2  y2


2. y  w w  4 ; x 
2

4 w3
w3

1
3. y  t  1 ; t 
2

x 1

2 1
4. y  ;m  2
3m  1 2x  3

v2  4 1
5. y  ; x
v2  4 v 1

Differential Calculus Module 11 – Chain Rule of Differentiation Page 109


UNIT 3 – DERIVATIVE OF ALGEBRAIC FUNCTION

ASAQ14
ACTIVITY 3.11 – K

NAME: ____________________________________________________ SCORE: ______________

SECTION: ___________DATE: _______________ PROF: __________________________________

Using the appropriate method of differentiation, find

4 y  3y2 −4(𝑦 2 + 3𝑦 − 2)
1. x  𝐴𝑛𝑠𝑤𝑒𝑟:
2  y2 (2 + 𝑦 2 )2


2. y  w w  4 ; x 
2

4 w3
w3
𝐴𝑛𝑠𝑤𝑒𝑟: (𝑤 2 + 4)3 (9𝑤 2 + 4)

1 −1
3. y  t  1 ; t 
2
𝐴𝑛𝑠𝑤𝑒𝑟:
x 1 (𝑥 − 1)2 1 + (𝑥 − 1)2

2 1 6
4. y  ;m  2 𝐴𝑛𝑠𝑤𝑒𝑟:
3m  1 2x  3 𝑥3

v2  4 1 −16𝑣(𝑣 + 1)2
5. y  ; x 𝐴𝑛𝑠𝑤𝑒𝑟:
v2  4 v 1 (𝑣 2 + 4)2

Differential Calculus Module 11 – Chain Rule of Differentiation Page 110


UNIT 3 – DERIVATIVE OF ALGEBRAIC FUNCTION

ACTIVITY 3.11 – L

NAME: ____________________________________________________ SCORE: ______________

SECTION: ___________DATE: _______________ PROF: __________________________________

I. Using the appropriate method of differentiation, find


1
1. y ; t  x2  4
t  12

2
2. + 2; ( )

2
3. −

4. +1 ( )
√ −1

II. A curve is given parametrically by the equations x  1  t  , y  1  t  .Find the equation of


2 2

the tangent to the curve at the point where x  y . Hint: Use the chain-rule of differentiation to
find the derivative. To find the point of intersection, use substitution method. Furthermore,
find the value of t using the given condition x  y , then, compute the corresponding value of x
and y .

Differential Calculus Module 11 – Chain Rule of Differentiation Page 111


MODULE 12
MAXIMUM AND MINIMUM
VALUE OF A FUNCTION

Specific Objectives:

At the end of the module, students must be able to:

1. Define and determine critical value and inflection point.


2. Learn concavity of a curve at the maximum and minimum
value of x.
3. Apply the first derivative and second derivative test to
determine whether a critical point is maximum or minimum.
4. Draw the graph of a given function after having gathered
some of its properties using concept of derivative.
UNIT 3 – DERIVATIVE OF ALGEBRAIC FUNCTION

MAXIMUM AND MINIMUM FUNCTION VALUE


Sketching the graph of function is better facilitated using the geometrical
interpretation of the derivative of a function as the slope of the tangent line at a point to
the graph of the function. The derivative serves as a great tool in determining at what
point on the curve is the tangent line horizontal; that is, where the slope of the tangent
line or y’ equals zero.

Definition: The function f is said to have a relative maximum value at if there exists an
open interval that contains c , on which f is defined such that f c   f x  for all
x in this interval. Figures A and B below each exhibit a sketch of a part of the
graph of the function that has a relative maximum value at c .

Figure A Figure B
Relative Maximum: Relative Maximum: is infinite (a cusp)

Definition: The function f is said to have a relative minimum value at c if there exists an
open interval that contains c , on which f is defined such that f c   f x  for all
x in this interval. Figures C and D shown below each exhibit a sketch of a
part of the graph of the function that has a relative minimum value at c .

Figure C Figure D
Relative Maximum: Relative Maximum: is infinite (a cusp)

Definition: A critical value of a function f is a value of x where f '  0 . A critical point of


a function f is the point  x, f  x  on the graph that corresponds to the critical
value x .

Differential Calculus Module 12 – Maximum and Minimum Value of Function Page 112
UNIT 3 – DERIVATIVE OF ALGEBRAIC FUNCTION

In Figures A and C on the preceding page, f ' c   0 since tangent at is horizontal,


while in Figures B and D, f ' c  is infinite since tangent line is vertical. In the succeeding
discussions, relative maximum (minimum) value will simply be termed maximum
(minimum) value.

CONCAVITY TEST

(a). First-Derivative Test


This test is used to classify whether a critical point is a maximum, minimum point
or neither maximum nor minimum.
1. Get f '  x  and equate it to zero to find the critical values.
2. Locate the critical values on the rectangular coordinate system to establish a
number of intervals.
3. Determine the sign of f '  x  on each interval.
4. If x  xo is a critical value, assume increasing value of x through x  xo . If f '  x 
 Changes from + to  , then, x  xo is a relative maximum value and
concavity of the curve is downward.
 Changes from  to +, then, x  xo is a relative minimum value and
concavity of the curve is upward.
 Does not change in sign, then, x  xo is neither a relative maximum nor a
minimum value.

(b). Second-Derivative Test


1. Set f '  x   0 and solve for the critical values.
2. Critical value x  x0 is
 A maximum value if f " x0   0 .
 A minimum value if f " x0   0 .
Note: If f "x0   0 or becomes infinite, the second-derivative test fails. In this case, the
first-derivative test must be used.

Differential Calculus Module 12 – Maximum and Minimum Value of Function Page 113
UNIT 3 – DERIVATIVE OF ALGEBRAIC FUNCTION

Definition: A point ( )on a curve is a point of inflection if f " xo   0 at this point and if
the curve changes its direction of concavity from upward to downward, or vice
versa.
Example 26.Determine and classify using the First-Derivative Test the critical point of
2
3
x
the curve represented by equation y  .
x2
 
1
 23
x  2 2 x 3 x

Solution: Find y'  x  and equate to zero. y ' x   3 
x  22
2 x  2
2

1
 x3
y ' x  
3
3x
x  22
2 x  4  3x
1

y ' ( x)  3x 3
x  22
4 x
y ' x   1
3x 3 x  2
2

4 x
At the critical points: y' x   0 = 1
3x 3 x  2
2

0 4 x
x4
y  0.42

Using the First-Derivative Test to classify point 4,0.42  :



When x  4 : x  3.9 , y ' 3.9   


When x  4 : x  4.1 , ' y ' 4.1   

Since y'  x  changes from  to  as x increases through x  4 , therefore, 4,0.42  is
a maximum point. Take note also that the line x  2 is a vertical asymptote of the given
curve. This is because when , .

Example 27.Examine the given curve for relative maxima and minima, concavity and
point of inflection and roughly sketch the curve.
(a). y  x  4 x  2 x  12x  8
4 3 2

Differential Calculus Module 12 – Maximum and Minimum Value of Function Page 114
UNIT 3 – DERIVATIVE OF ALGEBRAIC FUNCTION

Solution: Find y'  x  and equate it to zero.

y' x   4 x3  12x2  4 x  12 = 0 which is factorable by grouping.


 
y ' ( x)  4 x 3  3 x 2  x  3  0
y ' x)  x 3  3x 2  x  3  0
y' ( x)  x 2 x  3  x  3  0

y ' ( x)   x  3 x 2  1  0 
x  3x  1x  y' ( x)  1  0
x3 x  1 x 1
y  17 y  17 y  1

To classify the critical points 3,17 ,  1,17 , and 1,1 , we may perform either of the
following tests:
(a) First-Derivative Test:

3,17  : x  3 : x  2.9 , y ' 2.9       


x  3 : x  3.1, y ' 3.1)        

Since y'  x  changes from  to  as x increases through x  3 , therefore, 3,17  is a


minimum point.

 1,17  : x  1 : x  1.1 , y '  1.1       


x  1 : x  0.9 , y '  0.9       

Since y'  x  changes from  to  as x increases through x  1 , thus,  1,17  is a


minimum point.

1,1 : x  1: x  0.9 , y ' 0.9       


x  1: x  1.1 , y ' 1.1       

With ( ) changing from + to  as x increases through , hence, ( )is a


maximum point of the curve.

(b) Second-Derivative Test


y" x   12x 2  24x  4

y" x   4 3x 2  6 x  1 
( ) ( ) ( ) ( )

( ) ( ) ( ) ( )
( ) ( ) ( ) ( ) )

Differential Calculus Module 12 – Maximum and Minimum Value of Function Page 115
UNIT 3 – DERIVATIVE OF ALGEBRAIC FUNCTION

Find the points of inflection by setting y" x   0 .

 
y" x   4 3x2  6 x  1  0
3x  6 x  1  0
2

The trinomial above is not factorable. Solve for x by using quadratic formula.

  6   62  43 1
x
23
6  48
x
6
x  2.2 x  0.2
y  10.4 y  10.4

Thus, the curve has 2.2,10.4  and  0.2,10.4 as points of inflection.

Note: In some cases, finding the x-intercepts and the y-intercepts helps in roughly
sketching the graph of the given function. Likewise, knowing extra points on the curve is
sometimes necessary. Hence, assume extra points, when needed. Furthermore, it is will
be helpful if you determine horizontal and vertical asymptotes, if any.

2x
(b). y 
x 1
2

Solution: Find the critical points by setting y '  x   0

Differential Calculus Module 12 – Maximum and Minimum Value of Function Page 116
UNIT 3 – DERIVATIVE OF ALGEBRAIC FUNCTION

y' x  
x 2

 1 2   2 x2 x 
x 2
1 
2

2x2  2  4x2
0
1 x 2
 2

2  2x  0
2

 
2 1  x 2  1  x 1  x   0
x 1 x  1
y 1 y  1

Use the Second-derivative test to classify the critical points 1,1 and  1,1 .

y ' ' x  
x 2
 
 1  4 x   2  2 x 2 2 x 2  1 2 x 
2
  
x 2

1
4

y" x  
   
 4 x x  1 x  1  2  2 x2
2 2

x  1 2 2

 4 xx  13  x  2 2
y" x  
x  1 2 4

 4 x3  x  2
y" x  
x  1 2 3

   
1,1 : y" 1    , hence, 1,1 is a maximum point, concavity is downward.

   
 1,1 : y" x     , therefore,  1,1 is a minimum point, concavity is upward.

To find the points of inflection, set y" x   0

0

4 x 3  x2 
x 2
1  3


4x 3  x  0 2

x0 x   3  1.7 x  3  1.7
y0 y  0.9 y  0.9

Thus, the points inflection are ( ) ( ) ( )

As to the possible asymptotes of the curve, observe that when we equate the
denominator to zero, corresponding values of x are imaginary numbers. Thus, the curve

Differential Calculus Module 12 – Maximum and Minimum Value of Function Page 117
UNIT 3 – DERIVATIVE OF ALGEBRAIC FUNCTION

has no vertical asymptote. However, dividing both numerator and denominator of the
given function by the highest power of x which is yields equation Thus,
or the x-axis, is a horizontal asymptote.

1
(c). y  x 2 
x2
Solution: Find y'  x  and y"  x  .

y' x   2 x  2 x 3
 1
y ' x   2 x  3 
 x 
 x4  1 
0  2 3 
 x 
0  x4  1
 
0  x2  1 x2  1 

0  x  1x  1 x 2  1 
x 1 x  1
y2 y2

The critical points are 1,2  and  1,2  .

Use the Second-Derivative Test to classify the critical points.

1,2  : y" 1     


 1,2 : y" 1     

Therefore, both 1,2  and  1,2  are minimum points.

Find the points of inflection, if any. Set

Differential Calculus Module 12 – Maximum and Minimum Value of Function Page 118
UNIT 3 – DERIVATIVE OF ALGEBRAIC FUNCTION

 3
y" x   2  6 x 4  21  4 
 x 

 x4  3 
0  2 4 
 x 
0  x4  3

Therefore, no points of inflection since x-values are imaginary.

Equating the denominator to zero shows ( )is a vertical asymptote.


Analysis will show the curve has no x and y intercepts, thus, it does not cross the
coordinate axes.

Example 28. Find so that the curve will have a critical point
at ( )and pass through( ).

Condition (1): Points ( )and ( )satisfy equation of the curve .


( ):

( ):

Substitute ( )

-------- Equation (1)

Condition (2): At ( ), y’ = 0.

Differential Calculus Module 12 – Maximum and Minimum Value of Function Page 119
UNIT 3 – DERIVATIVE OF ALGEBRAIC FUNCTION

Differentiate y with respect to x.

( )

---------- Equation (2)

Subtract Equation (1) to Equation (2).

Substitute in Equation (2). ( )

Therefore, for the curve to have a critical point at ( )and pass through( ),

Example 29. Find so that the curve will pass


through( ), have critical point at ( ) and a point of inflection where .

Differentiate.

Condition (1): At , , y”=0.


( )

---------------- Equation (1)


Condition (2): At ( ), .
---------- Equation (2)
Condition (3). Points ( ) and ( ) satisfy given equation .
( ): ------------- Equation (3)
( )
Substitute into Equation (3) 1
--------------Equation (4)
Subtract Equation (2) to Equation (4). ---- Equation (5)
Substitute Equation (1) into Equation (5). ( )

Substitute into Equation (1) ( )

Substitute into Equation (2).

Therefore, so that the curve will pass through( ), will have


critical point at ( ) and have a point of inflection where ,

Differential Calculus Module 12 – Maximum and Minimum Value of Function Page 120
UNIT 3 – DERIVATIVE OF ALGEBRAIC FUNCTION

Example 30. Make the curve pass through ( ) and have


tangent line at the inflection point ( ) Find equation of
the curve.

Solution: The tangent line to the curve at its point of inflection is called an inflectional
tangent.

Differentiate to get

To get slope of the tangent line, reduce its equation to . Thus, at ( ) slope
of tangent is given by .

Condition (1). Points ( )and ( )satisfy .


( ) ( ) ( ) ( )

-------------- Equation (1)


( ) ( ) ( ) ( )

-----------------Equation (2)

Condition (2).At( ), . ( )

--------------------------- Equation (3)

Condition (3).At( ), . ( ) ( )

---------------Equation (4)

Subtract Equation (1) to Equation (2).

----------------Equation (5)

Subtract Equation (4) to Equation (5). --- Equation (6)

Substitute Equation (3) into Equation (6). ( )

Differential Calculus Module 12 – Maximum and Minimum Value of Function Page 121
UNIT 3 – DERIVATIVE OF ALGEBRAIC FUNCTION

Hence, ( )

Substitute into Equation (4). ( ) ( )

Substitute into Equation (2). ( ) ( )

Equation of the curve,

Differential Calculus Module 12 – Maximum and Minimum Value of Function Page 122
UNIT 3 – DERIVATIVE OF ALGEBRAIC FUNCTION

SAQ15
ACTIVITY 3.12 – M

NAME: ____________________________________________________ SCORE: ______________

SECTION: ___________DATE: _______________ PROF: __________________________________

I. Find the intercepts, the critical points and point of inflection for each of the following curves.
Determine the vertical and horizontal asymptotes, if any. Roughly sketch the curve.

1.

2.

3.

Differential Calculus Module 12 – Maximum and Minimum Value of Function Page 123
UNIT 3 – DERIVATIVE OF ALGEBRAIC FUNCTION

II. Find value of the arbitrary constants on the given equations.


1. If has a point of inflection at ( ), find values of

2. Determine a, b and c so that the curve will pass through point ( ) and
have tangent line to the curve at point ( ).

3. Determine a, b and c so that the curve will pass through ( ) and will have
a critical point at ( ).

Differential Calculus Module 12 – Maximum and Minimum Value of Function Page 124
UNIT 3 – DERIVATIVE OF ALGEBRAIC FUNCTION

ASAQ15
ACTIVITY 3.12 – M

NAME: ____________________________________________________ SCORE: ______________

SECTION: ___________DATE: _______________ PROF: __________________________________

I. Find the intercepts, the critical points and point of inflection for each of the following curves.
Determine the vertical and horizontal asymptotes, if any. Roughly sketch the curve.

𝐴𝑛𝑠𝑤𝑒𝑟

𝐴𝑛𝑠𝑤𝑒𝑟

𝐴𝑛𝑠𝑤𝑒𝑟

Differential Calculus Module 12 – Maximum and Minimum Value of Function Page 125
UNIT 3 – DERIVATIVE OF ALGEBRAIC FUNCTION

II. Find value of the arbitrary constants on the given equations.


1. If has a point of inflection at ( ), find values of 𝐴𝑛𝑠𝑤𝑒𝑟 𝑎 𝑏

2. Determine a, b and c so that the curve will pass through point ( ) and
have tangent line to the curve at point ( ). 𝐴𝑛𝑠𝑤𝑒𝑟 𝑎 𝑏 𝑐

3. Determine a, b and c so that the curve will pass through ( ) and will have a
critical point at ( ). 𝐴𝑛𝑠𝑤𝑒𝑟 𝑎 𝑏 𝑐

Differential Calculus Module 12 – Maximum and Minimum Value of Function Page 126
UNIT 3 – DERIVATIVE OF ALGEBRAIC FUNCTION

ACTIVITY 3.12 – N

NAME: ____________________________________________________ SCORE: ______________

SECTION: ___________DATE: _______________ PROF: __________________________________

I. Find the intercepts, the critical points and point of inflection for each of the following curves.
Determine the vertical and horizontal asymptotes, if any. Roughly sketch the curve.
1. ( )

2.

1.

Differential Calculus Module 12 – Maximum and Minimum Value of Function Page 127
UNIT 3 – DERIVATIVE OF ALGEBRAIC FUNCTION

II. Find value of the arbitrary constants on the given equations.


1. Make the curve pass through ( )( ) and have critical point at
( )

2. Find a, b, c and d so that the curve will pass through points ( )and
( ) and have inflection point at ( )

Differential Calculus Module 12 – Maximum and Minimum Value of Function Page 128
MODULE 13
OPTIMIZATION PROBLEMS

Specific Objectives:

At the end of the module, students must be able to:

1. Understand the concept of and steps in solving an


optimization problem.
2. Differentiate a constraint equation from optimization
equation and be able to set—up both equations.
3. Solve a given optimization problem.
UNIT 3 – DERIVATIVE OF ALGEBRAIC FUNCTION

MAXIMA AND MINIMA APPLICATIONS

If a contractor wants to construct a stainless steel cylindrical water tank with a


specified volume at the least possible cost, he is faced with the problem of finding a
minimum. Making the biggest window of constant perimeter and in the shape of a
rectangle surmounted by a semi-circle presents a problem of finding a maximum. Words
like biggest, largest, most, smallest, least, best, and others can be translated into
mathematical language in terms of maxima and minima.

One of the most important applications of derivative is illustrated on


maximum/minimum optimization problems. Many students find this application
intimidating because they are "word" problems, and no fixed pattern of solution exists to
these problems. However, with their patience, they can minimize their anxiety and
maximize their success with these problems by following the guidelines listed below:

1. Read the problem slowly and carefully. It is imperative to know exactly what the
problem is asking. If appropriate, draw a sketch or diagram of the problem to be
solved. Pictures are a great help in organizing and sorting out ones thought.
2. Identify the constant quantity in the given problem. Define variables to be used
and carefully label the picture or diagram with these variables. This step is very
important because it leads directly or indirectly to the creation of mathematical
equations.
3. Identify the quantity to be maximized or minimized and if it shall consist of more
than one variable, express it in terms of one variable (if possible and practical)
using the given conditions in the problem. Experience shows that most
optimization problems begin with two equations. One equation is a "constraint"
equation and the other is the "optimization" equation. The "constraint" equation is
used to solve for one of the variables. This is then substituted into the
"optimization" equation before differentiation occurs. Some problems may have
no constraint equation. Some problems may have two or more constraint
equations.
4. Then differentiate using the well-known rules of differentiation.
5. Verify that your result is a maximum or minimum value using the first or second
derivative test for extrema.

Example 31.What positive number added to its reciprocal gives the minimum sum?

Let : S be the minimum sum


x = the required positive number
1
= the reciprocal of the number
x

Differential Calculus Module 13 – Optimization Problems Page 129


UNIT 3 – DERIVATIVE OF ALGEBRAIC FUNCTION

 x  x 1 = f  x 
1
Optimization equation: S  x
x
dS 1
1 2
dx x
dS
For S to be a minimum,  0.
dx
1
0 1
x2
x2  1
0 2
x
0  x  1 x  1
x  1 (Answer) x  1 (Reject)

Verification of the critical value using the second-derivative test:

Since

Example 32. Find two positive integers having a sum of 132 and the sum of their cubes
has the minimum value.

Let: be the minimum sum of their cubes

be one positive number

Optimization equation: -------------------- Equation (1)


Constraint equation: ------------------------ Equation (2)
Therefore be the other number
Substitute Equation (2) into Equation (1).

 3x 2  3132  x   1
dS 2

dx

0  3 x 2  132  x 
2


0  x  17424  264x  x 2
2

0  17424  264x
264x  17424
x  66
Therefore, y  132  x  132  66  66
Verification of the critical value using the second-derivative test:

Differential Calculus Module 13 – Optimization Problems Page 130


UNIT 3 – DERIVATIVE OF ALGEBRAIC FUNCTION

Hence, the sum S is minimum at since is greater than zero.

Example 33. If the product of the square of one number by the cube of the other is to
be the greatest, find the two numbers if there sum equals 20.

Let be one number

of the square of one number by the cube of the other


number

Optimization equation:
Constraint equation:
Therefore, expression . Now, differentiate with respect to

[ ]

For Z to be the greatest,

Take note cannot be both zero and 20; otherwise, the product will have zero value
also. Hence, values and x= 20 are rejected. Hence, and and
the other number

Therefore, the numbers are 8 and 12.

MAXIMA-MINIMA PROBLEM USING IMPLICIT DIFFERENTIATION

Example 34. Resolve Example 33 by using implicit differentiation.


Let

Constraint Equation: ------------------- Equation (1)


Optimization Equation: --------------------- Equation (2)

No need to express in terms of one variable. Rather, to get , we use implicit


differentiation and equate the derivative to zero.
( )

Differential Calculus Module 13 – Optimization Problems Page 131


UNIT 3 – DERIVATIVE OF ALGEBRAIC FUNCTION

( )

Solve for . ------------------------- Equation (3)

Differentiate Equation (1) implicitly with respect to x.

------------------ Equation (4)

Substitute Equation (4) into Equation (3).

------------------- Equation (5)

Substitute Equation (5) into Equation (1) to get an equation in terms of one variable.
Solve the resulting equation for the value of the variable.

Substitution into Equation (1) or Equation (5) will give the corresponding value of y.

Hence,

Example 35.A rectangular lot of area 150 What should be the shape of the lot if it is
to be enclosed by the least amount of fencing?

Let and be the dimensions of derivative the rectangular lot


be the perimeter of the lot; the least amount, therefore, its derivative equals zero.
be the area of the lot; a constant
Method (1).

Constraint equation: ----------------- Equation (A)


----------------- Equation (1)

Optimization equation: -------- Equation (2)

Differential Calculus Module 13 – Optimization Problems Page 132


UNIT 3 – DERIVATIVE OF ALGEBRAIC FUNCTION

Substitute Equation (1) into equation (2):

( )

Differentiate with respect to variable x and equate its derivative to zero for to be the
least.

√ √

Substitute into Equation (1): √ √


Therefore, the rectangular lot is a square measuring √ by √ .

Method (2). Use implicit differentiation.

Differentiate Equation (A) with respect to x.

-------------- Equation (3)

Differentiate Equation (2) with respect to x, substitute Equation (3) into the resulting
equation and, then, equate the derivative of to zero.

-------- Equation (4)


( )

------------------ Equation (5)


Substitute Equation (5) into Equation (1).

√ √

Example 36.A rectangular lot is to be fenced off along a highway. If the fence on the
highway costs , on the other sides pesos per meter, find
the area of the largest lot that can be fenced off for pesos.

Differential Calculus Module 13 – Optimization Problems Page 133


UNIT 3 – DERIVATIVE OF ALGEBRAIC FUNCTION

Let be the total cost (in pesos) to fence the rectangular lot; a constant.

be the area ( of the lot; it has to be the largest.

Constraint equation:

------------ Equation (A)


---------------------Equation (1)
Optimization equation: ----------Equation (2)

Method (1).Express the quantity area A which is to be maximized in terms of a single


variable. Substitute Equation (1) into Equation (2), differentiate the resulting
equation, set the derivative to zero and solve for the value of the variable.

* + [ ]

[ ]

[ ]

------------ Equation (3)

Substitute the Equation (3) onto Equation (1).

-------------- Equation (4)

Substitute Equation (3) and Equation (4) into Equation (2).

* +

Method (2). Implicitly differentiate Equation (A) and Equation (2) with respect to x.
Since is a constant, its derivative with respect to x is zero.

Differential Calculus Module 13 – Optimization Problems Page 134


UNIT 3 – DERIVATIVE OF ALGEBRAIC FUNCTION

--------------- Equation (5)


Likewise, -------------Equation (6)

Substitute Equation (5) into Equation (6).Since is to be the greatest, its derivative
equals zero.

( )
( ) ---------Equation (7)
Substitute Equation (7) into Equation (1).
( )

Correspondingly, after substitution into Equation (7),

* +

* +

Example 37.A rectangular lot is bounded at the back by a river. It is to be fenced,


however, no fence is needed along the river and there is to be 24-ft opening in front. If
the fence along the front costs per foot, along the sides per foot, find the
dimensions of the largest lot which can be thus fenced in for $2208.

Constraint equation: Total cost C = ---------- Equation (1)

Optimization equation: ----------- Equation (2)

Area A is to be the greatest; =0.


( )
-------- Equation (3)

Substitute equation (3) into equation (1).

Differential Calculus Module 13 – Optimization Problems Page 135


UNIT 3 – DERIVATIVE OF ALGEBRAIC FUNCTION

( )

Substitute into Equation (3).

Example 38.A box is to be made of a piece of cardboard 9 inches square by cutting


equal squares out of the corners and turning up the sides. Find the volume of the
largest box that can be made in this way.

Volume of the box V is to be the largest; hence, its derivative equals zero.

Optimization equation:

Equate each factor to zero and solve for the value of x.

The value is rejected since when ( )

Therefore, inches.
Take note that this problem has no constraint equation.

Therefore, the volume of the largest box that can be made as described above is
* ( )+

Example 39. The strength of a rectangular beam is proportional to the breadth and the
square of the depth. Find the shape of the strongest beam that can be cut from a log of
diameter 24 inches.

Let W be the breadth of the beam and L be its depth .

Differential Calculus Module 13 – Optimization Problems Page 136


UNIT 3 – DERIVATIVE OF ALGEBRAIC FUNCTION

Optimization equation: Strength of the beam ; is to be a maximum,

( )

------------- Equation (1)

By Pythagorean Theorem, ------------- Equation (2)

----------------------- Equation (3)

Substitute Equation (3) into Equation (1).


( ) 24
L


For the beam to be the strongest one, the depth L must be √ times the breadth W.

Example 40.Find the shortest distance from the point (5, 0) to the curve 2y2 = x3.

Let (x, y) be the point on the curve nearest to point

be the shortest distance and .

Method (1).Use distance formula between two points, differentiate implicitly with respect
to x.

Optimization equation: √ -------- Equation (A)

[ ]

-------------- Equation (1)

Constraint equation: 2y2 = x3

Differentiate implicitly the given equation of the curve.

-------------- Equation (2)

Substitute Equation (1) into Equation (2).

* +

Differential Calculus Module 13 – Optimization Problems Page 137


UNIT 3 – DERIVATIVE OF ALGEBRAIC FUNCTION

x–2=0

We reject since corresponding y-value is imaginary. Thus, we use and


substitute it on the given equation of the curve to get the corresponding y-value.

The shortest distance √ √ √

Method (2). From the given equation of the curve,

Substitute on Equation (A), differentiate with respect to x and equate the derivative to
zero.

[ ]

x–2=0

Compare the results of Method (1) with those of Method (2). Thus, the shortest
distance √ .

Example 41.A cylindrical can is to hold .If the material for the top and bottom
costs and the material for the side costs , find the radius r and height h of
the most economical dimension.

The most economical dimensions refer to the dimensions of a cylindrical can that will
entail the minimum cost of manufacturing it.

Let r be the radius of the circular base and variable h the height of the cylinder.
C be the least cost of manufacturing a cylindrical can

Differential Calculus Module 13 – Optimization Problems Page 138


UNIT 3 – DERIVATIVE OF ALGEBRAIC FUNCTION

V be the volume of the cylindrical can;

Constraint equation: h ---------- Equation (1)

Optimization equation: Total Cost C ------------------- Equation (2)

Substitute Equation (1) into Equation (2). ( )

For to be the least cost, . [ ]


r

Substitute into Equation (1).


The least cost to manufacture a cylindrical
can.

Example 42.Find the dimensions of a rectangle of maximum perimeter that can be


inscribed in a circle of diameter 10 cm.
From the figure at the right, √ .
Square both sides of the equation.


Solve for the length y. √ √ √
Therefore, the rectangle of maximum perimeter is a square of dimensions √ √

Example 43.A trapezoidal gutter is to be made from a strip of tin by bending up the
edges. If the cross-section of the gutter is shown below, with the lower
base what width across the top gives maximum carrying
capacity?

Let A be the area of the trapezoidal cross-section

Differential Calculus Module 13 – Optimization Problems Page 139


UNIT 3 – DERIVATIVE OF ALGEBRAIC FUNCTION

𝑥
𝑥 𝑥 𝑥

h h

The carrying capacity is maximum if the area is maximum.

Optimization equation: ------------ Equation (1)

[ ]
Constraint equation: √ ( ) =√

√ --------------Equation (2)

Substitute Equation (2) into Equation (1). * √ +


[ ( ) √ ]

[√ √ * +]

Therefore, the width across the top that will give the maximum carrying capacity of the
gutter is 8 inches.

Example 44.A piece of wire 40cm long is to be cut into two pieces. One piece will be
bent to form a circle; the other will be bent to form a square. Find the lengths of the two
pieces that cause the sum of the area of the circle and square to be a minimum.
Let be the perimeter of the square of edge
be the circumference of the circle of radius .

Differential Calculus Module 13 – Optimization Problems Page 140


UNIT 3 – DERIVATIVE OF ALGEBRAIC FUNCTION

Therefore, ,

Likewise, ,

Hence, ( ) ( )
Differentiate.

cm

And, cm

Therefore, the lengths of the wire that will give the minimum combined area of the circle
and the square are 22.4 cm and 17.6 cm.

Example 45.A Norman window consists of a rectangle surmounted by a semicircle.


What shape admits the greatest amount of light if perimeter of the window is 30 feet?

The greatest amount of light is admitted by the window if its area is a maximum.
Furthermore, the diameter of the semi-circular part of the
window is equal to the width of the rectangular portion.

Constraint equation:

--------------------------- Equation (1)


Optimization equation:
----------- Equation (2)

Substitute Equation (1) into Equation (2).

Differential Calculus Module 13 – Optimization Problems Page 141


UNIT 3 – DERIVATIVE OF ALGEBRAIC FUNCTION

( ) ( )

Differentiate. [ ]

Hence, ( )

( )
Substitute into Equation (1).

Taking the ratio of the height to the width

Therefore, for the maximum amount of light be admitted, the width of the window equals
its height.

Example 46.Two posts, one 8 feet high and the other 12 feet high, stand 15 feet apart.
They are to be supported by wires attached to a single stake at ground level. Where
should the stake be placed so that the least amount of wire is used?
Let be the distance of the stake from the shorter pole.

From the figure at the right, √


𝐿
𝐿 12’
Likewise, √ 8

Thus, total length of wires used
𝑥 𝑥
√ √

Differentiate.
√ √

Differential Calculus Module 13 – Optimization Problems Page 142


UNIT 3 – DERIVATIVE OF ALGEBRAIC FUNCTION

√ √

√ (√ )
√ (√ )

(√ ) √

Solve the radical equation. √ √


[ ]

(Rejected)

Thus, the stake should be positioned 6 m from the shorter post or 9 m from the longer
post.

Differential Calculus Module 13 – Optimization Problems Page 143


UNIT 3 – DERIVATIVE OF ALGEBRAIC FUNCTION

SAQ16
ACTIVITY 3.13 – O

NAME: ____________________________________________________ SCORE: ______________

SECTION: ___________DATE: _______________ PROF: __________________________________

Solve the following maxima-minima problems using algebraic function.

1. Find two numbers having a sum of 9 if the product of one number by the square of the other
is a maximum.

2. What is the smallest sum of two numbers if their product is 16?

3. A rectangular field of area is to be fenced off along a straight river. If no fencing is


required along river, find the dimensions of the lot that will require the least amount of
fencing.

4. Find the area of the largest rectangular garden that can be made so that one side of the
house serves as the natural boundary and 10 m of fencing material is required for the
remaining three sides.

Differential Calculus Module 13 – Optimization Problems Page 144


UNIT 3 – DERIVATIVE OF ALGEBRAIC FUNCTION

5. Find the coordinates of the point or points on the curve which are nearest to
the point .

6. A manufacturer makes aluminum cups of volume and in the form of right circular
cylinder open at the top. What dimensions of the cup will require the least amount of
material?

7. A closed right circular cylinder (that means including the top and the bottom) has a surface
area of . What should the radius and altitude be in order to provide the largest
possible volume?

Differential Calculus Module 13 – Optimization Problems Page 145


UNIT 3 – DERIVATIVE OF ALGEBRAIC FUNCTION

ASAQ16
ACTIVITY 3.13 – O

NAME: ____________________________________________________ SCORE: ______________

SECTION: ___________DATE: _______________ PROF: __________________________________

Solve the following maxima-minima problems using algebraic function.

1. Find two numbers having a sum of 9 if the product of one number by the square of the other
is a maximum. 𝐴𝑛𝑠𝑤𝑒𝑟: 𝑎𝑛𝑑

2. What is the smallest sum of two numbers if their product is 16? 𝐴𝑛𝑠𝑤𝑒𝑟: 𝑎𝑛𝑑

3. A rectangular field of area is to be fenced off along a straight river. If no fencing is


required along river, find the dimensions of the lot that will require the least amount of
fencing. 𝐴𝑛𝑠𝑤𝑒𝑟: 𝑥 𝑎𝑛𝑑

4. Find the area of the largest rectangular garden that can be made so that one side of the
house serves as the natural boundary and 10 m of fencing material is required for the
remaining three sides. 𝐴𝑛𝑠𝑤𝑒𝑟: 𝑚 𝑥 𝑚

Differential Calculus Module 13 – Optimization Problems Page 146


UNIT 3 – DERIVATIVE OF ALGEBRAIC FUNCTION

5. Find the coordinates of the point or points on the curve which are nearest to
the point . 𝐴𝑛𝑠𝑤𝑒𝑟:

6. A manufacturer makes aluminum cups of volume and in the form of right circular
cylinder open at the top. What dimensions of the cup will require the least amount of
material? 𝐴𝑛𝑠𝑤𝑒𝑟: 𝑟 𝑐𝑚 𝑐𝑚

7. A closed right circular cylinder (that means including the top and the bottom) has a surface
area of . What should the radius and altitude be in order to provide the largest
possible volume? 𝐴𝑛𝑠𝑤𝑒𝑟: 𝑟 𝑐𝑚 𝑐𝑚

Differential Calculus Module 13 – Optimization Problems Page 147


UNIT 3 – DERIVATIVE OF ALGEBRAIC FUNCTION

ACTIVITY 3.13 – P

NAME: ____________________________________________________ SCORE: ______________

SECTION: ___________DATE: _______________ PROF: __________________________________

Solve the following optimization problems.

1. Find two positive numbers such that their product is 192 and the sum of the first plus three
times the second is a minimum.

2. An open box whose base is a square is to be made and will enclose . Find the dimensions
of the box that will minimize the material needed to construct the box.

3. A piece of cardboard measures 30 cm by 16 cm, corners cut-out and sides folded up to form a
box. Find the height of the box that will give the maximum volume.

Differential Calculus Module 13 – Optimization Problems Page 148


UNIT 3 – DERIVATIVE OF ALGEBRAIC FUNCTION

4. The combined perimeter of an equilateral triangle and a square is 10. Find the dimensions of the
triangle and square that produce a minimum total area.

5. Four feet of wire is to be used to form a square and a circle. How much of the wire should be
bent into a square and how much bent for the circle to enclose the maximum total area?

6. A tank with a rectangular base and rectangular sides is open at the top. It is to be constructed so
that its width is 4 meters and its volume 36 cubic meters. If building the tank costs $10/sq. m.
for the base and $5/sq. m. for the sides, what is the cost of the least expensive tank and what
are its dimensions?

Differential Calculus Module 13 – Optimization Problems Page 149


UNIT 3 – DERIVATIVE OF ALGEBRAIC FUNCTION

7. A poster having a total area of is to be printed in a way so that there will be inch
margin on the bottom, 1 inch margin on the right, 2 inch margin on the left and 4 inch margin on
the top. What dimensions of the poster will give the largest printed area?

8. Which point on the graph of y = x is closest to the point (5, 0)?

9. A rectangular page is to contain 24 square inches of print. The margins at the top and bottom of
the page are inches. The margins of each side are 1 inch. What should be the dimensions of
the page so that the least amount of paper is used?

Differential Calculus Module 13 – Optimization Problems Page 150


MODULE 14
TIME-RATES

Specific Objectives:

At the end of the module, students must be able to:

1. Understand the concept of and steps in solving a time-rate


problem.
2. Identify the quantities in the problem that change with
time..
3. Solve a given time-rate problem.
UNIT 3 – DERIVATIVE OF ALGEBRAIC FUNCTION

TIME – RATES

If the value of a variable depends on the time , then, is called its time-rate or
rate of change with respect to time. When two or more quantities, all functions of t, are
related by an equation, the relation between their time-rates may be attained by
differentiating both sides of the equation with respect to time Basic time-rates are
velocity , acceleration , discharge and angular speed . If the
time-rate is positive, it means the quantity is increasing with time.

Steps in Solving Time Rates Problem

1. Identify what quantities are changing and what are fixed with time.
2. Assign variables to those that are changing and appropriate value (constant) to
those that are fixed.
3. Find an equation relating all the variables and constants in Step 2.
4. Differentiate the equation with respect to time.

Example 47.Water is flowing into a vertical cylindrical tank at the rate of . If


the radius of the tank is 4 feet, how fast is the surface rising?

Solution: The quantities that are changing with time are the
depth and the volume of the water in the cylindrical tank.
The radius of the tank remains constant with time at a value of
4 feet. The required quantity in this problem is the time-rate
or . The time-rate of volume it is positive, just
like time-rate of depth , since the volume of water in the tank increases with time.

At any time volume of water in the cylindrical can is given by equation

( )

Differentiate both sides of the equation above with respect to time.

Example 48.A triangular trough 10 ft long is 4 ft across the top, and 4 ft deep. Water
flows in at the rate of . How fast is the surface rising when the water is 6 in
depth?

Differential Calculus Module 14 – Time-Rates Page 149


UNIT 3 – DERIVATIVE OF ALGEBRAIC FUNCTION

Solution:
The volume of water in the triangular trough
at any time equals the volume of a triangular prism
whose altitude is . The variables
with time are volume of water and its depth . The
other given constants are ; same as
the dimensions of the trough .
Required quantity is when .

Volume of water at any time * +( )


-------------- Equation (1)
By similar triangles: ----------- Equation (2)

Method (1). Substitute Equation (2) into Equation (1), then, differentiate the resulting
equation with respect to time. ( )

( )
( )

Method (2). Differentiate Equation (1) with respect to time.


* + ----------- Equation (3)

Differentiate Equation (2) with respect to time. ----------- Equation (4)

Substitute Equation (2) and Equation (4) into Equation (3). * +


* + * +

Example 49.A ladder 20 feet long leans against a vertical


wall. If the top slides downward at the rate of 2 ft/sec, find
how fast the lower end is moving when it is 16 feet from the
wall.

Solution: The quantity that remains fixed with time is the length of the ladder which is
equalto20 feet. The varying quantities with time are the distance of the foot or lower
end of the ladder from the wall and the distance of the top of the ladder from the

Differential Calculus Module 14 – Time-Rates Page 150


UNIT 3 – DERIVATIVE OF ALGEBRAIC FUNCTION

ground. Required quantity is time-rate of or when with the time-rate of


or . It is negative since decreases with time.

The equation that relates quantities dependent on time is obtained by Pythagorean


Theorem. ( ) ----------- Equation (1)

Differentiate with respect to time.


------------ Equation (2)

Substitute ( ) get the corresponding value of .


( ) ( )

Substitute into Equation (2). ( )

Take note that the computed is positive since is increasing with time.

Example 50.A man 6 feet tall walks away from a lamp post 16 feet high at the rate of 5
miles per hour. How fast does his shadow lengthen?

Solution: Let the length of the man’s shadow at any time be and his distance from the
lamp post be the given time-rate is time-rate is .

By similar triangles:
( )

Differentiate with respect to time ( )

Hence, the shadow of the man is lengthening at the rate of

Differential Calculus Module 14 – Time-Rates Page 151


UNIT 3 – DERIVATIVE OF ALGEBRAIC FUNCTION

SAQ17
ACTIVITY 3.14 – Q

NAME: ____________________________________________________ SCORE: ______________

SECTION: ___________DATE: _______________ PROF: __________________________________

Solve the following time-rate problems.

1. A conical cistern is 10 feet across the top and 12 feet deep. Water runs into the cistern at the
rate of ⁄ . How fast is the surface rising when the water is 8 feet deep?

2. A point moves along the upper half of parabola in such a way


that ⁄ √ . Find ⁄ when

3. The two equal sides of an isosceles triangle are each equal to 13 inches. If the third side is
increasing at ⁄ , at what is the altitude drawn to this side changing when the altitude is
12 inches?

Differential Calculus Module 14 – Time-Rates Page 152


UNIT 3 – DERIVATIVE OF ALGEBRAIC FUNCTION

4. Ohm’s law for a certain electrical circuit states that , where is the voltage in volts, the
current in amperes and the resistance in ohms. If the circuit heats up and the voltage is kept
constant, the resistance increases at the rate of ⁄ . Find the rate at which the current
decreases when and is kept constant at 10 volts.

5. The adiabatic law (no gain or heat loss) for the expansion of air is , where is the
pressure in ⁄ , V is the volume in cubic inches, and is a constant. At a specific instant, the
pressure is ⁄ an is increasing at the rate of ⁄ each second. If , what is the
rate of change of the volume at this instant?

6. The side of an equilateral triangle is 10 inches and is lengthening at the rate of ⁄ . How
fast is the area increasing?

Differential Calculus Module 14 – Time-Rates Page 153


UNIT 3 – DERIVATIVE OF ALGEBRAIC FUNCTION

ASAQ17
ACTIVITY 3.14 – Q

NAME: ____________________________________________________ SCORE: ______________

SECTION: ___________DATE: _______________ PROF: __________________________________

Solve the following time-rate problems.

1. A conical cistern is 10 feet across the top and 12 feet deep. Water runs into the cistern at the
rate of ⁄ . How fast is the surface rising when the water is 8 feet deep?
𝑖𝑛
𝐴𝑛𝑠𝑤𝑒𝑟:
𝑚𝑖𝑛

2. A point moves along the upper half of parabola in such a way


that ⁄ √ . Find ⁄ when 𝐴𝑛𝑠𝑤𝑒𝑟:

3. The two equal sides of an isosceles triangle are each equal to 13 inches. If the third side is
increasing at ⁄ , at what is the altitude drawn to this side changing when the altitude is
12 inches?
𝑖𝑛
𝐴𝑛𝑠𝑤𝑒𝑟:
𝑚𝑖𝑛

Differential Calculus Module 14 – Time-Rates Page 154


UNIT 3 – DERIVATIVE OF ALGEBRAIC FUNCTION

4. Ohm’s law for a certain electrical circuit states that , where is the voltage in volts, the
current in amperes and the resistance in ohms. If the circuit heats up and the voltage is kept
constant, the resistance increases at the rate of ⁄ . Find the rate at which the current
decreases when and is kept constant at 10 volts.
𝑎𝑚𝑝
𝐴𝑛𝑠𝑤𝑒𝑟:
𝑠𝑒𝑐

5. The adiabatic law (no gain or heat loss) for the expansion of air is , where is the
pressure in ⁄ , V is the volume in cubic inches, and is a constant. At a specific instant, the
pressure is ⁄ an is increasing at the rate of ⁄ each second. If , what is the
rate of change of the volume at this instant?
𝑖𝑛
𝐴𝑛𝑠𝑤𝑒𝑟:
7 𝑠𝑒𝑐

6. The side of an equilateral triangle is 10 inches and is lengthening at the rate of ⁄ . How
fast is the area increasing?
𝑖𝑛
𝐴𝑛𝑠𝑤𝑒𝑟: √
𝑠𝑒𝑐

Differential Calculus Module 14 – Time-Rates Page 155


UNIT 3 – DERIVATIVE OF ALGEBRAIC FUNCTION

ACTIVITY 3.14 – R

NAME: ____________________________________________________ SCORE: ______________

SECTION: ___________DATE: _______________ PROF: __________________________________

Solve the following time-rate problems.

1. A kite is 24 feet high with 25 feet cord out. If the kite moves horizontally at 4 miles per hour directly
away from the boy flying it, how fast is the cord being paid out?

2. A rectangular through is 9 feet long and 4 feet wide. How fast does the surface rise if water flows at
the rate of ?

3. . A triangular through is 10 feet long, 6 feet wide across the top and 3 feet deep. If water flows at the
rate of , find how fast is the surface rising when the water is 6 inches deep.

4. A rock is dropped into a pool of water, causing ripples to form in expanding outward circles. The
radius r of a ripple is increasing at a rate of feet per second. When the radius is 8 feet, at what rate is
the area A of the water inside the ripple changing?

Differential Calculus Module 14 – Time-Rates Page 156


UNIT 3 – DERIVATIVE OF ALGEBRAIC FUNCTION

5. A rock is dropped into a pool of water, creating ripples which move outward from it. One of these
ripples creates a circle with an area increasing at a rate of 28 ft2/sec. When the area is square feet,
at what rate is the radius expanding?

6. A tank of water in the shape of a cone has a base radius of 4 feet and a height of 12 feet. If water is
leaking at the bottom at a constant rate of 2 ft3/hour, at what rate is the depth of the water in the tank
changing when the depth of the water is 4 feet. At what rate is the radius of the top of the water
changing when the depth of water is 8 feet?

Differential Calculus Module 14 – Time-Rates Page 157


MODULE 15
DERIVATIVES OF
TRIGONOMETRIC FUNCTION
AND THEIR APPLICATIONS

Specific Objectives:

At the end of the module, students must be able to:

1. Know and use correctly the differentiation formulas for


trigonometric function.

2. Apply the differentiation formulas for trigonometric


function in solving problems on slope of tangent and
normal, rectilinear motion, angle between curves,
optimization problems and time-rates.
UNIT 4 – DERIVATIVE OF TRANSCENDENTAL FUNCTION

TRANSCENDENTAL FUNCTIONS
Transcendental function is a function that is not an algebraic function. Such a function cannot
be expressed as a solution of a polynomial equation whose coefficients are themselves polynomials
with rational coefficients. Transcendental functions include trigonometric functions, inverse
trigonometric functions, exponential and logarithmic functions.

DERIVATIVE OF TRIGONOMETRIC FUNCTION

Let be a differentiable function of x.

1. 4.

2. 5.

3. 6.

It is but wise to review some basic concepts and the trigonometric relations to facilitate the
differentiation and simplification of derivatives of trigonometric functions.

TRIGONOMETRIC FUNCTIONS OF ANGLE A WITH POINT ( ) ON ITS TERMINAL SIDE

1. 4.

2. 5.

3. 6.

THE UNIT CIRCLE AND TRIGONOMETRIC FUNCTIONS OF QUADRANTAL ANGLES


(0 1)

( 1 0) (1 0)

(0 1)

Differential Calculus Module 15 – Derivative of Trigonometric Functions and their Applications Page 160
UNIT 4 – DERIVATIVE OF TRANSCENDENTAL FUNCTION

TRIGONOMETRIC FUNCTIONS OF SPECIAL ANGLES

Use the SOH-CAH-TOA definitions of the trigonometric functions of the special acute acute angles.

BASIC RELATIONS FOR TRIGONOMETRIC FUNCTIONS


1. 5. 1

2. 6. 1

3. 7. 1

4. 8. 1

NEGATIVE ANGLE FORMULAS

1. ( ) 2. ( ) 3. ( )

SUM AND DIFFERENCE OF TWO ANGLES IDENTITIES

1. ( ) 3. ( )

2. ( ) 4. ( )

DOUBLE ANGLE IDENTITIES

1. 2 2 4. 2 1 2

2. 2 5. 2

3. 2 2 1 6. 2

Differential Calculus Module 15 – Derivative of Trigonometric Functions and their Applications Page 161
UNIT 4 – DERIVATIVE OF TRANSCENDENTAL FUNCTION

HALF - ANGLE IDENTITIES

1. √ 1
4. A
2

2. √ 1
5. A
2

3. 6.

SUM TO PRODUCT IDENTITIES


1. 2 ( ) ( )
2. 2 ( ) ( )
3. 2 ( ) ( )
4. 2 ( ) ( )

Find the

Example1 ( 4)

Solution: Use
With ( 4) , 3( 4) (2 ) 6 ( 4)
( 4) [6 ( 4) ]
6 ( 4) ( 4)

Example √
Solution: Use
With √

1
√ √ [ ]
2√
√ √

Example 2
Solution: Use the product formula
With
2 ( 2 )(2) 2 2

Differential Calculus Module 15 – Derivative of Trigonometric Functions and their Applications Page 162
UNIT 4 – DERIVATIVE OF TRANSCENDENTAL FUNCTION

1
[ (2 2 ) 2 ( )]
2
2 2
1 1
[ (2 1)] (2 )( )
2
2
2
2 2

Example (2 )
Solution: Use again the product formula
With (2 )
3 (4 )[ (2 )]
(4 )[ (2 )] 3 (2 )
[4 (2 ) 3 (2 )]

Example

Solution: Use the quotient formula ( )


With 1 2 1 2
( 2 )(2) 2 2 ( 2 )(2) 2 2
(1 2 )(2 2 ) (1 2 )( 2 2 )
(1 2 )
22 [1 2 1 2 ]
(1 2 )
2( 2 )(2)
(1 2 )
4 2
(1 2 )

Example ( )
Solution: Use the power formula with ( ) 4
1 1 1
( ) ( ) ( )
1 1
( ) ( )( 3 )
3 1 1
( ) ( )
1 3 1 1
4 ( )[ ( ) ( )]

Differential Calculus Module 15 – Derivative of Trigonometric Functions and their Applications Page 163
UNIT 4 – DERIVATIVE OF TRANSCENDENTAL FUNCTION

( ) ( )
( )
However, ( ) . Therefore, .

Example ( )
Solution: Again, use the power formula
1 1
( )( ) [ ]
( )
[– ( ) ]
( )
[ ]
( )

But, from Pythagorean Relation:1


Therefore, it follows that 1
(– ) ( )( )
Substitution yields, ( ) ( )

Thus, ( ) ( )

1 1
( ) ( )
3 1

3( ) (1 )

3√ (1 )

Example 8 ( )
Solution: Use Implicit differentiation.
( )(1 )
( ) ( )
( ) ( )
[1 ( )]
( )
1 ( )

Example 2
Solution: Use implicit differentiation. * + [ ] ( ) 0

Differential Calculus Module 15 – Derivative of Trigonometric Functions and their Applications Page 164
UNIT 4 – DERIVATIVE OF TRANSCENDENTAL FUNCTION

( )
dy y sin x  cos y

dx cos x  x sin y

DERIVATIVE AS SLOPE OF THE TANGENT LINE

Example 10. Find equation of the tangent line to the curve at the point where .

Solution: To get the equation of the tangent line, we need to know its slope and the point of tangency.

Find the slope the tangent by differentiating the given function. Hence,
( )
At ( ) (0) ( 1) 1

Substitute to the given equation of the curve to get the corresponding value of
( 1)

Thus, point of tangency is at ( )


Use the point-slope form to get equation of the tangent line at( )
( ) 1( )

Therefore, at , equation of the tangent line to curve is

Example 11. At what point does the tangent line to the curve 4 2 parallel to the
8 3 0

Solution: First, differentiate the given function and equate the resulting derivative to the slope of the
given line. They are parallel lines, so their slopes are equal. Solve for the value of and the
corresponding

4( 2 )(2) 8 2

At the unknown point, 8


8 8 2
2 1
( 2 1)( 2 1) 0
2 1 2 1
2 0 2 4 2 3
0 2

Differential Calculus Module 15 – Derivative of Trigonometric Functions and their Applications Page 165
UNIT 4 – DERIVATIVE OF TRANSCENDENTAL FUNCTION

Substitute the x-value on equation 4 2 .


When 0 0 0
When 0
When 0
When 3 0

Therefore, the four points on the curve 4 2 where the tangent line is parallel to line
8 3 0 are (0 0) ( 0) ( 0) ( 0)

MAXIMA-MINIMA APPLICATIONS USING TRIGONOMETRIC FUNCTIONS


The use of trigonometric functions facilitates solution to many maxima-minima applications.To
do it, identify the constant terms and the quantity/variable to be maximized (or minimized),
differentiate that quantity/variable, equate the derivative to zero and then, solve for the value of the
variable left on the resulting equation.

Example 12. Find the shape of the rectangle of maximum perimeter inscribed in a circle of diameter

Let x and y be the breadth and length of the rectangle whose perimeter P needs to be the
maximum;
be the acute angle the diameter makes with the breadth of the rectangle, as shown on
the accompanying figure.

The diameter of the circle is a constant quantity in this problem.

Optimization equation: 2 2 ------------ Equation (1)

Constraint equations:
----------------- Equation (2)

Likewise,
------------------ Equation (3)

Method (1).Substitute Equation (2) and Equation (3) into Equation (1) to express perimeter P, the
quantity to be maximized, in terms of variable .
2 2
2 ( )

Differentiate P and set its derivative to zero. 2 [ ]


0 2 [ ]
0

Differential Calculus Module 15 – Derivative of Trigonometric Functions and their Applications Page 166
UNIT 4 – DERIVATIVE OF TRANSCENDENTAL FUNCTION

Solve the above trigonometric equation for . 0

Divide the above equation by 1


1
45

Solve for by substituting into Equation (2) and Equation (3).


45

45

Therefore the rectangle of biggest perimeter that can be cut from a circle of radius has its breadth
equals its length; or, the rectangle is a square.

Method (2). Differentiate Equation (1) with respect to one of the variables, say set 0 and solve
for 0 2 2
1-------------- Equation (A)

Differentiate Equation (2) with respect to


1 ( )
------------ Equation (B)

Differentiate Equation (3) with respect to


-------- Equation (C)

Substitute Equation (A) and Equation (B) into Equation (C).


1 ( )
1
1
45

Therefore, we get same results as Method 1.



45

45

Differential Calculus Module 15 – Derivative of Trigonometric Functions and their Applications Page 167
UNIT 4 – DERIVATIVE OF TRANSCENDENTAL FUNCTION

Example 13.The strength of rectangular beam is proportional to the product of the breadth and the
square of the depth. Find the shape of the strongest beam that can be cut from a log of given size.

Let be the strength

be the acute angle the given diameter D of the log makes with the diameter d of the beam.

Optimization equation: ------------ Equation (1)

Constraint equations:
----------- Equation (2)
And,
----------- Equation (3)

Substitute Equation (2) and Equation (3) into Equation (1).


( )

Differentiate with respect to and set 0

[ (2 )( ) ( )]
0 [ (2 )]
0 [2 (1 )]
0 (3 1)
0 0 (Rejected)

Likewise, 3 1 0


Take note that is rejected since is an acute angle.

To avoid using calculator to get the value of from , draw a right triangle having as an

acute angle.

Therefore, ( ) 𝜃
√ √

√ √
And, ( ) √2 ( ) 1
√ √ √

Thus, the height is equal to √2 times the breadth.

Differential Calculus Module 15 – Derivative of Trigonometric Functions and their Applications Page 168
UNIT 4 – DERIVATIVE OF TRANSCENDENTAL FUNCTION

Example 14. A trapezoidal gutter is to be made, from a strip of metal 22 inches wide by bending up the
edges. If the base is 14 inches wide, what width across the top gives the greatest carrying capacity?

Let be the acute angle the leg of the trapezoidal cross-section makes with the horizontal

The gutter having the maximum cross-sectional area will have the greatest carrying capacity. Thus,
optimization equation: 2 +

2* + 14( ) 14 -------------- Equation (1)

From the given figure, constraint equations:


y
x 1 x
4 ------------ Equation (2) 4
And

4 ------------ Equation (3)


4 h 4
Substitute Equation (2) and Equation (3) into Equation (1).
(4 )(4 ) 14(4 ) 1
16 56 4

Differentiate the above equation with respect to


16[ ( ) ( )] 56
Set 0, express the resulting equation in terms of a single trigonometric function and solve for
value of variable
0 16[ ] 56
0 16[( 1) ] 56
0 2[2 1] 7
0 4 7 2
0 (4 1)( 2)
0 4 1 2 0

However, 2 is rejected since 1 1


Substitute into Equation (2). 4( ) 1
Width across the top that will give the gutter the maximum carrying capacity:
1 14 1 16

Differential Calculus Module 15 – Derivative of Trigonometric Functions and their Applications Page 169
UNIT 4 – DERIVATIVE OF TRANSCENDENTAL FUNCTION

Example 15. A pole 24 feet long is carried horizontally along a corridor 8 feet wide and into a second
corridor at right angles to the first. How wide must the second corridor be?

Let be the acute angle which the pole makes with the second corridor
be the minimum width of the second corridor that will permit the passage of the pole
from the first corridor.
Constraint equation: 8 ------------- Equation (1)
--------- Equation (2)

Optimization equation: 24
24 8 ------------------ Equation (3)

Substitute Equation (1) and Equation (2) into equation (3).


24 8

( )
24

24
24 8 ----Equation (4)

Differentiate Equation (4) with respect to


24 8
Set 0 and solve for value of variable
0 8 (3 )

0
3 1

√ 0 693361
46 10

Substitute 46 10 into Equation (4).


24 46 10 8 46 10
8 98 feet

Therefore, the second corridor needs to be at least 8.98 feet to permit the passage of the 27-foot pole
from the first corridor to the second corridor.

Differential Calculus Module 15 – Derivative of Trigonometric Functions and their Applications Page 170
UNIT 4 – DERIVATIVE OF TRANSCENDENTAL FUNCTION

TIME-RATE PROBLEM INVOLVING TRIGONOMETRIC FUNCTION

Example 16. A ladder 20 feet long leans against a vertical wall. If the top slides downward at the
rate of 2 ft/sec, find

a). how fast is the inclination of the ladder changing when the lower end is 16 feet
from the wall
b). the rate by which the foot of the ladder moves away from the wall
Solution:
Let be the inclination of the ladder at anytime
be the distance of the top of the ladder from the ground at anytime
be the distance of the foot of the ladder from the wall at anytime

Given quantities are the lengths of the ladder equal to 20 ft, 2 ⁄ and the required are
and .
20
a). We choose now a trigonometric function that involves 𝑦
and since time rate of y is known.
𝜃
20 𝑥

Differentiate above equation with respect to time t.


20 ------- Equation 1

Quantity 2 . Observe that a negative sign is assigned to it since y is decreasing with time t
while expectedly is negative since it also decreases with time. From the triangle, when 16,
. Substituting all known values into Equation 1 yields
2 20 ( )
( )

Thus, the inclination decreases by rad per second at that particular time the foot of the ladder is 16
feet away from the wall.

b). To find the rate by which the foot of the ladder moves horizontally away from the wall, we use
Pythagorean theorem to define relation between x and y. The rate is expected to be a positive
quantity since the distance of the foot of the ladder from the wall increases with time.
20 ---------- Equation 2
Differentiate with respect to time. 0 2 2

Differential Calculus Module 15 – Derivative of Trigonometric Functions and their Applications Page 171
UNIT 4 – DERIVATIVE OF TRANSCENDENTAL FUNCTION

0 --------- Equation 3

Use Equation 2 to find the value of y at that particular time when 16


400 (16)
√400 256 12

Substitute known quantities into Equation 3.


0 16 12( 2)
0 16 24

Therefore, we say that the foot of the ladder moves horizontally away from the wall at the rate of
feet per second.

Differential Calculus Module 15 – Derivative of Trigonometric Functions and their Applications Page 172
UNIT 4 – DERIVATIVE OF TRANSCENDENTAL FUNCTION

SAQ18
ACTIVITY 4.15 – A

NAME: ____________________________________________________ SCORE: ______________


SECTION: ___________DATE: _______________ PROF: __________________________________

Differentiate each of the following using the appropriate differentiation formula.

1. 1
6
3

2. 3 (2 3 1) 7 2

3. √1 2 8 √4

4 2 2 9.

5. 10.

Differential Calculus Module 15 – Derivative of Trigonometric Functions and their Applications Page 173
UNIT 4 – DERIVATIVE OF TRANSCENDENTAL FUNCTION

SAQ18
ACTIVITY 4.15 – A

NAME: ____________________________________________________ SCORE: ______________


SECTION: ___________DATE: _______________ PROF: __________________________________

Differentiate each of the following using the appropriate differentiation formula.

1. 1
6
3

2. 3 (2 3 1) 7 2

3. √1 2 8 √4

4 2 2 9.

5. 10.

Differential Calculus Module 15 – Derivative of Trigonometric Functions and their Applications Page 174
UNIT 4 – DERIVATIVE OF TRANSCENDENTAL FUNCTION

II. Find of the given functions.

1.

2.

III. Differentiate the given functions using the appropriate differentiation method.
1.

2. 3

3. ( ) ( ) 4 [Hint: Use sum to product identity to simplify .]

4. 2 ; 4

5. 4 2 ; 2

Differential Calculus Module 15 – Derivative of Trigonometric Functions and their Applications Page 175
UNIT 4 – DERIVATIVE OF TRANSCENDENTAL FUNCTION

IV. Solve for the unknown.

1. Find equation of the tangent to the graph of ( ) at .

2. Find the values of x on the graph of ( ) 2 for 0 2 where the tangent line
has a slope of 2.

3. A particle moves along a coordinate axis in such a way that its position at time t is given by
( ) √3 2 for 0 2 . At what time is the particle at rest?

4. An observer watches a rocket launched from a distance of 3 km. The angle of elevation is
increasing at 4 at the instant 45 . How fast is the rocket climbing at that instant?

5. If a ladder of length 30 feet that is leaning against a wall has its upper edge sliding down the wall
at the rate of ft/sec, what is the rate of change of the measure of the acute angle made by the
ladder with the ground when the upper end is 18 feet above the ground?

Differential Calculus Module 15 – Derivative of Trigonometric Functions and their Applications Page 176
UNIT 4 – DERIVATIVE OF TRANSCENDENTAL FUNCTION

ASAQ18
ACTIVITY 4.15 – A

NAME: ____________________________________________________ SCORE: ______________


SECTION: ___________DATE: _______________ PROF: __________________________________

Differentiate each of the following using the appropriate differentiation formula.

1. 1
6
3
3 1 1
1 ( 3 )
2 3 3
2

2. 3 (2 3 1) 7 2
3(3 4 ) (2 3 1) 6 2 2

3. √1 2 8 √4
√1 2 √1 2 2√4
2(1 2 ) √1 2 2√4

4 2 2 9.
2( 2 )( 2 2 ) 1

5. 10.
(3 2) 2
(1 )

Differential Calculus Module 15 – Derivative of Trigonometric Functions and their Applications Page 177
UNIT 4 – DERIVATIVE OF TRANSCENDENTAL FUNCTION

II. Find of the given functions.

1. 3

2.

III. Differentiate the given functions using the appropriate differentiation method.

1.

2. 3

3. ( ) ( ) 4 [Hint: Use sum to product identity to simplify .]

4. 2 ; 4 2

5. 4 2 ; 2 2( )(1 )

Differential Calculus Module 15 – Derivative of Trigonometric Functions and their Applications Page 178
UNIT 4 – DERIVATIVE OF TRANSCENDENTAL FUNCTION

IV. Solve for the unknown.

1. Find equation of the tangent to the graph of ( ) at . 2 2

2. Find the values of x on the graph of ( ) 2 for 0 2 where the tangent line
has a slope of 2.

3. A particle moves along a coordinate axis in such a way that its position at time t is given by
( ) √3 2 for 0 2 . At what time is the particle at rest?

4. An observer watches a rocket launched from a distance of 3 km. The angle of elevation is
increasing at 4 at the instant 45 . How fast is the rocket climbing at that instant?

15

5. If a ladder of length 30 feet that is leaning against a wall has its upper edge sliding down the wall
at the rate of ft/sec, what is the rate of change of the measure of the acute angle made by the
ladder with the ground when the upper end is 18 feet above the ground?

Differential Calculus Module 15 – Derivative of Trigonometric Functions and their Applications Page 179
UNIT 4 – DERIVATIVE OF TRANSCENDENTAL FUNCTION

Differential Calculus Module 15 – Derivative of Trigonometric Functions and their Applications Page 180
UNIT 4 – DERIVATIVE OF TRANSCENDENTAL FUNCTION

II. Find of the given functions.

1.

2.

III. Differentiate the given functions using the appropriate differentiation method.
1.

2. 3

3. ( ) ( ) 4 [Hint: Use sum to product identity to simplify .]

4. 2 ; 4

5. 4 2 ; 2

Differential Calculus Module 15 – Derivative of Trigonometric Functions and their Applications Page 181
UNIT 4 – DERIVATIVE OF TRANSCENDENTAL FUNCTION

IV. Solve for the unknown.


1. Find equation of the tangent to the graph of ( ) at .

2. Find the values of x on the graph of ( ) 2 for 0 2 where the tangent line
has a slope of 2.

3. A particle moves along a coordinate axis in such a way that its position at time t is given by
( ) √3 2 for 0 2 . At what time is the particle at rest?

4. An observer watches a rocket launched from a distance of 3 km. The angle of elevation is
increasing at 4 at the instant 45 . How fast is the rocket climbing at that instant?

5. If a ladder of length 30 feet that is leaning against a wall has its upper edge sliding down the wall
at the rate of ft/sec, what is the rate of change of the measure of the acute angle made by the
ladder with the ground when the upper end is 18 feet above the ground?

Differential Calculus Module 15 – Derivative of Trigonometric Functions and their Applications Page 182
UNIT 4 – DERIVATIVE OF TRANSCENDENTAL FUNCTION

ACTIVITY 4.15 – B

NAME: ____________________________________________________ SCORE: ______________


SECTION: ___________DATE: _______________ PROF: __________________________________

I. Differentiate each of the following using the appropriate differentiation formula.


1. 3 3

1 1
2
4 4

3. √ (Hint: Use 1 )

4. (2 )

5. 1

6. 4

Differential Calculus Module 15 – Derivative of Trigonometric Functions and their Applications Page 183
UNIT 4 – DERIVATIVE OF TRANSCENDENTAL FUNCTION

II. Find the equation of the tangent line to the given curve at the indicated value of x.
1

4

2 ,

III. Find the relative maxima and minima of given the curve for 0 2 and roughly sketch the
graph of the given equations.
a.

Differential Calculus Module 15 – Derivative of Trigonometric Functions and their Applications Page 184
UNIT 4 – DERIVATIVE OF TRANSCENDENTAL FUNCTION

b. 4 8 , 0 180

IV. Solve the given time-rate problems.


1. Given that 3 and that x is changing at a rate of unit/sec, find the rate of
change of y when x= ⁄4.

2. The base of an isosceles triangle remains constant at 10 feet with its base angles decreasing at
a rate of 2 Find the rate by which its area changes when its base angles measure
45

Differential Calculus Module 15 – Derivative of Trigonometric Functions and their Applications Page 185
UNIT 4 – DERIVATIVE OF TRANSCENDENTAL FUNCTION

3. The measure of one of the acute angles of a right triangle is decreasing at the rate of
⁄ . If the length of the hypotenuse remains a constant at 40 cm, find how fast the area
is changing when the measure of the acute angle is .

4. Each of the two sides of a triangle are increasing at the rate of ⁄ with their included
angle decreasing at the rate of ⁄ . Find the rate of change of area when the sides and
the included angle are respectively 5 feet, 8 feet and 60

5. If a ladder of length 30 feet that is leaning against a wall has its upper end sliding down the wall
at the rate of ⁄ , what is the rate of change of the measure of the acute made by the
ladder with the ground when the upper end is 18 feet above the ground?

Differential Calculus Module 15 – Derivative of Trigonometric Functions and their Applications Page 186
MODULE 16
DERIVATIVES OF INVERSE
TRIGONOMETRIC FUNCTION
AND THEIR APPLICATIONS

Specific Objectives:

At the end of the module, students must be able to:

1. Know and use correctly the differentiation formulas for


inverse trigonometric function.

2. Apply the differentiation formulas for inverse


trigonometric function in solving problems on slope of
tangent and normal, rectilinear motion, angle between
curves, optimization problems and time-rates.
UNIT 4 – DERIVATIVE OF TRANSCENDENTAL FUNCTION

DERIVATIVES OF INVERSE TRIGONOMETRIC FUNCTIONS

Let be a differentiable function of x.


1. ( ) ( )

( ) ( )
√ √

3. ( ) ( )

PRINCIPAL VALUES OF INVERSE TRIGONOMETRIC FUNCTIONS

Let us have a brief review of the principal values of an angle. The table below
shows the principal value of the inverse trigonometric functions in their domain and the
corresponding range.

Function Domain Range

Sin–1x [–1, 1]

Cos–1x [–1, 1] [0, π]

Tan–1x (–∞, ∞)

Cot–1x (–∞, ∞) (0, π)

Sec–1x (–∞, -1]  [1, ∞)

Csc–1x (–∞, -1]  [1, ∞)

In Calculus, Sin-1 x, Cos-1 x and Tan-1 x are the most important inverse
trigonometric functions. However, there is often disagreement in the choice of principal
value for inverse secant and inverse cosecant. Some authors define the value as

between   and  for negative value of trigonometric functions secant and cosecant.
2
Relative to this and in as much that they are seldom used, these inverse trigonometric
functions may be conveniently avoided.

Differential Calculus Module 16 – Derivative of Inverse Trigonometric Function and Their Applications Page 186
UNIT 4 – DERIVATIVE OF TRANSCENDENTAL FUNCTION

Example 17. Find x, if given.


1. ( )
( )
√ ( )

( )
√ ( )

2. √
(√ )
√ √(√ )

[ √ ]
√ √( )

[ ( )]
√ √ √

√ ( )

( )√

3. ( )

( )
( )

( ) ( ) ( ) ( )
[ ]
( ) ( )
( )
( ) ( )( ) ( )
* +
( ) ( ) ( )

Differential Calculus Module 16 – Derivative of Inverse Trigonometric Function and Their Applications Page 187
UNIT 4 – DERIVATIVE OF TRANSCENDENTAL FUNCTION

( )


4. when

[ ] ( )


√ √
( )

√ √

( ) ( ) ( )

( ) ( )

( )


( )√

( )
( )

* +

( )
( )

Differential Calculus Module 16 – Derivative of Inverse Trigonometric Function and Their Applications Page 188
UNIT 4 – DERIVATIVE OF TRANSCENDENTAL FUNCTION

( ) ( ) ( ) ( )

( )[ ] ( ) ( )( )
( )

( )[ ]( ) ( )

( )[ ]( )

Example 18. Find the equation of the tangent line to the curve at point
( )
Slope of tangent line at any point on the curve is

* ( )+ * ( )+ ( )

* + [ ( )] ( )

[ ] [ ( )] ( )

( )
( )

At point ( ) .
Use the point-slope form to get equation of the tangent line.
( )
( )

( )
( )
( ) ( )

Differential Calculus Module 16 – Derivative of Inverse Trigonometric Function and Their Applications Page 189
UNIT 4 – DERIVATIVE OF TRANSCENDENTAL FUNCTION

Example 19. A ladder 15 feet long leans against a vertical wall. If the top slides down at
2 feet/sec, how fast is the angle of elevation of the top of the ladder
decreasing as observed from its foot, when the lower end is 12 feet from
the wall? Use Inverse Trigonometric Function.
Let be the distance of the foot of the ladder from the wall at any time
be the distance of the top of the ladder from the ground at any time
be the angle of elevation of the top of the ladder as observed from its
foot at any time

Find the equation that relates variables . We use the definition of either sin or
csc Let us choose csc

Differentiate the above equation with respect to

( )

( ) √( )

[ ( )]
( )√

[ ( )]

[ ]
√ √

When √( ) ( ) √ √

Therefore, ( )
√ √

The negative sign of time rate of the angle of elevation of the top of the ladder,
means that as the top of the ladder slides down the wall is decreasing with time.

Differential Calculus Module 16 – Derivative of Inverse Trigonometric Function and Their Applications Page 190
UNIT 4 – DERIVATIVE OF TRANSCENDENTAL FUNCTION

Example 20. A kite is 60 feet high with 100 feet of cord out. If the kite is moving
horizontally 4 mi/hr directly away from the boy flying it, find the rate of
change of the angle of elevation of the cord. Solve using an inverse
trigonometric function.
The variables in the problem are the length of the cord that is out the horizontal
distance of the kite from the boy and the angle of elevation of the cord at anytime .
The height of the kite is fixed at 60 feet.
The equation relating the variables is

( )

( )
( )

( )

( )

When √ ( ) √( ) ( )

√ √ feet
( ) ( )
( )
( )

The negative sign of implies that as the cord of the kite lengthens, angle is
decreasing with time.

Example 21. A ship, moving 8 mi/hr, sails north for 30 min, then turns east. If a
searchlight at the point of departure follows the ship, how fast is the light
rotating 2 hr after the start.
Let be the total number of hours sailed by the ship
be the acute angle the searchlight makes with the vertical.

Differential Calculus Module 16 – Derivative of Inverse Trigonometric Function and Their Applications Page 191
UNIT 4 – DERIVATIVE OF TRANSCENDENTAL FUNCTION

The distance travelled when it sailed north for 30 minutes (0.5 hour) = ( )
. The distance travelled east for( ) = ( ) ( )
( )
( )

( )

( )
[ ( )]

( )
* +

Two hours after leaving the point of departure, ( )


.

Therefore, the searchlight following the ship from the point of departure is rotating at the
rate of

Example 22.The lower edge of the picture is 5 feet, the upper edge is 12 feet, above
the eye of an observer. At what horizontal distance should he stand, if the
angle subtended by the picture is to be the greatest?
Let be the horizontal distance the observer needs to stand so that the
angle subtended by the picture is the greatest.
On the Figure below, ---------------- Equation (1)

But, ---------- Equation (2)

------Equation (3)

Substitute Equation (2) and Equation (3) into Equation (1).

12
5

Differential Calculus Module 16 – Derivative of Inverse Trigonometric Function and Their Applications Page 192
UNIT 4 – DERIVATIVE OF TRANSCENDENTAL FUNCTION

Differentiate with respect to ( ) ( )


( ) ( )

[ ] [ ]
( ) ( )

For the subtended angle to be the greatest,

* + * +
( ) ( )
( )( )

√ ( ) √ feet

Therefore, for the subtended angle by the picture to be the greatest, the horizontal
distance of the observer from the picture should be √ feet.

Differential Calculus Module 16 – Derivative of Inverse Trigonometric Function and Their Applications Page 193
UNIT 4 – DERIVATIVE OF TRANSCENDENTAL FUNCTION

SAQ19
ACTIVITY 4.16 – C

NAME: ____________________________________________________ SCORE: ______________


SECTION: ___________DATE: _______________ PROF: __________________________________

I. Find of the given functions.


1.

2. √

3.

4.

5. √

Differential Calculus Module 16 – Derivative of Inverse Trigonometric Function and Their Applications Page 194
UNIT 4 – DERIVATIVE OF TRANSCENDENTAL FUNCTION

6. ( )

7. √

II. Evaluate at the indicated value of .


1. √ ;

2. √ ;

3. ;

4. ;

Differential Calculus Module 16 – Derivative of Inverse Trigonometric Function and Their Applications Page 195
UNIT 4 – DERIVATIVE OF TRANSCENDENTAL FUNCTION

III. Find the equation of the tangent line to the given graph of the given function at the indicated
value of x.
1. ( )

2. ( )

IV. Find the angle of intersection between curves ( ) .

V. A movie screen on the front wall of your classroom is high an above your
H w w m m “b w”?

Differential Calculus Module 16 – Derivative of Inverse Trigonometric Function and Their Applications Page 196
UNIT 4 – DERIVATIVE OF TRANSCENDENTAL FUNCTION

VI. The lower edge of a mural 12 feet high, b b ’ I m b


view is obtained when the angle subtended by the mural at the observer is a maximum, how far
should the observer stand from the wall?

VII. A ladder 25 feet long is leaning against a vertical wall. If the bottom of the ladder is pulled
horizontally away from the wall so that the top of the ladder is sliding down at ⁄ , how fast is
the measure of the measure of the acute angle between the ladder and the ground changing when
the bottom of the ladder is 15 feet from the wall?

VIII. The position of a particle at time is given by ( ) ( ) for Find the velocity
when .

Differential Calculus Module 16 – Derivative of Inverse Trigonometric Function and Their Applications Page 197
UNIT 4 – DERIVATIVE OF TRANSCENDENTAL FUNCTION

ASAQ19
ACTIVITY 4.16 – C

NAME: ____________________________________________________ SCORE: ______________


SECTION: ___________DATE: _______________ PROF: __________________________________

I. Find of the given functions.


1.

2. √ √

3.

4.

5. √ √

Differential Calculus Module 16 – Derivative of Inverse Trigonometric Function and Their Applications Page 198
UNIT 4 – DERIVATIVE OF TRANSCENDENTAL FUNCTION

( )
6. ( ) ( )

√ ( )
7. √ √ ( )

II. Evaluate at the indicated value of .


1. √ ;

2. √ ;

3. ;

4. ;

Differential Calculus Module 16 – Derivative of Inverse Trigonometric Function and Their Applications Page 199
UNIT 4 – DERIVATIVE OF TRANSCENDENTAL FUNCTION

III. Find the equation of the tangent line to the given graph of the given function at the indicated
value of x.
1. ( ) ( )

2. ( ) √ √

IV. Find the angle of intersection between curves ( ) .

V. A movie screen on the front wall of your classroom is high an above your
H w w m m “b w”?

Differential Calculus Module 16 – Derivative of Inverse Trigonometric Function and Their Applications Page 200
UNIT 4 – DERIVATIVE OF TRANSCENDENTAL FUNCTION

VI. The lower edge of a m b b ’ I m b


view is obtained when the angle subtended by the mural at the observer is a maximum, how far
should the observer stand from the wall? √

VII. A ladder 25 feet long is leaning against a vertical wall. If the bottom of the ladder is pulled
horizontally away from the wall so that the top of the ladder is sliding down at ⁄ , how fast is
the measure of the measure of the acute angle between the ladder and the ground changing when
the bottom of the ladder is 15 feet from the wall?

VIII. The position of a particle at time is given by ( ) ( ) for Find the velocity
when .

Differential Calculus Module 16 – Derivative of Inverse Trigonometric Function and Their Applications Page 201
UNIT 4 – DERIVATIVE OF TRANSCENDENTAL FUNCTION

ACTIVITY 4.16 – D

NAME: ____________________________________________________ SCORE: ______________


SECTION: ___________DATE: _______________ PROF: __________________________________

Solve the following problems using derivative of inverse trigonometric functions.

1. If √ , at what value of is the slope of the curve equal to ?

2. Find the equation of the tangent line to the curve represented by at the point
where .

3. A picture m w w b m b the level of the eye of the


observer. If the observer is approaching the wall at the rate of m⁄ , how fast is the measure of
the angle subtended at the observer’s eye by the picture changing when the observer is 1 meter from
the wall?

4. Two vertical poles respectively 1 meter and 9 meters high are 6 meters apart. How far from the
foot of the shorter pole where the line segment joining the tops of the poles subtends the greatest
angle?

Differential Calculus Module 16 – Derivative of Inverse Trigonometric Function and Their Applications Page 202
MODULE 17
DERIVATIVES OF LOGARITHMIC
AND EXPONENTIAL FUNCTION
AND THEIR APPLICATIONS

Specific Objectives:

At the end of the module, students must be able to:

1. Know and use correctly the differentiation formulas for


logarithmic and exponential function.

2. Apply the differentiation formulas for logarithmic and


exponential function in solving problems on slope of
tangent and normal, rectilinear motion, angle between
curves, optimization problems and time-rates.
UNIT 4 – DERIVATIVE OF TRANSCENDENTAL FUNCTION

DERIVATIVES OF LOGARITHMIC AND EXPONENTIAL FUNCTIONS


Let is a differentiable function of

1. ( ) ( )

( ) ( )

SYSTEMS OF LOGARITHM

The most common bases of logarithms are the base 10 and the base e.

a. Logarithms with a base 10 are called common logarithms. It is also known as the decadic
logarithm and also as the decimal logarithm, named after its base, or Briggsian logarithm,
after Henry Briggs, an English mathematician who pioneered its use. When the base is 10,
we can leave off the 10 in the notation.

b. Logarithms with a base e ( Napierian constant = 2.718281…) are called natural logarithm
or Napierian logarithm first defined by John Napier.

PROPERTIES OF LOGARITHMS
1.
2. (i.e., )
3.

4.

5.

6. √

7.

8. (This is known as the Change of Base Formula)

Example 23. Find and simplify whenever possible.


[ ( )]
√ √

Differential Calculus Module 17 – Derivative of Exponential and Logarithmic Function and Their Applications Page 203
UNIT 4 – DERIVATIVE OF TRANSCENDENTAL FUNCTION

( )
( )

b.

( )

[ ]

[( )( )]

( )( )

( )

[( ) ( )] ( )( )

( )( )

( )

( )

* +

[ ( )]

[ ( )]

[ ( ) ( ) ( )]

[ ( )( ) ]

Differential Calculus Module 17 – Derivative of Exponential and Logarithmic Function and Their Applications Page 204
UNIT 4 – DERIVATIVE OF TRANSCENDENTAL FUNCTION

[ ]

( )( )

( )

f.

( ) ( ) ( )

( )( )

( )

( )

( )

( ( ))
( )

[ ( )( )]
( )

( ) ( )

( )

( ) ( )


( )( )

j.

( )

( )( )

Differential Calculus Module 17 – Derivative of Exponential and Logarithmic Function and Their Applications Page 205
UNIT 4 – DERIVATIVE OF TRANSCENDENTAL FUNCTION

( )

( ) ( ) ( ) ( )
[ ]
( )

( )( )( ) ( )( )( )
( )( )
( )

k.

( )

( )

( )

l. ( )
( ) ( )

( )( ) ( )+( ) ( )
( )( )

( ) ( )
( )

( )

( )√

( )√

Differential Calculus Module 17 – Derivative of Exponential and Logarithmic Function and Their Applications Page 206
UNIT 4 – DERIVATIVE OF TRANSCENDENTAL FUNCTION

m. ( )

[ ]
( )
( )
( ) ( ) ( ) ( )( ) ( )
[ ]
( )
( ) ( ) ( )
[ ]
( )
( ) [ ]
( )
( )
( )

n. ( )
[ ( ) ( )]
( )
( )( )( )
( )

o. ( )
( ) ( )

( )( ) ( ) ( ) ( )( )

( )( )
( ) ( )

( ) ( )
( )

( )

( )

( )√

Differential Calculus Module 17 – Derivative of Exponential and Logarithmic Function and Their Applications Page 207
UNIT 4 – DERIVATIVE OF TRANSCENDENTAL FUNCTION

Example 24. Given that ( ) , find the equation of the normal line when

Solution: ( )

Transform the given logarithmic equation to equivalent exponential equation.

------------ Equation (1)

.
When ( ) .
Equation of the normal line to the curve at ( )

( )

Alternative Solution: This alternative solution is to derive Equation (1).

( )

( )
( )

Hence, (Same as Equation (1))

Example 25. Show that the curve has its maximum point at ( ).

Solution:

Differentiate. ( )

( )

( )

[ ]

Set each factor to zero.

(Rejected since has no value)

Differential Calculus Module 17 – Derivative of Exponential and Logarithmic Function and Their Applications Page 208
UNIT 4 – DERIVATIVE OF TRANSCENDENTAL FUNCTION

When

Perform the second derivative test. ( ) * +( )( )

[ ( )]

( )

When ( )( )

Therefore, point ( )is a relative maximum point of the curve .

Example 26. If ( ) and if the rate of change of is 4 units per second, find the rate of change
of when .

Solution: ( ) ( ) ( )

( )

( )[ ]

When , substitute on the given equation of the curve.

( )

Therefore, when and ,

( )[ ] ( )[ ]

( )[ ] ( )( )

units/second

Differential Calculus Module 17 – Derivative of Exponential and Logarithmic Function and Their Applications Page 209
UNIT 4 – DERIVATIVE OF TRANSCENDENTAL FUNCTION

Example 27. Find the critical points and roughly sketch the curve .

( )

At the critical point,


Solve the logarithmic equation.

Solve the corresponding value of ( )


Hence, the critical point is( ).
Use the second derivative test to classify the critical point. ( ) .
When ( ) ( ).
Therefore, ( ) is a minimum point.

Find the point of inflection by setting .

Therefore, the curve has no point of inflection.


To help roughly sketch the curve, find the intercepts. When ( )
When
Equate each factor to zero. . This value is rejected since
Likewise,

Hence, the curve passes through point( ).

Differential Calculus Module 17 – Derivative of Exponential and Logarithmic Function and Their Applications Page 210
UNIT 4 – DERIVATIVE OF TRANSCENDENTAL FUNCTION

SAQ20
ACTIVITY 4.17 – E

NAME: ____________________________________________________ SCORE: ______________


SECTION: ___________DATE: _______________ PROF: __________________________________

I. Differentiate the following functions.


1. ( )

2. ( )

3.

4. ( )

Differential Calculus Module 17 – Derivative of Exponential and Logarithmic Function and Their Applications Page 211
UNIT 4 – DERIVATIVE OF TRANSCENDENTAL FUNCTION

( )

II. Find the y’’’ of function .

Differential Calculus Module 17 – Derivative of Exponential and Logarithmic Function and Their Applications Page 212
UNIT 4 – DERIVATIVE OF TRANSCENDENTAL FUNCTION

III. If ( ), show .

IV. Find the equation of the tangent line to the given curve ( ) at

V. For the curve , find the equation of tangent line parallel to the line .

VI. Find the acute angle of intersection between the given pair of curves.
1.

2.

Differential Calculus Module 17 – Derivative of Exponential and Logarithmic Function and Their Applications Page 213
UNIT 4 – DERIVATIVE OF TRANSCENDENTAL FUNCTION

3. ( ) ( )

VII. Find and classify the critical point of the given function.

1. .

2. .

VIII. Let ( ) for . If is increasing at a constant rate of ⁄ , find the rate at


which is changing when

Differential Calculus Module 17 – Derivative of Exponential and Logarithmic Function and Their Applications Page 214
UNIT 4 – DERIVATIVE OF TRANSCENDENTAL FUNCTION

ASAQ20
ACTIVITY 4.17 – E

NAME: ____________________________________________________ SCORE: ______________


SECTION: ___________DATE: _______________ PROF: __________________________________

I. Differentiate the following functions.


( )

( )
( )

( )( )

( )

( )

( )

( )( )

Differential Calculus Module 17 – Derivative of Exponential and Logarithmic Function and Their Applications Page 215
UNIT 4 – DERIVATIVE OF TRANSCENDENTAL FUNCTION

( )

( )
( )

( )

II. Find the y’’’ of function . ( )

Differential Calculus Module 17 – Derivative of Exponential and Logarithmic Function and Their Applications Page 216
UNIT 4 – DERIVATIVE OF TRANSCENDENTAL FUNCTION

III. If ( ), show .

IV. Find the equation of the tangent line to the given curve ( ) at

V. For the curve , find the equation of tangent line parallel to the line .

VI. Find the acute angle of intersection between the given pair of curves.
3.

4.

Differential Calculus Module 17 – Derivative of Exponential and Logarithmic Function and Their Applications Page 217
UNIT 4 – DERIVATIVE OF TRANSCENDENTAL FUNCTION

3. ( ) ( )

VII. Find and classify the critical point of the given function.

1. . ( )

2. . ( ) ( )

VIII. Let ( ) for . If is increasing at a constant rate of ⁄ , find the rate at


which is changing when

Differential Calculus Module 17 – Derivative of Exponential and Logarithmic Function and Their Applications Page 218
UNIT 4 – DERIVATIVE OF TRANSCENDENTAL FUNCTION

ACTIVITY 4.17 – F

NAME: ____________________________________________________ SCORE: ______________


SECTION: ___________DATE: _______________ PROF: __________________________________

I. Differentiate using the appropriate formula or method.

1. ( )

2.

3. ( )

4.

5.

Differential Calculus Module 17 – Derivative of Exponential and Logarithmic Function and Their Applications Page 219
UNIT 4 – DERIVATIVE OF TRANSCENDENTAL FUNCTION

II. Find equation of the tangent line to the curve ( ) at .

III. Find the angle of intersection between the given pair of curves.
1.

2. ( )

IV. Find the point of inflection of the curve .

V. Find and classify the critical points, find point of inflection and sketch the curve .

Differential Calculus Module 17 – Derivative of Exponential and Logarithmic Function and Their Applications Page 220
MODULE 18
DERIVATIVE OF A VARIABLE
RAISED TO ANOTHER VARIABLE

Specific Objectives:

At the end of the module, students must be able to:

1. Understand the derivation of the formula to differentiate


a variable raised to a variable.

2. Apply the differentiation formula for derivative of a


variable with a variable exponent
UNIT 4 – DERIVATIVE OF TRANSCENDENTAL FUNCTION

DERIVATIVE OF A VARIABLE WITH A VARIABLE EXPONENT

Let where both are functions of x. To derive the differentiation formula for
let us take the logarithm of both sides of the given equation, then, differentiate implicitly with
respect to x.

Differentiate using the power formula.

Differentiate using the product formula. ( )

Apply the formula ( ) ( )

Multiply both sides of the equation above, then, replace by .

* ( ) +

* ( ) +

𝑑𝑦 𝑑𝑢 𝑑𝑣
𝑣𝑢𝑣 𝑢𝑣 𝑢
𝑑𝑥 𝑑𝑥 𝑑𝑥

The boxed-formula above is best to use to differentiate a variable raised to another variable.

Example28. Find , given ( )

Solution: To differentiate, one may opt to use the formula above or do the step by step process as
shown on the derivation of the above formula. Let me show both ways.
Method 1. Using the formula,

Apply the derived differentiation formula.


( ) ( )

Method 2. Follow the step by step process as shown on the derivation of the formula.
( )
Take the logarithm of both sides of the given equation. ( )

Apply

Differentiate implicitly. * ( )+ ( )( )

Multiply both sides of equation by * +

Differential Calculus Module 18 – Derivative of A Variable Raised to Another Variable Page 221
UNIT 4 – DERIVATIVE OF TRANSCENDENTAL FUNCTION

But, ( ) . ( ) * +
( )( )
Apply distributive law. ( )

Use . ( )( ) ( )

Apply commutative law. ( ) ( )

Observe that both methods of differentiation yield same .

Example 29. F ’( ) e ( )

Method 1.

Use the formula.

( )( ) ( ) ( ) ( )( )
( )( ) ( ) ( )
Add similar terms. ( ) ( )
Method 2. ( )
( )( ) ( )
( ) ( )

( )( )

( ) ( )
But ( ) . ( )

Example 3. Find ( ) given ( )

Use the formula.

( )( ) ( ) ( ) ( )( )

Simplify. ( ) [( ) ( ) ]

Differential Calculus Module 18 – Derivative of A Variable Raised to Another Variable Page 222
UNIT 4 – DERIVATIVE OF TRANSCENDENTAL FUNCTION

But, ( ) . ( ) [ ]

Method 2. ( )

( ) ( )

* ( )+ ( )( )

* ( ) ( )( )+

( ) [ ( ) ( )( )]

( ) [ ( ) ]

Example 30. What positive number when raised to itself will give a minimum value?

Solution: Let be the positive number and be theminimum value of the number raised to itself.
That is,

( )( )

( )

( )

F e e of to be a minimum, ( )

Equate each factor to zero.

(Rejected since has no value)

Differential Calculus Module 18 – Derivative of A Variable Raised to Another Variable Page 223
UNIT 4 – DERIVATIVE OF TRANSCENDENTAL FUNCTION

SAQ21
ACTIVITY 4.18 – G

NAME: ____________________________________________________ SCORE: ______________

SECTION: ___________DATE: _______________ PROF: __________________________________

Find

1. ( )

2. ( )

3. ( )

Differential Calculus Module 18 – Derivative of A Variable Raised to Another Variable Page 224
UNIT 4 – DERIVATIVE OF TRANSCENDENTAL FUNCTION

ASAQ21
ACTIVITY 4.18 – G

NAME: ____________________________________________________ SCORE: ______________

SECTION: ___________DATE: _______________ PROF: __________________________________

Find

1. ( ) ( )

2. ( ) ( ) ( )

3. ( ) ( ) ( )

Differential Calculus Module 18 – Derivative of A Variable Raised to Another Variable Page 225
MODULE 19
CONCEPT OF INDETERMINATE
FORMS

Specific Objectives:

At the end of the module, students must be able to:

1. Recognize when to apply L’ Hospital’s Rule.

2. Identify indeterminate forms produced by quotient,


products, difference, powers and apply L’ Hospital’s Rule.
UNIT 5 – INDETERMINATE FORMS

CONCEPT OF INDETERMINATE FORMS

In Chapter 1, we experienced getting limit of expression equal to o as


approaches a certain value, say . This limit is a meaningless symbol and is called
indeterminate form. In Calculus, evaluating limit that involves indeterminate form is
better facilitated using derivatives. Application (or repeated application) of L’ Hospital’s
Rule, (pronounced: lopiˈtal) named after 17th-century French mathematician Guillaume
de l'Hôpital (also written L'Hopital) often converts an indeterminate form to a
determinate form, hence, allowing easy evaluation of limits. Other indeterminate forms
are n

L’ Hospital’s Rule: If and are both continuous and differentiable functions


in an open interval containing , except possibly at and if n are
both when or when both approach hen , provided
n
𝑓 𝑥 𝑓 𝑥
lm lm 𝐿
𝑥 𝑎 𝑔 𝑥 𝑥 𝑎 𝑔 𝑥

provided n o o . Moreover, this statement is also true in the case of a


limit as

We will discuss limit evaluation resulting to any of the indeterminate forms


; and, 00 , 0 , 1.

A. Indeterminate Forms n
When limit of a given function takes the indeterminate form n apply L’Hospital’s
Rule repeatedly until limit becomes a finite number. Let us see how to do it from the
illustrative examples.

I. Evaluate the limit, if it exists. Differentiate separately the numerator


𝑥 𝑆𝑖𝑛 1
𝑥 and denominator since the limit is
1.l m
𝑥 𝑥3

Reapply L’ Hospital’s
lm 𝑥2
Rule since limit is again
𝑥 3𝑥 2

2𝑥
𝑥2 2 𝑥2
Reapply L’ Hospital’s Rule. lm
𝑥 6𝑥

Differential Calculus Module 19 – Concept of Indeterminate Forms Page 226


UNIT 5 – INDETERMINATE FORMS

𝑥
Simplify. 𝑥2 𝑥2
lm lm 3 The finite value of
𝑥 6𝑥 𝑥 6
6 𝑥2 2
𝑆𝑖𝑛 𝑥1 𝑥
𝑥𝑥 𝑆𝑖𝑛
2 l𝑥lm
m
𝑠𝑒𝑐 2 𝑥 2 n𝑥 2 2 𝑥 𝑥𝑥3 3
2. lm
1
𝑥 𝜋 co 4𝑥
4
2
2 c𝑥 c𝑥 n𝑥 2𝑠𝑒𝑐 2 𝑥 2𝑠𝑒𝑐 2 𝑥 n 𝑥 𝑠𝑒𝑐 2 𝑥 n𝑥 2
lm
𝑥
1
𝜋 4 n 4𝑥 4 n 4𝑥 n 4𝑥
4

𝑠𝑒𝑐 2 𝑥 𝑠𝑒𝑐 2 𝑥 n𝑥 2𝑠𝑒𝑐 2 𝑥 n𝑥 Use of L’ Hospital’s


lm [ ]
1
𝑥 𝜋 2 4 co 4𝑥 Rule yields
4

2 2 2 2 Reapply L’
[ ] Hospital’s Rule
2

ln co 2𝑥 ln co 2𝜋 ln
3. lm 2
𝑥 𝜋 𝜋 𝑥 2 𝜋 𝜋 Result of
using
L’ Hospital’s
n 2𝑥 2 n 2𝑥
l m co 2𝑥
Rule
lm
𝑥 𝜋 2 𝜋 𝑥 𝑥 𝜋 𝜋 𝑥
𝑠𝑒𝑐 2 2𝑥 2 2 2
lm 2
𝑥 𝜋
Reapplying L’ Hospital’s Rule
yields finite limit of value -2
4. 𝑥 ln 𝑥 ln
lm
𝑥 co 𝑥
𝑥
𝑥 ln 𝑥 ln 𝑥 ln
lm 𝑥 lm 𝑥
𝑥 n𝑥 𝑥 n𝑥 n

𝑥 𝑥 ln 𝑥 𝑥 𝑥 ln 𝑥
lm lm
𝑥 x n𝑥 𝑥 𝑥 co 𝑥 n𝑥

ln 𝑥 2 ln 𝑥 2 ln
lm lm 2
𝑥 x co 𝑥 n𝑥 𝑥 x co 𝑥 n𝑥

𝑒𝑥 2 2
5. 𝑥l m 𝑥 n𝑥
2 𝑒𝑥 𝑒𝑥 2 𝑒𝑥 𝑒𝑥 2
lm lm
𝑥 𝑥 co 𝑥 n𝑥 𝑥 𝑥 co 𝑥 n𝑥
lm
2 𝑒𝑥 𝑒 𝑥
𝑒𝑥 𝑒𝑥 2𝑒 𝑥 𝑒 𝑥 𝑒 𝑥 2 2
lm lm
𝑥 𝑥 n𝑥 co 𝑥 co 𝑥 𝑥 𝑥 n𝑥 2 co 𝑥 2
2
2
Differential Calculus Module 19 – Concept of Indeterminate Forms Page 227
UNIT 5 – INDETERMINATE FORMS

𝑒𝑥 𝑒 𝑥
6. 𝑥l m ln 𝑥

𝑒𝑥 𝑒 𝑥
lm lm 𝑒𝑥 𝑒 𝑥
𝑥 2
𝑥 𝑥
𝑥l m

ln 𝑥 11 ln
7. 𝑥 l m ln𝑥𝑥
l m lm
𝑥 𝑥

1 𝑥9
lm 𝑥 lm
𝑥 lm 𝑥 𝑥
lm

2𝑥 2
l
8. 𝑥 m lm
4𝑥 2 𝑥
4𝑥
lm
𝑥 𝑥
4 lm
lm
𝑥 2
lm
2𝑥
𝑒
9. lm
𝑥 𝑥 3l m
2𝑒 2𝑥
lm
𝑥 3𝑥 2
4𝑒 2𝑥 l m
lm
𝑥 6𝑥
lm
𝑒 2𝑥
lm Lm o no x
𝑥 6 6 6
lm

ln 𝑥
10. lm lm
𝑥 1 𝑒
𝑒𝑥

𝑥 𝑥
lm 1 lm 1
𝑥
l m𝑥 𝑒
𝑒𝑥 𝑒𝑥
𝑥2

lm

Differential Calculus Module 19 – Concept of Indeterminate Forms Page 228


UNIT 5 – INDETERMINATE FORMS

B. Indeterminate Form

If functions n as , the difference is said to


assume the indeterminate form The technique to evaluate 𝑥l m𝑎 𝑓 𝑥 𝑔 𝑥 is to
rewrite the given difference into a quotient. This is done through subtraction of
fractions, that is , and on the result, apply L’Hospital’s Rule.

II. Evaluate the limit, if it exists, Since limit is , transform


1 1
to quotient form to
1. 𝑥l m1 ln 𝑥 𝑥
ln 𝑥 𝑥 1
allow the use of L’ Hospital’s Rule
𝑥 ln 𝑥
lm
𝑥 1 𝑥 ln 𝑥
lm 𝑥
lm 𝑥 lm 𝑥
𝑥 1 𝑥 1 𝑥 𝑥 ln 𝑥
𝑥 𝑥 lnl 𝑥
m Simplify the expression
𝑥
resulting from the use of
𝑥 L’ Hospital’s Rule
lm
𝑥 1 𝑥 𝑥 ln 𝑥
lm
lm lm
𝑥 1
𝑥 𝑥 ln 𝑥
𝑥 1 ln 𝑥 2

Apply
L’ Hospital’s Rule the 2nd time.
5 5 1
2. 𝑥l m2 𝑥 2 𝑥 6 𝑥 2
Result is finite limit of 2.

5 𝑥 2 𝑥2 𝑥 6 4 4𝑥 𝑥 2
lm lm
𝑥 2 𝑥 2 𝑥2 𝑥 6 𝑥 2 𝑥 2 𝑥2 𝑥 6
4 2𝑥 4 2𝑥
lm lm
𝑥 2 𝑥 2 2𝑥 𝑥2 𝑥 6 𝑥 2 3𝑥 2 2𝑥

2 2
lm
𝑥 2 6 2𝑥 5

l m𝜋 𝜋 c𝑥 2𝑥 n𝑥 𝜋 2
3. 𝑥
2
𝜋
2 𝑥 co 𝑥 n𝑥 2 2 2
l m𝜋 2
𝑥 n𝑥
2

Differential Calculus Module 19 – Concept of Indeterminate Forms Page 229


UNIT 5 – INDETERMINATE FORMS

2 2
4. lm
𝑥 𝑠𝑖𝑛2 𝑥 co 𝑥
2 2
lm
𝑥 𝑠𝑖𝑛2 𝑥 co 𝑥
2 co 𝑥 𝑠𝑖𝑛2 𝑥 2
lm [ ]
𝑥 𝑠𝑖𝑛2 𝑥 co 𝑥
2 n𝑥 2 n 𝑥 co 𝑥
lm [ 2 ]
𝑥 𝑠𝑖𝑛 𝑥 n𝑥 co 𝑥 2 n 𝑥 co 𝑥
2 n co
lm 2 2
n 2 co 2

2 co 𝑥
lm [ 2 ]
𝑥 𝑠𝑖𝑛 𝑥 2 co 𝑥 2𝑐𝑜𝑠 2 𝑥
2 n𝑥
lm
𝑥 2 n 𝑥 co 𝑥 2 n 𝑥 4 co 𝑥 n𝑥
2 n𝑥 2 n𝑥
lm lm
𝑥 2 n 𝑥 6 n 𝑥 co 𝑥 𝑥 2 n𝑥 3 co 𝑥

3 2

C. Indeterminate Form

If functions n nc o lm as then,
the product assumes the indeterminate form Again, transformation
needs to be done on the product in order to bring it to a quotient. This
can be done in either of the two ways below.

or

Then, apply L’ Hospital’s Rule. Since limit is


, transform
3
III. Evaluate the limit, if it exists. 𝑥 n 𝑥 to
3 3 quotient form
1. lm𝑥 n n n
𝑥 𝑥 to allow the use
3 of L’ Hospital’s
n𝑥 n
lm Rule
𝑥
𝑥

Differential Calculus Module 19 – Concept of Indeterminate Forms Page 230


UNIT 5 – INDETERMINATE FORMS

3 3
co 3
lm
𝑥 𝑥2 l m 3 co 3 co 3 3
𝑥 𝑥 𝑥
𝑥2
Result of using
2. l m+ 𝑥 ln 𝑥 ln L’ Hospital’s Rule
𝑥

ln 𝑥
l m+ l m+ 𝑥 l m+ 𝑥
𝑥 𝑥 𝑥
𝑥 𝑥2
𝑥
3. l m 𝑥𝑒 𝑒
𝑥 𝑒
𝑥 𝑥
lm lm
𝑥 𝑥 𝑒𝑥
𝑒𝑥
Since limit is , transform 𝑥 3 ln 𝑥
lm to quotient form to allow the use of L’
𝑥 𝑒𝑥 𝑒
Hospital’s Rule
4. l m 𝑥 3 ln 𝑥
𝑥

ln 𝑥 𝑥 𝑥3
lm lm lm
𝑥 𝑥 3 𝑥 3 3
𝑥3 𝑥4

5. 𝑥l m𝜋 n𝑥 c 2𝑥
4

n𝑥
l m𝜋
𝑥 co 2𝑥
4
2
𝑠𝑒𝑐 2 𝑥 2
l m𝜋
𝑥 n 2𝑥 2 2
4

2
𝑠𝑒𝑐 2 𝑥 2 2
6. l ml m
c c 𝜋𝑥 ln 𝑥
𝑥 1 𝜋 n 2𝑥 2 2 2
𝑥
4

ln 𝑥 𝑥
lm lm
𝑥 1 n 𝜋𝑥 𝑥 1 co 𝜋𝑥 𝜋 𝜋 𝜋

D. Indeterminate Forms 0 0 ,  0 , 1
𝑔 𝑥 𝑔 𝑥
Limits of the form 𝑥l m𝑎 𝑓 𝑥 or 𝑥l m 𝑓 𝑥 frequently give rise to indeterminate

of the types Indeterminate forms of these types can be evaluated as follows:

Differential Calculus Module 19 – Concept of Indeterminate Forms Page 231


UNIT 5 – INDETERMINATE FORMS

(1). Let ------------- Equation (1)

(2). Take the logarithm of both sides of the Equation in (1).

Thus, ln ln ln

(3). Take 𝑥l m𝑎 ln 𝑦 l m 𝑔 𝑥 ln 𝑓 𝑥 . The limit on the right-hand side of the equation


𝑥 𝑎

will usually be an indeterminate limit of the type 0   . Evaluate this limit using the
technique previously described. Assume that 𝑥l m𝑎 𝑔 𝑥 ln 𝑓 𝑥 𝐿
𝑔 𝑥
(4). Finally, if l m ln 𝑦 𝐿, then, l m 𝑓 𝑥 𝑒𝐿
𝑥 𝑎 𝑥 𝑎

IV. Solve the limit, if it exists.


1 1
1. l m co. 𝑥 𝑥
𝑥
1 Step (1)
L 𝑦 co 𝑥 𝑥 Step (2)
1
ln 𝑦 ln co 𝑥 𝑥 ln co 𝑥
𝑥
Step (3)
l m ln 𝑦 lm ln co 𝑥
𝑥 𝑥 𝑥
ln co 𝑥
𝐿 l m ln 𝑦 lm
𝑥 𝑥 𝑥

n𝑥
𝐿 l m co 𝑥
𝑥
Step (4)
Therefore, 1
𝐿
T l m co 𝑥 𝑥 𝑒 𝑒
𝑥

1 1
2. l m 𝑒𝑥 3𝑥 𝑥 3
𝑥
1
L 𝑦 𝑒𝑥 3𝑥 𝑥

1
ln 𝑦 ln 𝑒 𝑥 3𝑥 𝑥 ln 𝑒 𝑥 3𝑥
𝑥
𝑥
𝐿 l m ln 𝑦 𝑒l𝑥m ln3𝑥𝑒 3𝑥 ln
𝑥 𝑥 𝑥

𝑥
𝐿 l m𝑒
𝑥 3𝑥 𝑒 3𝑥 𝑒 3 3
4
𝑥 𝑒
Therefore, 1
l m 𝑒𝑥 3𝑥 𝑥 𝑒𝐿 𝑒4
𝑥

Differential Calculus Module 19 – Concept of Indeterminate Forms Page 232


UNIT 5 – INDETERMINATE FORMS

𝑥
3. lm
𝑥 𝑥
𝑥
L 𝑦
𝑥
𝑥
ln 𝑦 ln 𝑥 ln
𝑥 𝑥

𝐿 l m ln 𝑦 l m 𝑥 ln ln ln
𝑥 𝑥 𝑥

ln 𝑥
𝐿 l m ln 𝑦 lm
𝑥 𝑥
𝑥

𝑥2
𝐿 lm 𝑥
𝑥
𝑥2
𝑥

Therefore, 𝑥l m 𝑒𝐿 𝑒1 𝑒
𝑥

𝜋 1
tan 𝑥
4. 𝑥l m1 𝑥 2
𝜋
L 𝑡𝑦 𝑥 tan 2
𝑥

𝜋 𝜋
ln 𝑦 ln 𝑥 tan 2
𝑥
n 𝑥 ln 𝑥
2
𝜋
𝐿 l m ln 𝑦 lm n 𝑥 ln 𝑥
𝑥 1 𝑥 1 2
ln 𝑥 𝑥 2
𝐿 lm 𝜋 lm 𝜋 𝜋 𝜋
𝑥 co 𝑥 𝑥 1 2
𝑐𝑠𝑐 2 𝑥 𝜋
2 2 2
𝜋 2
Therefore, l m 𝑥 tan 2 𝑥 𝑒𝐿 𝑒 𝜋
𝑥 1

.
cos 𝑥
5. lm
𝜋−
n𝑥
𝑥
2
cos 𝑥
L 𝑡𝑦 n𝑥

cos 𝑥
ln 𝑦 ln n𝑥 co 𝑥 ln n𝑥

𝐿 lm
𝜋−
ln 𝑦 lm
𝜋−
co 𝑥 ln n𝑥 ln ln
𝑥 𝑥
2 2

Differential Calculus Module 19 – Concept of Indeterminate Forms Page 233


UNIT 5 – INDETERMINATE FORMS

2
𝐿 lm
ln n 𝑥
lm n 𝑥 𝑠𝑒𝑐 𝑥 lm
c𝑥
𝜋− c𝑥 𝜋− c𝑥 n𝑥 𝜋− 𝑡𝑎𝑛2 𝑥
𝑥 𝑥 𝑥
2 2 2

c𝑥 n𝑥
𝐿 lm
𝜋−
lm
𝜋−
𝑥 2 n 𝑥𝑠𝑒𝑐 2 𝑥 𝑥 2 c𝑥 2
2 2

cos 𝑥
Therefore, l m
𝜋−
n𝑥 𝑒𝐿 𝑒
𝑥
2

Differential Calculus Module 19 – Concept of Indeterminate Forms Page 234


UNIT 5 – INDETERMINATE FORMS

SAQ22
ACTIVITY 5.19 – A

NAME: ____________________________________________________ SCORE: ______________

SECTION: ___________DATE: _______________ PROF: __________________________________

Evaluate the limit, if it exists.


𝑥5 𝑥4 𝑥3 𝑥2 𝑥
1. l m
𝑥 2𝑥 5 𝑥 4 𝑥 3 𝑥

𝑥 2
2. 3.
lm
𝑥 2 ln 𝑥 3

3. l m 𝑥3
𝑥 𝑥 n𝑥

4. 𝑥2
lm 𝑥
𝑥 𝑒

5. ln c𝑥
lm
𝑥 𝑥2

Differential Calculus Module 19 – Concept of Indeterminate Forms Page 235


UNIT 5 – INDETERMINATE FORMS

6. 𝑥4
lm
𝑥 𝑒𝑥

1 1
7. l m𝑥 +
𝑥 sin 𝑥

4
8. l m
𝑥 2 𝑥2 4 𝑥 2

9.
l m 𝑒𝑥 co 𝑥
𝑥

10. l m+ 2𝑥 n 𝜋𝑥
1
𝑥
2

Differential Calculus Module 19 – Concept of Indeterminate Forms Page 236


UNIT 5 – INDETERMINATE FORMS

1
11. l m ln 𝑥 𝑥
𝑥

𝑥
12. lm
𝑥 𝑥

tan 𝑥
13. l m+ n𝑥
𝑥

𝑥2
14. lm 𝑥
𝑥

1
15. l m+ 𝑥 𝑥
𝑥

Differential Calculus Module 19 – Concept of Indeterminate Forms Page 237


UNIT 5 – INDETERMINATE FORMS

ASAQ22
ACTIVITY 5.19 – A

NAME: ____________________________________________________ SCORE: ______________

SECTION: ___________DATE: _______________ PROF: __________________________________

Evaluate the limit, if it exists.


𝑥5 𝑥4 𝑥3 𝑥2 𝑥 1
l
1. 𝑥 m
2𝑥 5 𝑥 4 𝑥 3 𝑥 2

𝑥 2
lm
2. 𝑥3. 2 ln 𝑥 3

𝑥3
3. l m 6
𝑥 𝑥 n𝑥

𝑥2
4. 𝑥l m 𝑥
𝑒

ln c𝑥 1
5. l m
𝑥 𝑥2 2

Differential Calculus Module 19 – Concept of Indeterminate Forms Page 238


UNIT 5 – INDETERMINATE FORMS

6. 𝑥4
lm
𝑥 𝑒𝑥

1 1
7. l m𝑥 +
𝑥 sin 𝑥

4 1
8. l m
𝑥 2 𝑥2 4 𝑥 2 4

9. l m 𝑒 𝑥 co 𝑥
𝑥

l m+ 2𝑥 n 𝜋𝑥 2
10. 1
𝑥
2

Differential Calculus Module 19 – Concept of Indeterminate Forms Page 239


UNIT 5 – INDETERMINATE FORMS

1
11. l m ln 𝑥 𝑥
𝑥

𝑥
12. lm
𝑥 𝑥

tan 𝑥
13. l m+ n𝑥 2
𝑥

𝑥2
14. lm 𝑥
𝑥

1
15. l m+ 𝑥 𝑥
𝑥

Differential Calculus Module 19 – Concept of Indeterminate Forms Page 240


UNIT 5 – INDETERMINATE FORMS

ACTIVITY 5.19 – B

NAME: ____________________________________________________ SCORE: ______________

SECTION: ___________DATE: _______________ PROF: __________________________________

Evaluate the limit, if it exists.


5𝑥 n 5𝑥
1. l m
𝑥 𝑥3

2. lm 𝑥2
𝑥
n 𝑥

𝑒𝑥 𝑒 𝑥
3. lm
𝑥 ln 𝑥

4 𝑥 4 𝑥
4. lm
𝑥 𝑥

5. l m+
𝑥 4𝑥 𝑒 4𝑥

Differential Calculus Module 19 – Concept of Indeterminate Forms Page 241


UNIT 5 – INDETERMINATE FORMS

6. l m 𝑥 2 𝑒 3𝑥
𝑥

2 𝑥
7. lm
𝑥 2 ln 𝑥2 3 ln 𝑥2 3

𝑥
4
8. lm
𝑥 𝑥

1
9. l m+ 𝑥 𝑥
𝑥

10. l m co 𝑥 𝑥
𝑥

Differential Calculus Module 19 – Concept of Indeterminate Forms Page 242


MODULE 20
CONCEPT OF PARTIAL
DIFFERENTIATION

Specific Objectives:

At the end of the module, students must be able to:

1. Recognize when to differentiate partially.

2. Find the higher order derivative of a function of more


than one independent variable.
UNIT 6 – PARTIAL DIFFERENTIATION

PARTIAL DIFFERENTIATION

From the previous chapters, we only dealt with derivatives of the functions with
respect to only one independent variable. However, not all functions are dependent on
only one variable; there are functions in which a variable may be dependent to more
than one independent variable. In finding the derivative of such function, we will still be
differentiating it with respect to one of the independent variables while holding the other
independent variables as constants. Such differentiation is called a partial
differentiation. Suppose a function exists where the independent variables
are x and y and z being the dependent variable, then, the partial derivative of a function
with respect to variable x is given by and the partial derivative of the same
function with respect to variable y is given by . In the derivative , all variable y’s in
the function of z are held constant and, in the derivative , all variable x’s in the
function of z are held constant.

Example 1. In the functions given below, perform the corresponding partial derivatives
with respect to all its independent variable.

a.
b.
c.

Solution:
a.
Differentiating with respect to x means holding all variable y’s constant.

Notice below that when we do partial differentiation with respect to y, all variable x’s are
held constant.

b.

Again, observe above that when we do partial differentiation with respect to x, all
variable y’s are held constant.

Differential Calculus Module 20 – Concept of Partial Derivative Page 243


UNIT 6 – PARTIAL DIFFERENTIATION

In partially differentiating with respect to y, all variable x’s are held constant as
observed above.

c.

Note that when we get the partial derivative of with respect to x, the variables y
and z are considered constants, so then, the derivative of is 0.

Notice that when we do partial differentiation with respect to y, all variable x’s and z’s
are held constant. Thus, the partial derivative of with respect to y is 0.

Observe that the partial derivative of with respect to z, x and y are considered
constants, thus, the derivative of is 0.

HIGHER ORDER PARTIAL DIFFERENTIATION


Now that we have learned the basics of single partial derivative, we can study its
higher ordered differentiations.

Example 2. Find , given .

Differentiate with respect to x by holding y as a constant.


[ ]

Differential Calculus Module 20 – Concept of Partial Derivative Page 244


UNIT 6 – PARTIAL DIFFERENTIATION

Differentiate partially with respect to x holding y a constant to find .

( )

[ ]

Differentiate with respect to y by holding x as a constant.


[ ]

Differentiate partially with respect to y holding x a constant to find .

( ) [ ]

[ ]

Differentiate with respect to y to get , holding x a constant.


( ) [ ] [ ]

Differentiate with respect to x to get , holding y a constant.


( ) [ ] [ ]

Notice that . It is because that the partial derivatives are continuous


otherwise if the partial derivatives are discontinuous, .

Example 3. In the function , determine the following partial

differentiations: .

Given: ,

For , ( ) * +

Differential Calculus Module 20 – Concept of Partial Derivative Page 245


UNIT 6 – PARTIAL DIFFERENTIATION

[ ]

Therefore,

For , ( ) * +

[ ]

Therefore,

Differential Calculus Module 20 – Concept of Partial Derivative Page 246


UNIT 6 – PARTIAL DIFFERENTIATION

ASAQ23
ACTIVITY 6.20 – A

NAME: ____________________________________________________ SCORE: ______________

SECTION: ___________DATE: _______________ PROF: __________________________________

In the functions shown below, determine the partial derivatives with respect to each
independent variable.

1.

2.

3. √ √

4.

Differential Calculus Module 20 – Concept of Partial Derivative Page 247


UNIT 6 – PARTIAL DIFFERENTIATION

ASAQ23
ACTIVITY 6.20 – A

NAME: ____________________________________________________ SCORE: ______________

SECTION: ___________DATE: _______________ PROF: __________________________________

In the functions shown below, determine the partial derivatives with respect to each
independent variable.

1.

2.
𝜕𝑓 𝜕𝑓
𝐴𝑛𝑠𝑤𝑒𝑟 𝑥 𝑥𝑡𝑎𝑛 𝑦 𝑥𝑠𝑖𝑛 𝑦 𝑥 𝑦𝑠𝑒𝑐 𝑦 𝑐𝑜𝑠 𝑥 𝑦𝑠𝑖𝑛 𝑦
𝜕𝑥 𝜕𝑦

3. √ √
√ √ √ √
√ √

4.

Differential Calculus Module 20 – Concept of Partial Derivative Page 248


UNIT 6 – PARTIAL DIFFERENTIATION

ACTIVITY 6.20 – B

NAME: ____________________________________________________ SCORE: ______________

SECTION: ___________DATE: _______________ PROF: __________________________________

In the functions shown below, determine the corresponding partial differentiations.


1.

2.

3.

Differential Calculus Module 20 – Concept of Partial Derivative Page 249


BIBLIOGRAPHY
Books
Catolos, Elvira C. Worktext in Differential Calculus. University of Rizal System. 2018
Love, Clyde E. and Earl D. Rainville. Differential and Integral Calculus. Sixth Edition.
The Macmillan Company: 1969
Leithold, Louis. The Calculus with Analytic Geometry. 6th Edition Harper and Row
Publishers: 1990
Peterson, Thurman S. Calculus with Analytic Geometry. Harper and Row Publishers:
1969
Protter, Murray H. and Philip E. Protter. Calculus with Analytic Geometry. Fourth
Edition. Jones and Bartlett Publishers: 1988
Committee on Calculus. Calculus (Differential and Integral) Part I. Royal Publishing
House, Inc.: 1962
Reyes, Francisco G. and Jenny L. Chua. Elements of Calculus and Analytic Geometry.
University of Santo Tomas: 1984
Ayres, Frank, Jr. Schaums Outline Series. Theory and Problems of Differential and
Integral Calculus. 2nd ed. McGook of Company.1964
Sy, Jose C. and Ricardo C. Asin.Calculus Reviewer (ifferential Calculus). Merriam and
Webster Bookstore, Inc:1991.

Internet
http://math.about.com/cs/calculus/g/calculusdef.htm
http://home.windstream.net/okrebs/page191.html
http://www.regentsprep.org/Regents/math/algtrig/ATP5/Lfunction.htm
http://www.mathalino.com/reviewer/differential-calculus/application-of-maxima-and-
minima

http://whatis.techtarget.com/definition/increment

https://www.math.ucdavis.edu/~kouba/CalcOneDIRECTORY/invtrigderivsoldirectory/Inv
TrigDerivSol2.html#SOLUTION 12
http://connectatkmtc.wordpress.com/application-of-differentiation/
https://sakai.wfu.edu/access/.../Limits%20(L'Hospital's%20Rule)

Differential Calculus Page 250

You might also like